Esophagus and Diaphragm Flashcards

1
Q

A 42-year-old male for whom you recently performed a
successful inguinal hernia repair, presents to your clinic
with a chief complaint of increasing difficulty swallowing some solids and occasionally liquids. He reports
this has been worsening for the last 5 months. He has
been treated by his primary care physician with daily
omeprazole, but this does not seem to be improving his
symptoms. Since you did such a good job in repairing
his hernia, he is now coming to you for advice on this
problem. He appears healthy, his vital signs are normal,
and he is maintaining his normal weight.

  1. The most appropriate initial study to help you sort
    through the differential diagnosis of this patients
    dysphagia is?

A. Contrast esophagram

B. Esophageal manometry

C. Esophagogastroduodenoscopy

D. H. pylori testing

E. Chest X-ray

A

A. Contrast esophagram offers the most useful information in initially sorting through this differential.

The esophagram shown essentially eliminates
diverticula and a hiatal hernia.

How well did you know this?
1
Not at all
2
3
4
5
Perfectly
2
Q

A contrast esophagram image is shown (showing contrast cut-off at the mid to distal esophagus).

What additional study or studies should be ordered
next in the diagnostic workup of this patient?

A. Esophageal manometry

B. Esophagogastroduodenoscopy (EGD)

C. 24-hour pH monitoring

D. 24-hour pH monitoring and EGD

E. Esophageal manometry and EGD

A

B.

This esophagram would typically be followed
up with an EGD.

While the esophagram shown makes the diagnosis of a diverticula or an hiatal hernia less likely, there are still several diagnoses that are not ruled out or confirmed.

An EGD would be needed to exclude anatomical causes for dysphagia, such as neoplasm or stricture.

The EGD would also be used to document any evidence of reflux or eosinophilic esophagitis.

If there is no reflux seen on EGD, a 24-hour pH study could confirm the presence of functional reflux disease.

If the 24-hour pH study is normal, then a manometry study could be done to rule out motility disorders like achalasia, diffuse esophageal spasm,
connective tissue disorders, and nutcracker esophagus.

Even if the 24-hour pH study is confirmatory, though, an esophageal manometry should be
done to rule out a concominant underlying motility disorder.

By skipping the EGD and proceeding directly to manometry, a diagnosis, such as pseudoachalasia secondary to a distal anatomic partial obstruction, may be missed.

How well did you know this?
1
Not at all
2
3
4
5
Perfectly
3
Q
  1. Upper endoscopy reveals no evidence of esophagitis, but some food is present in the distal esophagus. Manometry shows aperistalsis and a non-relaxing LES. Which initial treatment offers the patient the best chance of long term-relief?

A. Pneumatic dilation of the LES

B. Calcium channel blocker therapy

C. Esophageal myotomy with fundoplication

D. Botulinum toxin injection at LES

E. Esophagectomy

A

C.

This patient has achalasia. In the absence of an obstructing entity (neoplasm, hiatal hernia, diverticuli) retained food in the esophagus is suspicious for achalasia.

The diagnosis is confirmed by manometry. In patients with achalasia, the manometry demonstrates an esophagus in which there is complete absence of peristalsis and an LES pressure that is normal to moderately elevated, but fails to completely relax.

Although a hypertensive LES and an LES that fails to completely relax are often associated with achalasia, only the complete absence of peristalsis is required for the diagnosis.

The pathophysiology of achalasia is loss of ganglion cells in the myenteric plexus and interruption of inhibitory vagal nerve innervation.

The treatment for achalasia requires relaxing the hypertensive smooth muscles of the LES. In general, that can be done surgically by dividing the muscles of the LES, injecting botulinum toxin endoscopically, or dilating the LES using a balloon.

This is also done endoscopically. Surgical treatment with myotomy provides long-term treatment of achalasia with a high success rate.

Calcium channel blockers have shown inconsistent success and do not have a role in achalasia treatment.

Botulinum toxin usually requires repeated interventions every 6 to 12 months, as does
dilation. Both eventually become less effective. These may be good options in persons who are poor operative risks or in those who do not want surgery.

Esophagectomy would be required in the setting of cancer and is indicated as a last resort if multiple myotomies fail.

The POEM or per oral endoscopic myotomy technique involves dividing the LES muscle endoscopically. This has the advantage of avoiding most surgical risks. The long-term success rate, though, is unknown and is not yet considered the equivalent of a surgical myotomy.

How well did you know this?
1
Not at all
2
3
4
5
Perfectly
4
Q

Suppose in this same patient, that the results of the manometry do not show achalasia. Instead, the standard esophageal manometry shows 10/10 normal propagated swallows with a mean distal esophageal amplitude pressure of 293mm Hg.

The lower esophageal sphincter pressure is normal and relaxes completely. The results of ambulatory pH monitoring and EGD were both normal.

What is the most likely diagnosis?

A. Achalasia

B. Pseudoachalsia

C. Diffuse esophageal spasm

D. Nutcracker esophagus

A

D.

Manometry is used to rule out esophagility motility disorders. Not only does it measure how well food travels down the esophagus into the stomach, it also measures the pressure inside the esophagus and the LES.

The normal pressure of an LES is 10 to 15 mmHg.

Patients with GERD will often have a hypotensive LES with a pressure of around 5 to 8 mm Hg and normal to moderately abnormal peristalsis. The low
LES pressure is thought to allow gastric contents to reflux back into the esophagus.

Twenty-four-hour ambulatory manometry can be used to diagnose spastic disorders such as Diffuse Esophageal Spasm (DES), nutcracker esophagus, or hypercontractile esophageal motility disorder is a rare cause of dysphagia.

The nutcracker esophagus is characterized by very high LES pressures (>50 mm Hg) during swallowing with otherwise normal peristalsis.

DES would be characterized by high pressures throughout the esophagus (25-50 mm Hg) and poor peristalsis.

Pseudoachalasia would show a slightly hypertensive but relaxing LES and abnormal but not absent peristalsis.

Connective tissue disorders, like scleroderma, would be hallmarked by poor peristalsis and a normal LES.

How well did you know this?
1
Not at all
2
3
4
5
Perfectly
5
Q

Locations of anatomic narrowing of the esophagus seen on an esophagram include all of the following EXCEPT

A. Lower esophageal sphincter

B. Crossing of the left mainstem bronchus and aortic arch

C. Thoracic outlet

D. Cricopharyngeal muscle

A

Answer: C

Three normal areas of esophagea narrowing are evident on the barium esophagogram or during esophagoscopy.

The uppermost narrowing is located at the entrance into the esophagus and is caused by the cricopharyngeal muscle. Its luminal diameter is 1.5 cm, and it is the narrowest point of the esophagus.

The middle narrowing is due to an indenta- tion of the anterior and left lateral esophagea wall caused by the crossing of the left main stem bronchus and aortic arch. The luminal diameter at this point is 1.6 cm.

The lowermost narrowing is at the hiatus of the diaphragm and is caused by the gastroesophageal sphincter mechanism. The luminal diameter at this point varies somewhat, depending on the distention of the esophagus by the passage of food, but has been measured at 1.6 to 1.9 cm.

These norma constrictions tend to hold up swallowed foreign objects, and the overying mucosa is subject to injury by swallowed corrosive liquids because of their slow passage through these areas.

(See Schwartz 10th ed., p. 942.)

How well did you know this?
1
Not at all
2
3
4
5
Perfectly
6
Q

The cervical esophagus receives its blood supply primarily from the

A. Internal carotid artery

B. Inferior thyroid artery

C. Superior thyroid artery

D. Inferior cervical artery

E. Facial artery

A

Answer: B

The cervical portion of the esophagus receives its main blood supply from the inferior thyroid artery.

The thoracic portion receives its blood supply from the bronchial arteries, with 75% of individuals having one right-sided and two left-sided branches.

Two esophageal branches arise directly from the aorta.

The abdominal portion of the esophagus receives its bood supply from the ascending branch of the left gastric artery and from inferior phrenic arteries. On entering the wall of the esophagus, the arteries assume a T-shaped division to form a longitudinal plexus, giving rise to an intramural vascular network in the muscular and submucosal layers. As a consequence, the esophagus can be mobilized from the stomach to the level of the aortic arch without fear of devascularization and ischemic necrosis.

Caution should be exercised as to the extent of esophageal mobilization in patients who have had a previous thyroidectomy with ligation of the inferior thyroid arteries proximal to the origin of the esophageal branches.

(See Schwartz 10th ed., pp. 945–946.)

How well did you know this?
1
Not at all
2
3
4
5
Perfectly
7
Q

All of the following cranial nerves are involved in the swallowing mechanism EXCEPT

A. V

B. VII

C. VIII

D. X

E. XI

F. XII

A

Answer: C

Swallowing can be started at will, or it can be reflexively elicited by the stimulation of areas in the mouth and pharynx, among them the anterior and posterior tonsillar pillars or the posterior lateral walls of the hypopharynx.

The afferent sensory nerves of the pharynx are the glossopharyngeal nerves and the superior laryngeal branches of the vagus nerves.

Once aroused by stimuli entering via these nerves, the swallowing center in the medulla coordinates the complete act of swallowing by discharging impulses through cranial nerves V, VII, X, XI, and XII, as well as the motor neurons of C1 to C3.

Discharges through these nerves occur in a rather specific pattern and last for approximately 0.5 seconds. Little is known about the organization of the swallowing center, except that it can trigger swallowing after a variety of different inputs, but the response is always a rigidly ordered pattern of outflow.

Following a cerebrovascular accident, this coordinated outflow may be altered, causing mild to severe abnormalities of
swallowing.

In more severe injury, swallowing can be grossly disrupted, leading to repetitive aspiration.

(See Schwartz 10th ed., p. 948.)

How well did you know this?
1
Not at all
2
3
4
5
Perfectly
8
Q

All of these are parts of the human antireflux mechanism EXCEPT

A. Adequate gastric reservoir

B. Mechanically functioning lower esophageal sphincter (LES)

C. Mucus secreting cells of the distal esophagus

D. Efficient esophageal clearance

A

Answer: C

If the pharyngeal swallow does not initiate a peristaltic contraction, then the coincident relaxation of the lower esophageal sphincter (LES) is unguarded and reflux of gastric juice can occur.

This may be an explanation for the observation of spontaneous lower esophageal relaxation, thought by some to be a causative factor in gastroesophageal reflux disease (GERD).

The power of the worm-drive pump of the esophageal body is insufficient to force open a valve that does not relax.

In dogs, a bilateral cervical parasympathetic blockade abolishes the relaxation of the LES that occurs with pharyngeal swallowing or distention of the esophagus. Consequenty, vagal function appears to be important in coordinating the relaxation of the LES with esophageal contraction.

The antireflux mechanism in human beings is composed of three components: a mechanically effective LES, efficient esophageal clearance, and an adequately functioning gastric reservoir.

A defect of any one of these three components can lead to increased esophageal exposure to gastric juice and the deveopment of mucosal injury.

(See Schwartz 10thed.,p.949.)

How well did you know this?
1
Not at all
2
3
4
5
Perfectly
9
Q

Physiologic reflux happens most commonly when a person is

A. Awake and supine

B. Awake and upright

C. Asleep and supine

D. Asleep and semi-erect

A

Answer: B

On 24-hour esophageal pH monitoring, healthy individuals have occasional episodes of gastroesophageal reflux.

This physiologic reflux is more common when awake and in the upright position than during sleep in the supine position.

When reflux of gastric juice occurs, normal subjects rapidly clear the acid gastric juice from the esophagus regardless of their position.

There are several explanations for the observation that physiologic reflux in norma subjects is more common when they are awake and in the upright position than during sleep in the supine position.

First, reflux episodes occur in healthy volunteers primarily during transient losses of the gastroesophageal
barrier, which may be due to a relaxation of the LES or intragastric pressure overcoming sphincter pressure.

Gastric juice can also reflux when a swallow-induced relaxation of the LES is not protected by an oncoming peristaltic wave.

The average frequency of these “unguarded moments” or of transient losses of the gastroesophageal barrier is far less while asleep and in the supine position than while awake and in the upright position. Consequently, there are fewer opportunities for reflux to occur in the supine position.

Second, in the upright position, there is a 12-mm Hg pressure gradient between the resting, positive intra-abdominal pressure measured in the stomach and the most negative intrathoracic pressure measured in the esophagus at midthoracic level. This gradient favors the flow of gastric juice up into the thoracic esophagus when upright. The gradient diminishes in the supine position.

Third, the LES pressure in normal subjects is significantly higher in the supine position than in the upright position. This is due to the apposition of the hydrostatic pressure of the abdomen to the abdominal portion of the sphincter when supine.

In the upright position, the abdominal pressure surrounding the sphincter is negative compared with atmospheric pressure, and, as expected, the abdominal pressure gradually increases the more caudally it is measured.

This pressure gradient tends to move the gastric contents toward the cardia and encourages the occurrence of reflux into the esophagus when the individual is upright.

In contrast, in the supine position, the gastro- esophageal pressure gradient diminishes, and the abdominal hydrostatic pressure under the diaphragm increases, causing an increase in sphincter pressure and a more competent cardia.

(See Schwartz 10th ed., p. 949

How well did you know this?
1
Not at all
2
3
4
5
Perfectly
10
Q

All of the following hormones decrease LES tone EXCEPT

A. Gastrin

B. Estrogen

C. Somatostatin

D. CCK

E. Glucagon

A

Answer: A

The LES has intrinsic myogenic tone, which is modulated by neural and hormonal mechanisms.

Alpha-adrenergic neurotransmitters or beta-blockers stimulate the LES, and alpha-blockers and beta-stimulants decrease its pressure.

It is not clear to what extent cholinergic nerve activity controls LES pressure.

The vagus nerve carries both excitatory and inhibitory fibers to the esophagus and sphincter.

The hormones gastrin and motilin have been shown to increase LES pressure; and cholecystokinin, estrogen, glucagon, progesterone, somatostatin, and secretin decrease LES pressure.

The peptides bombesin, l-enkephalin, and substance P increase LES pressure; and calcitonin gene-related peptide, gastric inhibitory peptide, neuropeptide Y, and vasoactive intestinal polypeptide decrease LES pressure.

Some pharmacologic agents such as antacids, cholinergics, agonists, domperidone, metoclopramide, and prostaglandin F2 are known to increase LES pressure; and anticholinergics, barbiturates, calcium channel blockers, caffeine, diazepam, dopamine, meperidine, prostaglandin E1 and E2, and theophylline decrease LES pressure.

Peppermint, chocolate, coffee, ethanol, and fat are all associated with decreased LES pressure and may be responsible for esophageal symptoms after a sumptuous meal.

(See Schwartz 10th ed., pp. 949–950.)

How well did you know this?
1
Not at all
2
3
4
5
Perfectly
11
Q

The most common cause of a deficient LES is

A. Inadequate length

B. Mean resting pressure >6mmHg

C. Inadequate intraabdominal length

D. Failure of receptive relaxation

A

Answer: C

It is important that a portion of the total length of the LES be exposed to the effects of an intraabdominal pressure.

That is, during periods of elevated intraabdominal pressure, the resistance of the barrier would be overcome if pressure were not applied equally to both the LES and stomach simultaneously.

Thus, in the presence of a hiatal hernia, the sphincter resides entirely within the chest cavity and cannot respond to an increase in intra abdominal pressure, because the pinch valve mechanism is lost and gastroesophageal reflux is more liable to occur.

Therefore, a permanently defective sphincter is defined by one or more of the following characteristics:

1) An LES with a mean resting pressure of <6mmHg;
2) an overall sphincter length of <2cm; and
3) intraabdominal sphincter length of <1cm.

Compared to normal subjects without GERD, these values are below the 2.5 percentile for each parameter.

The most common cause of a defective sphincter is an inadequate abdominal length.

How well did you know this?
1
Not at all
2
3
4
5
Perfectly
12
Q

Maximal esophageal mucosal damage is caused by exposure to

A. Acidic fluid alone

B. Acidic fluid, food contents, and pepsin

C. Acidic fluid, trypsin, and food contents

D. Acidic fluid, pepsin, and bile salts

E. Neutral fluid, pepsin, and trypsin

A

Answer: D

The potential injurious components that reflux into the esophagus include gastric secretions such as acid and pepsin, as well as biliary and pancreatic secretions that regurgitate from the duodenum into the stomach.

There is a considerable body of experimental evidence to indicate that maximal epithelial injury occurs during exposure to bile salts combined with acid and pepsin.

These studies have shown that acid alone does minimal damage to the esophageal mucosa, but the combination of acid and pepsin is highly deleterious.

Similarly, the reflux of duodenal juice alone does little damage to the mucosa, although the combination of duodenal juice and gastric acid is particularly noxious.

(See Schwartz 10th ed., p. 967.)

How well did you know this?
1
Not at all
2
3
4
5
Perfectly
13
Q

The incidence of metaplastic Barrett esophagus (BE) progressing to adenocarcinoma is

A. Less than 0.1% per year

B. 0.2 to 0.5% per year

C. 1 to 3% per year

D. 3 to 5% per year

E. Greater than 5% per year

A

Answer: B

If reflux of gastric juice is allowed to persist, and sustained or repetitive esophageal injury occurs, two sequelae can result.

First, a luminal stricture can develop from submucosa and eventually intramural fibrosis.

Second, the tubular esophagus may become replaced with columnar epithelium. The columnar epithelium is resistant to acid and is associated with the alleviation of the complaint of heartburn. This columnar epithelium often becomes intestinalized, identified histologically by the presence of goblet cells. This specialized intestinal metaplasia (IM) is currently required for the diagnosis of Barrett esophagus (BE).

Endoscopically, BE can be quiescent or associated with complications of esophagitis, stricture, Barrett ulceration, and dysplasia. The complications associated with BE may be due to the continuous irritation from refluxed duodenogastric juice.

This continued injury is pH-dependent and may be modified by medical therapy.

The incidence of metaplastic Barrett epithelium becoming dysplastic and progressing to adenocarcinoma is approximatey 0.2 to 0.5% per year.

(See Schwartz 10th ed., pp. 968–969.)

How well did you know this?
1
Not at all
2
3
4
5
Perfectly
14
Q

The histologic hallmark of BE is

A. Columnar epithelium

B. Goblet cells

C. Parietal cells

D. Cuboidal epithelium

A

Answer: B

The definition of BE has evolved considerably over the past decade. Traditionally, BE was identified by the presence of columnar mucosa extending at least 3 cm into the esophagus.

It is now recognized that the specialized, intestinal type epitheium found in the Barrett mucosa is the only tissue predisposed to malignant degeneration.

Consequently, the diagnosis of BE is presently made given any length of endoscopically identifiable columnar mucosa that proves, on biopsy, to show IM.

Although long segments of columnar mucosa without IM do occur, they are uncommon and might be congenital in origin.

The hallmark of IM is the presence of intestinal goblet cells. There is a high prevalence of biopsy-demonstrated IM at the cardia, on the gastric side of the squamocolumnar junction, in the absence of endoscopic evidence of a columnar-lined esophagus (CLE).

Evidence is accumulating that these patches of what appears to be Barrett in the cardia have a similar malignant potential as in the longer segments, and are precursors for carcinoma of the cardia.

(See Schwartz 10thed.,p.969.)

How well did you know this?
1
Not at all
2
3
4
5
Perfectly
15
Q

Relief from respiratory symptoms can be expected in approximately what percent of patients with reflux associated asthma with medical therapy?

A. <10%

B. 25%

C. 50%

D. 75%

A

Answer: C
Once the diagnosis is established, treatment may be initiated with either proton pump inhibitor (PPI) therapy or antireflux surgery.

A trial of high-dose PPI therapy may help establish the facts that reflux is partly or completely responsible for the respiratory symptoms.

It is important to note that the persistence of symptoms in the face of aggressive PPI treatment does not necessarily rule out reflux as a possible cofactor or sole etiology.

Although there is probably some elements of a placebo effect, relief of respiratory symptoms can be anticipated in up to 50% of patients with reflux-induced asthma treated with antisecretory medications.

However, when examined objectively, <15% of patients can be expected to have improvement in their pulmonary function with medical therapy.

In properly selected patients, antireflux surgery improves respiratory symptoms in nearly 90% of children and 70% of adults with asthma and reflux disease.

Improvements in pulmonary function can be demonstrated in around 30% of patients.

Uncontrolled studies of the two forms of therapy (PPI and surgery) and the evidence from the two randomized controlled trials of medical versus surgical therapy indicate that surgical valve reconstruction is the most effective therapy for reflux-induced asthma.

The superiority of the surgery over PPI is most noticeable in the supine position, which corresponds with the nadir of PPI blood levels and resultant acid breakthrough and is the time in the circadian cycle when asthma symptoms are at their worst.

(See Schwartz 10th ed., p. 971.)

How well did you know this?
1
Not at all
2
3
4
5
Perfectly
16
Q

All of the following patients are good candidates for reflux surgery EXCEPT

A. A 31-year-old man with typical GERD with disease becoming resistant to medical therapy.

B. A 55-year-old woman with disease well-controlled with PPIs who wishes to discontinue medical therapy.

C. A 75-year-old man with new onset heartburn which is not relieved by PPIs.

D. A 52-year-old man with volume reflux and a large paraesophageal hernia.

A

Answer: C

Studies of the natural history of GERD indicate that most patients have a relatively benign form of the disease that is responsive to lifestyle changes and dietary and medical therapy, and do not need surgical treatment.

Approximately 25 to 50% of the patients with GERD have persistent or progressive disease, and it is this patient population that is best suited to surgical therapy.

In the past, the presence of esophagitis and a structurally defective LES were the primary indications for surgical treatment, and many internists and surgeons were reluctant to recommend operative procedures in their absence. However, one should not be deterred from considering antireflux surgery in a symptomatic patient with or without esophagitis or a defective sphincter, provided the disease process has been objectively documented by 24-hour pH monitoring.

This is particularly true in patients who have become dependent upon therapy with PPIs, or require increasing doses to control their symptoms.

It is important to note that a good response to medical therapy in this group of patients predicts an excellent outcome following antireflux surgery.

In general, the key indications for antireflux surgery are:

(a) objectively proven gastroesophageal reflux disease, and
(b) typical symptoms of gastroesophageal reflux disease (heartburn and/or regurgitation) despite adequate medical management, or
(c) a younger patient unwilling to take life-long medication.

In addition, a structurally defective LES can also predict which patients are more likely to fail with medical therapy.

Patients with normal sphincter pressures tend to remain well-controlled with medical therapy, whereas patients with a structurally defective LES may not respond as well to medical therapy, and often develop recurrent symptoms within 1 to 2 years of beginning therapy.

Such patients should be considered for an antireflux operation, regardless of the presence or absence of endoscopic esophagitis.

(See Schwartz 10th ed., p. 972.)

How well did you know this?
1
Not at all
2
3
4
5
Perfectly
17
Q

Preoperative testing for anti-reflux surgery typically includes all of the following EXCEPT

A. Computed tomography (CT) scan of the chest and abdomen

B. Contrast esophagram

C. 24 hour pH probe

D. Esophageal manometry

E. Esophagogastroduodenostomy

A

Answer: A

Before proceeding with an antireflux operation, several factors should be evaluated.

The clinical symptoms should be consistent with the diagnosis of gastroesophageal reflux.

Patients presenting with the typical symptoms of heartburn and/or regurgitation who have responded, at least partly, to PPI therapy, will generally do well following surgery, whereas patients with atypical symptoms have a less predictable response.

Reflux should also be objectively confirmed by either the presence of ulcerative esophagitis or an abnormal 24-hour pH study.

The propulsive force of the body of the esophagus should be evaluated by esophageal manometry to determine if it has sufficient power to propel a bolus of food through a newly reconstructed valve.

Patients with normal peristaltic contractions can be considered for a 360° Nissen fundoplication or a partial fundoplication, depending on patient and surgeon preferences.

When peristalsis is absent a partial fundoplication is probably the procedure of choice, but only if achalasia has been ruled out.

Hiatal anatomy should also be assessed. In patients with smaller hiatal hernias endoscopy evaluation usually provides sufficient information.

However, when patients present with a very large hiatus hernia or for revision surgery after previous antireflux surgery, contrast radiology provides better anatomical information.

The concept of anatomic shortening of the esophagus is controversial, with divergent opinions held about how common this problem is.

Believers claim that anatomic shortening of the esophagus compromises the ability of the surgeon to perform an adequate repair without tension, and that this can lead to an increased incidence of breakdown or thoracic displacement of the repair.

Some of those who hold this view claim that esophageal shortening is present when a barium swallow X-ray identifies a sliding hiatal hernia that will not reduce in the upright position, or that measures more than 5 cm in length at endoscopy.

When identified these surgeons usually undertake add a gastroplasty to the antireflux procedure.

Others claim that esophageal shortening is overdiagnosed and rarely seen, and that the morbidity of adding a gastroplasty outweighs any benefits.

These surgeons would recommend a standard antireflux procedure in a patients undergoing primary surgery.

(See Schwartz 10th ed., pp. 972–973.)

How well did you know this?
1
Not at all
2
3
4
5
Perfectly
18
Q

The valve created during an antireflux procedure should be at least

A. 1cm

B. 2cm

C. 3cm

D. 4cm

E. 5cm

A

Answer: C

The primary goal of antireflux surgery is to safely create a new antireflux valve at the gastroesophageal junction (GEJ), while preserving the patient’s ability to swallow normally and to belch to relieve gaseous distention.

Regardless of the choice of the procedure, this goal can be achieved if attention is paid to some basic principles when reconstructing the antireflux mechanism.

First, the operation should create a flap valve which prevents regurgitation of gastric contents into the esophagus.

This will result in an increase in the pressure of the distal esophageal sphincter region. Following a Nissen fundopication, the expected increase is to a level twice the resting gastric pressure (ie, 12 mm Hg or a gastric pressure oF 6 mm Hg).

The extent of the pressure rise is often less following a partial fundoplication, although with all types of fundoplication the length of the reconstructed valve should be at least 3 cm.

This not only augments sphincter characteristics in patients in whom they are reduced before surgery, but prevents unfolding of a normal sphincter in response to gastric distention.

Preoperative and postoperative esophageal manometry measurements have shown that the resting sphincter pressure and the overall sphincter length can be surgically augmented over preoperative values, and that the change in the former is a function of the degree of gastric wrap around the esophagus.

However, the aim of any fundoplication is to create a loose wrap, and to maintain the position of the gastric fundus close to the distal intra-abdominal esophagus, in a flap valve arrangement.

The efficacy of this relies on the close relationship between the fundus and the esophagus, not the“tightness” of the wrap.

(SeeSchwartz10thed.,p.973.)

How well did you know this?
1
Not at all
2
3
4
5
Perfectly
19
Q

A Toupet fundoplication involves

A. A 180° anterior wrap

B. A 90° posterior wrap

C. A 180° posterior wrap

D. A 270° posterior wrap

A

Answer: D

Partial fundoplications were developed as an alternative to the Nissen procedure in an attempt to minimize the risk of post fundoplication side effects, such as dysphagia, inability to belch, and flatulence.

The commonest approach has been a posterior partial or Toupet fundoplication. Some surgeons use this type of procedure for all patients presenting for anti-reflux surgery, whereas others apply a tailored approach in which a partial fundoplication is constructed in patients with impaired esophagea motility, in which the propulsive force of the esophagus is thought to be insufficient to overcome the outflow obstruction of a complete fundoplication.

The Toupet posterior partial fundoplication consists of a 270° gastric fundoplication around the distal 4 cm of esophagus.

It is usually stabilized by anchoring the wrap posteriorly to the hiatal rim.

(See Schwartz 10th ed., pp. 975–976.)

How well did you know this?
1
Not at all
2
3
4
5
Perfectly
20
Q

What percentage of patients should be expected to have relief of symptoms at 5 years out from antireflux surgery?

A. <50%

B. 50–60%

C. 60–80%

D. 80–90%

E. >90%

A

Answer: D

Studies of long-term outcome following both open and laparoscopic fundoplication document the ability of laparoscopic fundoplication to relieve typical reflux symptoms
(heartburn, regurgitation, and dysphagia) in more than 90% of patients at follow-up intervals averaging 2 to 3 years, and 80 to 90% of patients 5 years or more following surgery.

This incudes evidence-based reviews of antireflux surgery, prospective randomized trials comparing antireflux surgery to PPI therapy and open to laparoscopic fundoplication
and analysis of U.S. national trends in use and outcomes.

(See Schwartz 10th ed., p. 977.)

How well did you know this?
1
Not at all
2
3
4
5
Perfectly
21
Q

An upward dislocation of both the cardia and gastric fundus is which type of hiatal hernia?

A. I

B. II

C. III

D. IV

A

Answer: C

With the advent of clinical radiology, it became evident that a diaphragmatic hernia was a relatively common abnormality and was not always accompanied by symptoms.

Three types of esophageal hiatal hernia were identified:

(a) the sliding hernia, type I, characterized by an upward dislocation of the cardia in the posterior mediastinum;
(b) the rolling or paraesophageal hernia (PEH), type II, characterized by an upward dislocation of the gastric fundus alongside a normally positioned cardia; and
(c) the combined sliding-rolling or mixed hernia, type III, characterized by an upward dislocation of both the cardia and the gastric fundus.

The end stage of type I and type II hernias occurs when the whole stomach migrates up into the chest by rotating 180° around its longitudinal axis, with the cardia and pylorus as fixed points.

In this situation the abnormality is usually referred to as an intrathoracic stomach (Fig. 25-1).

In some taxonomies, a type IV hiatal hernia is declared when an additional organ, usually the colon, herniates as well. Type II-IV hiatal hernias are also referred to as paraesophageal hernia (PEH), as a portion of the stomach is situated adjacent to the esophagus, above the GEJ.

(See Schwartz 10th ed., Figure 25-39D, pp. 980–981.)

How well did you know this?
1
Not at all
2
3
4
5
Perfectly
22
Q

The most common form of esophageal cancer diagnosed in the United States is

A. Adenocarcinoma

B. Squamous carcinoma

C. Anaplastic carcinoma

D. Leiomyosarcoma

A

Answer: A

Adenocarcinoma of the esophagus, once an unusual malignancy, is diagnosed with increasing frequency and now accounts or more than 50% of esophageal cancer in most Western countries.

The shift in the epidemiology of esophageal cancer from predominantly squamous carcinoma seen in association with smoking and alcohol, to adenocarcinoma in the setting of BE, is one of the most dramatic changes that have occurred in the history of human neoplasia.

Although esophageal carcinoma is a relatively uncommon malignancy, its prevalence is exploding, largely secondary to the well-established association between gastroesophageal reflux, BE, and esophageal adenocarcinoma.

Once a nearly uniformly lethal disease, survival has improved slightly because of advances in the understanding of its molecular biology, screening and surveillance practices, improved staging, minimally invasive surgical techniques, and neoadjuvant therapy.

(See Schwartz 10th ed., p. 1003.)

How well did you know this?
1
Not at all
2
3
4
5
Perfectly
23
Q

Squamous cell carcinomas of the esophagus most commonly occur

A. At the GEJ

B. In the cervical and upper thoracic esophagus

C. In the lower thoracic esophagus

D. Evenly distributed throughout the esophagus

A

Answer: B

It is estimated that 8% of the primary malignant tumors of the esophagus occur in the cervical portion.

They are almost always squamous cell cancer, with a rare adenocarcinoma arising from a congenital inlet patch or columnar lining.

These tumors, particularly those in the postcricoid area, represent a separate pathologic entity or two reasons: (a) They are more common in women and appear to be a unique entity in this regard; and (b) the efferent lymphatics from the cervical esophagus drain completely differently from those of the thoracic esophagus.

The latter drain directly into the paratracheal and deep cervical or internal jugular lymph nodes (LNs) with minimal flow in a longitudinal direction.

Except in advanced disease, it is unusual for intrathoracic LNs to be involved.

(See Schwartz 10th ed., p. 1005.)

How well did you know this?
1
Not at all
2
3
4
5
Perfectly
24
Q

The preoperative test most heavily correlated with the ability to tolerate an esophagectomy is:

A. DLCO

B. FEV1

C. Ability to climb a flight of stairs

D. FVC

A

Answer: B

Patients undergoing esophageal resection should have sufficient cardiopulmonary reserve to tolerate the proposed procedure.

The respiratory function is best assessed with the forced expiratory volume in 1 second, which ideally should be 2L or more.

Any patient with a forced expiratory volume in 1 second of <1.25 L is a poor candidate for thoracotomy, because he or she has a 40% risk of dying from respiratory insufficiency within 4 years.

In patients with poor pulmonary reserve, the transhiatal esophagectomy should be considered, as the pulmonary morbidity of this operation is less than is seen following thoracotomy.

Clinical evaluation and electrocardiogram are not sufficient indicators of cardiac reserve.

Echocardiography and dipyridamole-thallium imaging provide accurate in formation on wall motion, ejection fraction, and myocardial blood flow.

A defect on thallium imaging may require further evaluation with preoperative coronary angiography. Arresting ejection fraction of <40%, particularly if there is no increase with exercise, is an ominous sign.

In the absence of invasive testing, observed stair-climbing is an economical (albeit not quantitative) method of assessing cardiopulmonary reserve.

Most individuals who can climb three flights of stairs without stopping will do well with two-sided open esophagectomy, especially if an epidural catheter is used for postoperative pain relief.

(See Schwartz 10th ed., p. 1007.)

How well did you know this?
1
Not at all
2
3
4
5
Perfectly
25
Q

Which test most accurately assess the T stage of esophageal cancer?

A. High-resolution CT scan

B. Magnetic resonance imaging (MRI)

C. Echocardiography

D. Endoscopic ultrasonography (EUS)

E. Esophagogastroduodenoscopy (EGD)

A

Answer: D

For years, clinical staging, contrast radiography, endoscopy, and CT scanning formed the backbone of esophageal cancer staging.

More recently, preoperative decision making is guided by endoscopic ultrasonography (EUS) and positron emission tomography (PET) scanning.

EUS provides the most reliable method of determining depth of cancer invasion.

In the absence of enlarged LNs, the degree of wall invasion dictates surgical therapy.

(See Schwartz 10th ed., p. 1008.)

How well did you know this?
1
Not at all
2
3
4
5
Perfectly
26
Q

Which of the following patients would not be considered a candidate for esophagectomy?

A. A 55-year-old man with GEJ adenocarcinoma confined to the muscularis mucosa.

B. A 47-year-old woman with mid-esophageal cancer and an involved cervical LN.

C. A 60-year-old man with a large GEJ carcinoma with invasion into the pleura without a malignant effusion.

D. A 70-year-old woman with a small GEJ cancer and three pathologic LNs nearby on EUS.

A

Answer: B

If the tumor invades into the submucosa, without visible LN involvement, most individuals would suggest esophagectomy with LN dissection, as positive nodes can be found in 20 to 25% of those with cancer limited to the mucosa and submucosa.

If EUS demonstrates spread through the wall of the esophagus, especially if LNs are enlarged, then induction chemoradiation therapy (neoadjuvant therapy) should be strongly considered.

Lastly, when the EUS demonstrates invasion of the trachea, bronchus, aorta, or spine, then surgical resection is rarely indicated.

If there is invasion into the pleura (T4a), then surgical resection can be considered in the absence of a malignant effusion. Thus, it can be seen that the therapy of esophageal cancer is largely driven by the findings of an endoscopic ultrasonography.

It is difficult to provide modern treatment of esophageal cancer without access to this modality.

(See Schwartz 10th ed., p. 1008.)

How well did you know this?
1
Not at all
2
3
4
5
Perfectly
27
Q

The technique of resecting an esophageal cancer which remains symptomatic after definitive chemoradiotherapy is referred to as

A. Palliative esophagectomy

B. Savage esophagectomy

C. Rescue esophagectomy

D. None of the above, the procedure is not performed

A

Answer: B

Salvage esophagectomy is the nomenclature applied to esophagectomy performed after failure of definitive radiation and chemotherapy.

The most frequent scenario is one in which distant disease (bone, lung, brain, or wide LN metastases) renders the patient nonoperable at initial presentation.

Then, systemic chemotherapy, usually with radiation of the primary tumor, destroys a foci of metastasis, as demonstrated by CT and CT-PET, but the primary remains present and symptomatic.

Following a period of observation, to make sure no new disease with become evident, salvage esophagectomy is performed, usually with an open two-field approach.

Surprisingly, the cure rate of savage esophagectomy is not inconsequential. One in our patients undergoing this operation will be disease free 5 years after, despite the presence of residual cancer in the operative specimen.

Because of the dense scarring created by radiation treatment, this procedure is the most technically challenging of all esophagectomy techniques.

(See Schwartz 10th ed., p. 1011.)

How well did you know this?
1
Not at all
2
3
4
5
Perfectly
28
Q

Patients with dysphagia secondary to esophageal cancer treated with radiation can expect the benefit to last

A. <1month

B. 2–3months

C. 6–12months

D. >12months

A

Answer: B

Primary treatment with radiation therapy does not produce results comparable with those obtained with surgery.

Currently, the use of radiotherapy is restricted to patients who are not candidates for surgery, and is usually combined with chemotherapy.

Radiation alone is used for palliation of dysphagia but the benefit is short-lived, fasting only 2 to 3 months.

Furthermore, the length and course of treatment are difficult to justify in patients with a limited life expectancy.

Radiation is effective in patients who have hemorrhage from the primary tumor.

(See Schwartz 10th ed., p. 1012.)

How well did you know this?
1
Not at all
2
3
4
5
Perfectly
29
Q

How long after completion of neoadjuvant chemoradiotherapy should esophagectomy be performed?

A. 2weeks

B. 4–6weeks

C. 6–8 weeks

D. 8–10weeks

E. >10 weeks

A

Answer: C

The timing of surgery after chemoradiation induction is generally felt to be optimal between 6 and 8 weeks following the completion of induction therapy.

Earlier than this time, active inflammation may make the resection hazardous, and the patients have not had time to recover fully from the chemoradiation.

After 8 weeks, edema in the periesophageal tissue starts to turn to scar tissue, making dissection more difficult.

With chemoradiation, the complete response rates for adenocarcinoma range from 17 to 24%.

No tumor is detected in the specimen after esophagectomy.

Patients demonstrating a complete response to chemoradiation have a better survival rate than those without complete response, but distant failure remains common.

(See Schwartz 10th ed., pp. 1012–1013.)

How well did you know this?
1
Not at all
2
3
4
5
Perfectly
30
Q

The optimal treatment of an incidentally discovered 3 cm leiomyoma of the upper esophagus in a 45-year-old otherwise healthy man is?

A. Observation

B. Esophagectomy

C. Enucleation

D. Endoscopic resection

A

Answer: C

Despite their slow growth and limited potential for malignant degeneration, leiomyomas should be removed unless there are specific contraindications.

The majority can be removed by simple enucleation. If, during removal, the mucosa is inadvertently entered, the defect can be repaired primarily.

After tumor removal, the outer esophageal wall should be reconstructed by closure of the muscle layer.

The location of the lesion and the extent of surgery required will dictate the approach.

Lesions of the proximal and middle esophagus require a right thoracotomy, whereas distal esophageal lesions require a left thoracotomy.

Videothoracoscopic and laparoscopic approaches are now frequently used.

The mortality rate associated with enucleation is low, and success in relieving the dysphagia is near 100%.

Large lesions for those involving the GEJ may require esophageal resection.

(See Schwartz 10thed., pp. 1017–1018.)

How well did you know this?
1
Not at all
2
3
4
5
Perfectly
31
Q

Following a night of heavy drinking, a 43-year-old otherwise healthy man has sudden onset of severe chest pain after vomiting. Esophagram confirms esophageal ruptures just proximal to the GEJ. What is the preferred operative exposure?

A. Right thoracotomy

B. Right thoracotomy with laparotomy

C. Left thoracotomy

D. Left thoracotomy with laparotomy

E. Midline laparotomy

A

Answer: C

The key to optimum management is early diagnosis.

The most favorable outcome is obtained following primary closure of the perforation within 24 hours, resulting in 80 to 90% survival.

The most common location for the injury is the left lateral wall of the esophagus, just above the GEJ.

To get adequate exposure of the injury, a dissection similar to that described for esophageal myotomy is performed.

A flap of stomach is pulled up and the soiled fat pad at the GEJ is removed.

The edges of the injury are trimmed and closed primarily.

The closure is reinforced with the use of a pleural patch or
construction of a Nissen fundopication.

(See Schwartz 10th ed., pp. 1018–1019.)

How well did you know this?
1
Not at all
2
3
4
5
Perfectly
32
Q

A 34-year-old man presents to the emergency department (ED) after an episode of hematemesis. EGD confirms a Mallory-Weiss tear with no residual bleeding.

Treatment should consist of

A. Esophagectomy

B. Observation

C. Proximal gastrectomy with esophagojejunostomy

D. Injection of botulinum toxin

A

Answer: B

Mallory-Weiss tears are characterized by arterial bleeding, which may be massive.

Vomiting is not an obligatory factor, as there may be other causes of an acute increase in intraabdominal pressure, such as paroxysmal coughing, seizures, and retching.

The diagnosis requires a high index of suspicion, particularly in the patient who develops upper gastrointestinal (GI) bleeding following prolonged vomiting or retching.

Upper endoscopy confirms the suspicion by identifying one or more longitudinal fissures in the mucosa of the herniated stomach as the source of bleeding.

In the majority of patients, the bleeding will stop spontaneously with nonoperative management.

In addition to blood replacement, the stomach should be decompressed and antiemetics administered, as a distended stomach and continued vomiting aggravate further bleeding. A Sengstaken-Blakemore tube will not stop the bleeding, as the pressure in the balloon is not sufficient to overcome arterial pressure.

Endoscopic injection of epinephrine may be therapeutic if bleeding does not stop spontaneously.

Only occasionally will surgery be required to stop blood loss.

The procedure consists of laparotomy and high gastrotomy with oversewing of the linear tear.

Mortality is uncommon, and recurrence is rare.

(See Schwartz 10th ed., p. 1020.)

How well did you know this?
1
Not at all
2
3
4
5
Perfectly
33
Q

Successful treatment of a Zenker diverticulum involves

A. Diverticulopexy

B. Resection of the diverticulum

C. Observation

D. Either diverticulopexy for resection with cricopharyngeal myotomy

A

Answer: D

When a pharyngoesophageal diverticulum is present, localization of the pharyngoesophageal segment is easy.

The diverticulum is carefully freed from the overlying areolar tissue to expose its neck, just below the inferior pharyngeal constrictor and above the cricopharyngeus muscle.

It can be difficult to identify the cricopharyngeus muscle in the absence of a diverticulum.

A benefit of local anesthesia is that the patient can swallow and demonstrate an area of persistent narrowing at the pharyngoesophageal junction.

Furthermore, before closing the incision, gelatin can be fed to the patient to ascertain whether the symptoms have been relieved, and to inspect the opening of the previously narrowed pharyngoesophageal segment.

Under general anesthesia, and in the absence of a diverticulum, the placement of a nasogastric tube to the level of the manometrically determined cricopharyngeal sphincter helps in localization of the structures.

The myotomy is extended cephalad by dividing 1 to 2 cm of inferior constrictor muscle of the pharynx, and caudad by dividing the cricopharyngeal muscle and the cervical esophagus for a length of 4 to 5 cm.

If a diverticulum is present and is large enough to persist after a myotomy, it may be sutured in the inverted position to the prevertebra fascia using a permanent suture (ie, diverticulopexy).

If the diverticulum is excessively large so that it would be redundant if suspended, or if its walls are thickened, then a diverticulectomy should be performed.

This is best performed under general anesthesia by placing a Maloney dilator (48F) in the esophagus, after controlling the neck of the diverticulum and after myotomy.

A linear stapler is placed across the neck of the diverticulum and the diverticulum is excised distal to the staple line.

The security of this staple line and effectiveness of the myotomy may be tested before hospital discharge with a water soluble contrast esophagogram.

Postoperative complications include fistula formation, abscess, hematoma, recurrent nerve paralysis, difficulties in phonation, and Horner syndrome.

The incidence of the first two can be reduced by performing a diverticulopexy rather than diverticulectomy.

(See Schwartz 10th ed., p. 989.)

How well did you know this?
1
Not at all
2
3
4
5
Perfectly
34
Q

Which of the following disorders involves simultaneous nonperistaltic contractions of the esophagus?

A. Achalasia

B. Diffuse esophageal spasm (DES)

C. Hypertensive lower esophageal sphincter

D. Nutcracker esophagus

A

Answer: B

The classic manometric findings in these patients are characterized by the frequent occurrence of simultaneous waveforms and multipeaked esophageal contractions, which may be of abnormally high amplitude or long duration.

Key to the diagnosis of diffuse esophageal spasm (DES) is that there remain some peristaltic wave forms in excess of those seen in achalasia.

A criterion of 30% or more peristaltic wave forms out of 10 wet swallows has been used to differentiate DES from vigorous achalasia. However, this figure is arbitrary and often debated.

The LES in patients with DES usually shows a normal resting pressure and relaxation on swallowing.

A hypertensive sphincter with poor relaxation may also be present. In patients with advanced disease, the radiographic appearance of tertiary contractions appears helical, and has been termed corkscrew esophagus or pseudodiverticulosis.

Patients with segmental or DES can compartmentalize the esophagus and develop an epiphrenic or midesophageal diverticulum between two areas of high pressure occurring simultaneously.

MANOMETRIC CHARACTERISTICS

1) ACHALASIA
- Incomplete lower esophageal sphincter (LES) relaxation (<75% relaxation)
- Aperistalsis in the esophageal body
- Elevated LES pressure ≤26 mmHg
- Increased intraesophageal baseline pressures relative to gastric baseline

2) DIFFUSE ESOPHAGEAL SPASM (DES)
- Simultaneous (nonperistaltic contractions) (>20% of wet swallows)
- Repetitive and multipeaked contractions
- Spontaneous contractions
- Intermittent normal peristalsis
- Contractions may be of increased amplitude and duration

3) NUTCRACKER ESOPHAGUS
- Mean peristaltic amplitude (10 wet swallows) in distal esophagus
≥180mmHg
- Increased mean duration of contractions (>7.0 s)
- Normal peristaltic sequence

4) HYPERTENSIVE LOWER ESOPHAGEAL SPHINCTER
- Elevated LES pressure (≥26 mmHg)
- Normal LES relaxation
- Normal peristalsis in the esophageal body

5) INEFFECTIVE ESOPHAGEAL MOTILITY DISORDERS
- Decreased or absent amplitude of esophageal peristalsis (<30 mmHg)
- Increased number of nontransmitted contractions

How well did you know this?
1
Not at all
2
3
4
5
Perfectly
35
Q

What layer is absent in the esophagus?

A

The esophagus lacks a serosa.

How well did you know this?
1
Not at all
2
3
4
5
Perfectly
36
Q

What are the 2 nerve plexuses in the esophagus?

A

1) Meissner’s
- Submucosa

2) Auerbach’s
- Between muscle layers
- Responsible for peristalsis

How well did you know this?
1
Not at all
2
3
4
5
Perfectly
37
Q

The 3 anatomic constrictions of the esophagus?

A

1) Cricopharyngeal muscle
- Uppermost narrowing
- 1.5cm (narrowest)
- Main contributor to the Upper Esophageal Sphincter.

2) Crossing of left mainstem bronchus and aortic arch
- 1.6cm

3) Diaphragmatic hiatus
- 1.6 to 1.9cm

These normal constrictions tend to hold up swallowed foreign objects, and the overlying mucosa is subject to injury by the slow passage of corrosive liquids.

How well did you know this?
1
Not at all
2
3
4
5
Perfectly
38
Q

Pressure in the UES?

A

The UES is a high-pressure zone that can reach 90mmHg, and return to average resting pressure of 60mmHg when swallowing.

How well did you know this?
1
Not at all
2
3
4
5
Perfectly
39
Q

Pressure in the LES?

A

The LES has a resting pressure zone of 6 to 26mmHg.

It is 2-5cm long, and vagus-mediated relaxation occurs during normal food transit.

Pressure is increased by GASTRIN and MOTILIN.

Pressure is decreased by CCK and SECRETIN.

How well did you know this?
1
Not at all
2
3
4
5
Perfectly
40
Q

Blood supply of the esophagus?

A

1) Cervical Esophagus
- Inferior thyroid artery

2) Proximal Thoracic Esophagus
- Bronchial arteries

3) Distal Thoracic Esophagus
- Aortic branches

4) Intraabdominal Esophagus
- Ascending branch of the left gastric artery
- Inferior phrenic arteries

How well did you know this?
1
Not at all
2
3
4
5
Perfectly
41
Q

Discuss esophageal contractions.

A

Esophageal contractions are coordinated in the medulla.

1) Oral
- Voluntary
- Medial temporal lobes and limbic system

2) Pharyngeal
- Involuntary (once initiated, swallowing is an entirely reflex act.)
- Larynx closes.

3) Esophageal
- Initiated by pharyngeal activity in swallowing.
- Primary peristalsis: Becomes involuntary after initiation
- Secondary: Triggered by distension/irritation; reflux contents back.
- Tertiary: Nonprogressive, nonperistaltic, spasm

How well did you know this?
1
Not at all
2
3
4
5
Perfectly
42
Q

The parasympathetic innervation of the pharynx and esophagus is provided by?

A

Vagus nerves

How well did you know this?
1
Not at all
2
3
4
5
Perfectly
43
Q

Why do lesions in the cervical esophagus have less submucosal extension, and a more regionalized lymphatic spread?

A

The cervical esophagus has more direct segmental lymph drainage into the regional nodes.

In the upper 2/3 of the esophagus, the lymph flow is mostly cephalad.

In the lower 2/3, caudad.

In the thoracic portion, the submucosal lymph plexus extends over a long distance in a longitudinal direction before penetrating the muscle layer to enter lymph vessels in the adventitia. Consequently, a primary tumor can extend for a considerable length superiorly or inferiorly in the submucosal plexus. Free tumor cells can follow the submucosal lymphatic plexus in either direction for a long distance before they pass through the muscularis and on into the regional LNs.

How well did you know this?
1
Not at all
2
3
4
5
Perfectly
44
Q

Drainage of efferent lymphatics from the esophagus?

A

Cervical: Drain into the paratracheal and deep cervical LNs.

Upper thoracic: Empty mainly into the paratracheal LNs.

From the lower esophagus: Into the subcarinal nodes and inferior pulmonary ligament nodes

Superior gastric nodes receive lymph not only from the abdominal portion of the esophagus, but also from the adjacent lower thoracic segment.

How well did you know this?
1
Not at all
2
3
4
5
Perfectly
45
Q

Sequence of events during the oropharyngeal phase of swallowing?

A
  1. Elevation of tongue
  2. Posterior movement of tongue
  3. Elevation of soft palate (closing passage between oropharynx and nasopharynx)
  4. Elevation of hyoid
  5. Elevation of larynx
  6. Tilting of epiglottis
How well did you know this?
1
Not at all
2
3
4
5
Perfectly
46
Q

What are the afferent sensory nerves of the pharynx?

A

1) Glossopharyngeal nerve (CN IX)
2) Superior laryngeal branch of the vagus nerves

Once aroused by stimuli entering via these nerves, the swallowing center in the medulla coordinates the complete act of swallowing by discharging impulses through CN V, VII, X, XI and XII, as well as motor neurons of C1 to C3.

How well did you know this?
1
Not at all
2
3
4
5
Perfectly
47
Q

What is the maximum weight that the propulsive force of the esophagus can overcome?

A

5 to 10g

How well did you know this?
1
Not at all
2
3
4
5
Perfectly
48
Q

What are the 3 components of the antireflux mechanism in human beings?

A

1) A mechanically effective LES
2) Efficient esophageal clearance
3) An adequately functioning gastric reservoir

How well did you know this?
1
Not at all
2
3
4
5
Perfectly
49
Q

In what position is physiologic reflux more common?

A

When awake and in the upright position.

Reflux occurs in healthy volunteers primarily during transient losses of the gastroesophageal barrier, which may be due to a relaxation of the LES or intragastric pressure overcoming sphincter pressure.

Gastric juice can also reflux when a swallow-induced relaxation of the LES is not protected by an oncoming peristaltic wave.

The average frequency of these unguarded moments or of transient losses of the gastroesophageal barrier is far less while asleep and in the supine position than while awake and in the upright position.

Hence there are fewer opportunities for reflux in the supine position.

Second, in the upright position, there is a 12mmHg pressure gradient between the resting, positive intraabdominal pressure measured in the stomach and the most negative intrathoracic pressure measured in the esophagus at midthoracic level. This gradient favors the flow of gastric juice up into the thoracic esophagus when upright. The gradient diminishes in the supine position.

Third, LES pressure in normal subjects in significantly higher in the supine position than in the upright position. This is due to the apposition of the hydrostatic pressure of the abdomen to the abdominal portion of the sphincter when supine.

How well did you know this?
1
Not at all
2
3
4
5
Perfectly
50
Q

Neural and hormonal mechanisms affecting the LES?

A

INCREASE LES Pressure:

  • Alpha-adrenergic neurotransmitters or beta-blockers
  • Gastrin and motilin
  • Bombesin, l-enkephalin, substance P
  • Antacids, cholinergics, agonists, domperidone, metoclopramide, prostaglandin F2

DECREASE LES Pressure:

  • Alpha-blockers and beta-stimulants
  • CCK, estrogen, glucagon, progesterone, somatostatin, secretin
  • Calcitonin gene-related peptide, gastric inhibitory peptide, neuropeptide Y, vasoactive intestinal polypeptide
  • Anticholinergics, barbiturates, calcium channel blockers, caffeine, diazepam, dopamine, meperidine, prostaglandin E1 and E2, theophylline
  • Peppermint, chocolate, coffee, ethanol, fat
How well did you know this?
1
Not at all
2
3
4
5
Perfectly
51
Q

Tests to detect structural abnormalities?

A

1) Endoscopic evaluation
- First diagnostic test (UGI Endoscopy) in any patient complaining of DYSPHAGIA.
- For initial assessment, flexible fiberoptic esophagoscope is the instrument of choice.

  • GERD: Esophagitis, Barrett’s columnar-lined esophagus (CLE– columnar instead of normal squamous epithelium; histologically, intestinal metaplasia).
  • Abnormalities of the gastroesophageal flap (via retroflexion): Grading of I-IV
  • Hiatal hernia: Pouch lined with gastric rugal folds lying 2cm or more above the margins of the diaphragmatic crura, identified by having the patient sniff. A prominent sliding hiatal hernia is associated with increased exposure to gastric juice.
  • Paraesophageal hernia: Exclude gastric (Cameron’s) ulcers or gastritis within the pouch. (Intragastric retroflex or J maneuver)
  • Esophageal diverticulum: Exclude ulceration or neoplasia. Biopsy is NOT usually performed for submucosal lesions.

2) Radiographic evaluation
- Barium swallow: Assess anatomy and motility

  • Hiatal hernias: Best observed prone because increased intraabdominal pressure promotes displacement of the esophagogastric junction above the diaphragm.
  • Lower esophageal narrowing (rings, strictures): Fully distended views of the esophagogastric region
  • Circumferential carcinomas, peptic strictures, large ulcers, hiatal hernias: Full-column technique
  • Small esophageal neoplasms, mild esophagitis and esophageal varices may be missed, hence the full-column technique must be supplemented by mucosal relief or double-contrast films
  • Motion recording techniques aid in evaluating functional disorders or pharyngoesophageal and esophageal phases of swallowing.
  • The radiographic assessment of the esophagus is not complete unless the entire stomach and duodenum have been examined. A gastric/duodenal ulcer, partially obstructing gastric neoplasm, or scarred duodenum and pylorus may contribute to symptoms.
How well did you know this?
1
Not at all
2
3
4
5
Perfectly
52
Q

Grading for esophagitis?

A

LA Grade A
- Mucosal break(s) ≤5 mm, without continuity across mucosal folds

LA Grade B
- Mucosal break(s) >5 mm, without continuity across mucosal folds

LA Grade C
- Mucosal break(s) continuous between ≥2 mucosal folds, involving <75% of the esophageal circumference

LA Grade D
- Mucosal break(s) involving ≥75% of the esophageal circumference

When a stricture is observed, the absence of esophagitis above a stricture suggests the possibility of a chemical-induced injury or a neoplasm. The latter should always be considered and is ruled out only by evaluation of a tissue biopsy of adequate size.

How well did you know this?
1
Not at all
2
3
4
5
Perfectly
53
Q

Principles in taking a biopsy from Barrett’s epithelium?

A

Multiple biopsy specimens should be taken in a cephalad direction to confirm the presence of intestinal metaplasia.

The earliest sign of malignant degeneration is high grade dysplasia or intramucosal adenocarcinoma. These dysplastic changes have a patchy distribution, so a minimum of 4 biopsy samples spaced 2cm apart should be taken from Barret’s lined portion of the esophagus.

Tumors occur in an area of specialized columnar epithelium near the squamocolumnar junction in 85% of patients, and within 2cm of the squamocolumnar junction in virtually all patients.

How well did you know this?
1
Not at all
2
3
4
5
Perfectly
54
Q

What can be employed when a patient complains of dysphagia, and no obstructing lesion is seen on the barium swallow?

A

Have the patient swallow a barium-impregnated marshmallow, a barium-soaked piece of bread, or a hamburger mixed with barium. This may bring out a functional disturbance in esophageal transport that can be missed when liquid barium is used.

How well did you know this?
1
Not at all
2
3
4
5
Perfectly
55
Q

Tests to detect functional abnormalities?

A

1) Esophageal motility study (EMS)
- Motor function and sphincters are evaluated
- Dysphagia, odynophagia, noncardiac chest pain, and when the barium swallow/endoscopy does not show a clear structural abnormality.
- Confirms dx of primary esophageal motility disorders (achalasia, DES, nutcracker esophagus, hypertensive LES).
- Confirms dx of motility disorders secondary to systemic disease (scleroderma, dermatomyositis, polymyositis, mixed connective tissue disease).
- GERD: Mechanically defective LES, evaluate esophageal peristalsis and contraction amplitude.
- A rise in pressure above the gastric baseline is the beginning of the LES.

2) High-Resolution Manometry
- Multiple circumferential recording sites are used to create a “map” of the esophagus and its sphincters.
- LES is a high-pressure zone that should relax at the inception of swallowing, and contract after the water or solid bolus passes through the LES.
- May detect overlooked focal motor abnormalities, predict bolus propagation and measure pressure gradients.

3) Esophageal Impedance
- Intraluminal electrical impedance catheter is used to measure GI function.
- Impedance: ratio of voltage to current, and is a measure of the electrical conductivity of a hollow organ and its contents.
- Intraluminal electrical impedance is inversely proportional to the electrical conductivity of the luminal contents and the cross-sectional area of the lumen.
- Air has very low electrical conductivity, and therefore has high impedance.
- Selectively used for dx of GERD. (24h pH monitoring has limitations– insensitive to higher pH values, can only detect abnormal acid reflux at pH <4, which is altered in those taking PPIs.

4) Esophageal Transit Scintigraphy
- Esophageal transit of a 10mL water bolus containing technetium-99m sulfur colloid can be recorded with a gamma camera. Using this technique, delayed bolus transit has been shown in patients with a variety of esophageal motor disorders, including achalasia, scleroderma, DES, and nutcracker esophagus.

5) Video and Cineradiography
- Allows re-evaluation by reviewing the studies at various speeds.
- More useful than manometry in evaluating the pharyngeal phase of swallowing.
- Manofluorography: Simultaneous computerized capture of videofluoroscopic images and manometric tracings (for complex functional abnormalities).

How well did you know this?
1
Not at all
2
3
4
5
Perfectly
56
Q

What is the respiratory inversion point?

A

The respiratory inversion point is identified when the positive excursions in the abdominal cavity that occur with breathing change to negative deflections in the thorax. This serves as a reference point at which the amplitude of the LES pressure and the length of the sphincter exposed to abdominal pressure are measured.

As the pressure-sensitive station is withdrawn into the body of the esophagus, the upper border of the LES is identified by the drop in pressure to the esophageal baseline. From these measurements, the pressure, abdominal length, and overall length of the sphincter are determined.

To account for the asymmetry of the sphincter, the pressure profile is repeated with each of the five radially oriented transducers, and the average values for sphincter pressure above gastric baseline, overall sphincter length, and abdominal length of the sphincter are calculated.

How well did you know this?
1
Not at all
2
3
4
5
Perfectly
57
Q

How is a mechanically defective sphincter identified?

A

A mechanically defective sphincter is identified by having one or more of the following:

1) Average LES pressure of <6mmHg
2) Average length exposed to the positive-pressure environment in the abdomen of ≤1cm
3) Average overall sphincter length of ≤2cm

How well did you know this?
1
Not at all
2
3
4
5
Perfectly
58
Q

Tests to detect increased exposure to gastric juice?

A

1) 24h Ambulatory pH Monitoring
- 48H* esophageal pH monitoring is the Gold standard for the diagnosis of GERD.
- the most direct method of measuring increased esophageal exposure, via an indwelling pH electrode or a radiotelemetric pH monitoring capsule
- measures the actual time the esophageal mucosa is exposed to gastric juice
- measures the ability of the esophagus to clear refluxed acid
- correlates esophageal acid exposure with the patient’s symptoms
- measurements made over 1-2 circadian cycles (24-48h)
- should NOT be considered a test for reflux, but a measurement of esophageal exposure to gastric juice,

2) Radiographic Detection of GERD
- 40% patients with classic symptoms: Spontaneous reflux
- In most patients who show spontaneous reflux on radiography, the diagnosis of increased esophageal acid exposure is confirmed by 24h esophageal pH monitoring.
- Hence the radiographic demonstration of spontaneous regurgitation of barium into the esophagus in the upright position is a reliable indicator that reflux is present.
- Failure to see this does not indicate the absence of the disease, hence this test is rarely used for clinical diagnosis.

How well did you know this?
1
Not at all
2
3
4
5
Perfectly
59
Q

Units used to express esophageal exposure to gastric juice?

A

1) Cumulative time the esophageal pH is below a chosen threshold, expressed as percentage of total, upright, and supine monitored time
2) Frequency of reflux episodes below a chosen threshold, expressed as number of episodes per 24h
3) Duration of the episodes, expressed as number of episodes >5 minutes per 24h, and the time in minutes of the longest episode recorded.

How well did you know this?
1
Not at all
2
3
4
5
Perfectly
60
Q

Tests of duodenogastric function?

A

Esophageal disorders are frequently associated with abnormalities of duodenogastric function: reflux of alkaline duodenal juice, including bile salts, pancreatic enzymes, and bicarbonate, is thought to have a role in the pathogenesis of esophagitis and complicated Barrett’s esophagus.

Functional disorders are also not confined to the esophagus but are associated with those of the rest of the foregut.

1) Gastric Emptying Study
- Radionuclide-labeled meals
- After ingestion, gamma camera images of the stomach are obtained at 5 to 15 minute intervals for 2-4hours.
- After correction for decay, the counts in the gastric area are plotted as the percentage of total counts at the start of imaging.
- The resulting emptying curve can be compared with data obtained in normal volunteers.
- Normal subjects will empty 59% of a meal within 90minutes.

How well did you know this?
1
Not at all
2
3
4
5
Perfectly
61
Q

What is the most specific symptom of foregut disease?

A

Dysphagia

It can be a sign of underlying malignancy and should be aggressively investigated until a diagnosis is established.

Oropharyngeal
- Difficulty transferring food out of the mouth into the esophagus, nasal regurgitation, and/or aspiration.

Esophageal

  • Sensation of food sticking in the lower chest or epigastrium
  • May or may not be accompanied by pain (odynophagia) relieved by passage of the bolus
How well did you know this?
1
Not at all
2
3
4
5
Perfectly
62
Q

What is the common denominator for most episodes of gastroesophageal reflux?

A

The loss of the high pressure zone and a decrease in the resistance it imparts to the retrograde flow of gastric juice into the esophageal body.

How well did you know this?
1
Not at all
2
3
4
5
Perfectly
63
Q

What characteristics of the LES work in unison to maintain its barrier function?

A

1) Resting LES pressure
2) Overall length
3) Intraabdominal length

As the sphincter becomes shorter, a higher amount of pressure is needed to prevent a given amount of reflux. Much like the neck of a balloon as it is inflated, as the stomach fills and distends, sphincter length decreases.

Therefore, if the overall length of the sphincter is permanently short from repeated distention of the fundus secondary to large volume meals, then with minimal episodes of gastric distention and pressure, there will be insufficient sphincter length for the barrier to remain competent, and reflux will occur.

How well did you know this?
1
Not at all
2
3
4
5
Perfectly
64
Q

A permanently defective sphincter is defined by one or more of the following:

A

1) An LES with a mean resting pressure of <6mmHg
2) An overall sphincter length of <2cm;
3) Intraabdominal sphincter length of <1cm

How well did you know this?
1
Not at all
2
3
4
5
Perfectly
65
Q

What is the pathophysiology of GERD?

A

It is believed that GERD has its origins within the stomach. Distension of the fundus occurs because of overeating and delayed gastric emptying secondary to a high-fat diet. The resultant distension causes “unrolling” of the sphincter by the expanding fundus, and this exposes the squamous epithelium in the region of the distal LES to gastric juice.

Repeated exposure results in inflammation and development of columnar epithelium at the cardia. This is the initial step in the development of carditis and explains why in early disease, esophagitis is mild and commonly limited to the very distal aspect of the esophagus. The patient attempts to compensate by increased swallowing, allowing the saliva to neutralize the refluxed gastric juice and thus, alleviate discomfort.

Increased swallowing leads to aerophagia, bloating and belching, and creates a vicious cycle of repetitive injury. Additionally, this can lead to a fibrotic mucosal ring at the squamocolumnar junction, called a “Schatzki ring” which may result in dysphagia.

This explanation for the pathophysiology of GERD is supported by the observation that severe esophagitis is almost always associated with a defective LES.

How well did you know this?
1
Not at all
2
3
4
5
Perfectly
66
Q

Complications of gastroesophageal reflux such as esophagitis, stricture, and Barrett’s metaplasia occur in the presence of which predisposing factors?

A

1) A mechanically defective LES

2) Increased esophageal exposure to fluid containing duodenal content, that includes bile and pancreatic juice

How well did you know this?
1
Not at all
2
3
4
5
Perfectly
67
Q

What is the incidence of metaplastic Barrett’s epithelium becoming dysplastic and progressing to adenocarcinoma?

A

0.2 - 0.5% per year

How well did you know this?
1
Not at all
2
3
4
5
Perfectly
68
Q

What is the hallmark of intestinal metaplasia?

A

The presence of intestinal goblet cells

How well did you know this?
1
Not at all
2
3
4
5
Perfectly
69
Q

What is the primary reason for performing antireflux surgery in patients with Barrett’s esophagus?

A

Long-term relief of symptoms

Secondary goals: healing of esophageal mucosal injury and prevention of disease progression

How well did you know this?
1
Not at all
2
3
4
5
Perfectly
70
Q

What is the incidence of adenocarcinoma developing in Barrett’s mucosa?

A

It occurs at approximately 0.2 - 0.5% per year of follow-up, which represents a risk 40x that of the general population.

Most, if not all cases of adenocarcinoma of the esophagus arise in Barrett’s epithelium.

About 1/3 of all patients with BE present with malignancy.

How well did you know this?
1
Not at all
2
3
4
5
Perfectly
71
Q

What are some respiratory symptoms that develop in patients with GERD?

A

1) Laryngopharyngeal reflux-type symptoms
2) Adult-onset asthma
3) Idiopathic pulmonary fibrosis

How well did you know this?
1
Not at all
2
3
4
5
Perfectly
72
Q

What are the mechanisms behind reflux-induced respiratory symptoms?

A

1) Reflux theory
- Symptoms are the direct result of laryngopharyngeal exposure and aspiration of gastric contents
- Vagal-mediated afferent fibers result in bronchoconstriction during episodes of distal esophageal acidification.

2) Ambulatory pH testing
- Acid exposure within the proximal esophagus is more frequently identified in those with gastroesophageal reflux and respiratory symptoms, than those with gastroesophageal reflux symptoms alone.

3) Impedance testing
- A correlation between proximal fluid movement and laryngopharyngeal symptoms such as cough can be demonstrated.

4) Bronchoconstriction occurs with the infusion of acid into the distal esophagus. There is a shared embryologic origin of the tracheoesophageal tract and vagus nerve, and this reflex is though to be an afferent fiber-mediated reflex that protects the aerodigestive system from the aspiration of refluxate.

How well did you know this?
1
Not at all
2
3
4
5
Perfectly
73
Q

Medical therapy for GERD?

A

First-line:
When initially identified with mild symptoms of uncomplicated GERD, patients can be placed on 12 weeks of simple antacids before diagnostic testing is initiated.

Patients should be counseled to elevated the head of the bed, avoid tight-fitting clothing, eat small frequent meals, avoid eating the nighttime meal immediately prior to bedtime, and avoid alcohol, coffee, chocolate, and peppermint, which are known to reduce resting LES pressure and may aggravate symptoms.

Other supplements:
1) Alginic acid: may augment the relief of symptoms by creating a physical barrier to reflux, as well as by acid reduction; reacts with sodium bicarbonate in the presence of saliva to form a highly viscous solution that floats like a raft on the surface of gastric contents. When reflux occurs, this protective layer is refluxed into the esophagus, and acts as a protective barrier.

2) Metoclopramide or Domperidone: Meds to promote gastric emptying are beneficial in early disease but of little value in more severe disease.

Persistent symptoms:
Mainstay is acid suppression. High-dose regimens of hydrogen potassium PPIs, such as omeprazole (up to 40mg/d), can reduce gastric acidity by as much as 80-90%. This usually heals mild esophagitis.

In severe esophagitis, healing may occur in only one-half of the patients. In patients who reflux a combination of gastric and duodenal juice, acid-suppression therapy may give relief of symptoms, while still allowing mixed reflux to occur. This can allow persistent mucosal damage in an asymptomatic patient.

Within 6 months of discontinuation of any form of medical therapy for GERD, 80% of patients have a recurrence of symptoms, and 40% of individuals with daily GERD eventually develop symptoms that breakthrough adequately dosed PPIs. Once initiated, most patients will require lifelong treatment.

Suggested approach:
First-line therapy entails antisecretory medication, usually PPIs, in most patients. Failure of medication to adequately control GERD symptoms suggests either that the patient may have relatively severe disease, or a non-GERD cause for symptoms.

Endoscopic evaluation is recommended to assess degree of injury and presence of BE.

Treatment options entails either long term PPI use vs. antireflux surgery. Laparoscopic antireflux surgery in these patients achieves long-term control of symptoms in 85-90%. The measurement of esophageal acid exposure via 24h pH should be done when patients are considered for surgery. The status of the LES and esophageal body function should also be performed.

How well did you know this?
1
Not at all
2
3
4
5
Perfectly
74
Q

Key indications for antireflux surgery?

A

1) Objectively proven gastroesophageal reflux disease
2) Typical symptoms of gastroesophageal reflux disease (heartburn/ regurgitation) despite adequate medical management
3) Younger patient unwilling to take lifelong medication (ie, <49yo)
4) Severe endoscopic esophagitis in a symptomatic patients with a structurally defective LES
5) Development of a stricture in a patient– represents failure of medical therapy (malignancy and drug-related etiology must be ruled out, and the stricture should be progressively dilated up to a 50-60F bougie prior to surgery. If dysphagia is relieved and the amplitude of esophageal contractions is adequate, an antireflux procedure should be performed; if there is a global loss of esophageal contractility, caution should be exercised in performing an antireflux procedure, and a partial fundoplication should be considered).

A structurally defective LES can also predict which patients are more likely to fail with medical therapy. Patients with normal sphincter pressures tend to remain well-controlled with medical therapy, whereas patients with a structurally defective LES may not respond as well and often develop recurrent symptoms within 1-2 years.

How well did you know this?
1
Not at all
2
3
4
5
Perfectly
75
Q

What is the treatment approach if high grade dysplasia or intramucosal carcinoma is found on mucosal biopsy specimens?

A

Treatment should be directed at the BE and the lesion, using either evaluation endoscopic ablation, endoscopic resection, or esophageal resection.

How well did you know this?
1
Not at all
2
3
4
5
Perfectly
76
Q

Preop evaluation for antireflux surgery?

A

1) Reflux objectively confirmed by 24h pH monitoring or by the presence of ulcerative esophagitis

2) Propulsive force of body of esophagus evaluated by esophageal manometry (if it has sufficient power to propel a bolus of food through a newly-constructed valve)
- Normal peristaltic contractions: consider for a 360deg Nissen fundoplication or a partial fundoplication
- Absent peristalsis: Partial fundoplication, if achalasia has been ruled out

3) Hiatal anatomy should be assessed
- Smaller hiatal hernias: Endoscopy is sufficient
- Larger hiatal hernias: Contrast radiology

How well did you know this?
1
Not at all
2
3
4
5
Perfectly
77
Q

Primary goal of antireflux surgery?

A

To safely create a new antireflux valve at the gastroesophageal junction, while preserving the patient’s ability to swallow normally and belch to relieve gaseous distention.

The aim of any fundoplication is to create a loose wrap and to maintain the position of the gastric fundus close to the distal intraabdominal esophagus, in a flap valve arrangement.

The efficacy of this relies on the close relationship between the fundus and esophagus, not the tightness of the wrap.

How well did you know this?
1
Not at all
2
3
4
5
Perfectly
78
Q

Basic principles in constructing the antireflux mechanism?

A

1) Operation should create a flap valve which prevents regurgitation of gastric contents into the esophagus. This will result in an increase in the pressure of the distal esophageal sphincter region.
2) Operation should place an adequate length of the distal esophageal sphincter in the positive-pressure environment of the abdomen by a method that ensures its response to changes in intraabdominal pressure. The permanent restoration of 2 or more cm of abdominal esophagus ensures the preservation of the relationship between the fundus and the esophagus. All of the popular antireflux procedures increase the length of the sphincter exposed to abdominal pressure by an average of at least 1cm.
3) Operation should allow the reconstructed cardia to relax on deglutition. In normal swallowing, a vagally mediated relaxation of the distal esophageal sphincter and the gastric fundus occurs. The relaxation lasts for approximately 10 secs and is followed by a rapid recovery to the former tonicity.
4) The fundoplication should not increase the resistance of the relaxed sphincter to a level that exceeds the peristaltic power of the body of the esophagus. The resistance depends on the degree, length, and diameter of the gastric fundic wrap, and on the variation in intraabdominal pressure. A 360deg wrap should be no longer than 2cm and constructed over a large (50-60F) bougie. This will ensure that the relaxed sphincter will have adequate diameter with minimal resistance. A bougie is not necessary when constructing a partial wrap.
5) The operation should ensure that the fundoplication can be placed in the abdomen without undue tension and maintained there by approximating the crura of the diaphragm above the repair. Leaving the fundoplication in the thorax converts a sliding hernia into a PEH, with all the complications associated with that condition. Maintaining the repair in the abdomen under tension predisposes to an increased incidence of recurrence.

How well did you know this?
1
Not at all
2
3
4
5
Perfectly
79
Q

What factors are important to ensure relaxation of the sphincter in antireflux surgery?

A

1) Only the fundus of the stomach should be used to buttress the sphincter, because it is known to relax in concert with the sphincter.
2) The gastric wrap should be properly placed around the sphincter and not incorporate a portion of the stomach or be placed around the stomach itself, because the body of the stomach does not relax with swallowing.
3) Damage to the vagal nerves during dissection of the thoracic esophagus should be avoided because it may result in failure of the sphincter to relax.

How well did you know this?
1
Not at all
2
3
4
5
Perfectly
80
Q

Discuss Nissen fundoplication.

A

A 360degree fundoplication around the lower esophagus for a distance of 4-5cm, without division of the short gastric vessels (now modified– with division of vessels).

Elements common to both lap and open procedures:
1) Hiatal dissection and preservation of both vagi along their entire length

2) Circumferential esophageal mobilization
3) Hiatal closure, usually posterior to the esophagus
4) Creation of a short and floppy fundoplication over an esophageal dilator

How well did you know this?
1
Not at all
2
3
4
5
Perfectly
81
Q

Discuss Posterior Partial fundoplication.

A

Partial fundoplications were developed as an alternative to the Nissen procedure in an attempt to minimize the risk of postfundoplication side effects, such as dysphagia, inability to belch, and flatulence. The commonest approach has been a posterior partial or TOUPET fundoplication.

270-degree gastric fundoplication around the distal 4cm of esophagus.

How well did you know this?
1
Not at all
2
3
4
5
Perfectly
82
Q

Discuss the Anterior Partial fundoplication.

A

Following posterior hiatal repair, the anterior fundus is rolled over the front of the esophagus and sutured to the hiatal rim and the esophageal wall. Division of the short gastric vessels is NOT needed.

180-degree anterior partial fundoplication provides a more robust wrap and achieves an excellent longer-term outcome in 90% of patients at follow up of at least 10 years.

The fundus and esophagus are sutured to the right side of the hiatal rim to create a flap valve at the gastroesophageal junction and to stabilize a 3-4cm long intraabdominal esophagus.

How well did you know this?
1
Not at all
2
3
4
5
Perfectly
83
Q

Summary of antireflux surgical approaches?

A

1) Transoral incisionless fundoplication (TIF)
- Totally endoscopic procedure for performing a partial fundoplication using T-bar fasteners
- 270 to 290deg wrap
- Approved for hernias >2cm.
- ARS remains an option in case of TIF failure.
- Does NOT provide cruroplasty.

2) Belsey-Mark IV
- Recreates the LES while reducing an intrathoracic stomach and closing a hiatal defect.
- Approached via a left thoracotomy hence may avoid a hostile abdomen.
- Useful in the dissection of large and giant chronic paraesophageal hernias.
- 240-deg partial wrap; imbricating cardia against the distal esophagus.

3) Toupet
- 270 degree, posterior partial wrap.

4) Dor
- 180 degree, anterior partial warp.

5) Hill
- Creation of a gastroesophageal flap valve, in an attempt to recreate normal anatomy.

6) Gastropexy
- For highest risk patients with hernias at risk for volvulus.
- May include a percutaneous endoscopic gastrostomy tube.

How well did you know this?
1
Not at all
2
3
4
5
Perfectly
84
Q

Management of Barrett’s Esophagus following antireflux surgery?

A

1) Low-grade dysplasia
- Biopsy specimens should be repeated after 12 weeks of high-dose acid suppression therapy.

2) High-grade dysplasia or Intramucosal cancer in >1 biopsy specimen
- If invasive CA (stage T1b or deeper) is present, or for multifocal long segment BE in younger and fit patients, in whom endoscopic treatments are unlikely to be adequate: ESOPHAGEAL RESECTION.
- For smaller intramucosal tumors to be removed with clear pathology margins, to obtain better pathological staging as a “big biopsy”, or to excise shorter segments of BE piecemeal: ENDOSCOPIC MUCOSAL RESECTION.
- To reduce the rate of progression from high grade dysplasia to invasive cancer by 50%: RF ABLATION.

3) Indeterminate/lower grade
- Suppress dysplasia by a course of acid suppression therapy in high doses for 2-3 months, followed by rebiopsy of the Barrett’s segment.

How well did you know this?
1
Not at all
2
3
4
5
Perfectly
85
Q

Most frequent cause of failure after open antireflux procedures?

A

Placement of the wrap around the stomach

How well did you know this?
1
Not at all
2
3
4
5
Perfectly
86
Q

Most frequent cause of failure after laparoscopic antireflux procedures?

A

Herniation of repair into the chest

How well did you know this?
1
Not at all
2
3
4
5
Perfectly
87
Q

Indications for esophageal resection in patients with failed antireflux repair?

A

When dysphagia is associated with poor motility and multiple previous repairs, further revision fundoplication is unlikely to be successful, and in otherwise fit patients it is appropriate to seriously consider esophageal resection.

How well did you know this?
1
Not at all
2
3
4
5
Perfectly
88
Q

Types of esophageal hiatal hernia?

A

Type I: Sliding hernia

  • Upward dislocation of the cardia in the posterior mediastinum
  • 48 years old

Type II: Rolling or Paraesophageal Hernia (PEH/Giant hiatal hernia)

  • Upward dislocation of gastric fundus alongside a normally positioned cardia
  • 61 years old
  • women > men (4:1)

Type III: Combined sliding-rolling/mixed hernia

  • Upward dislocation of both cardia and gastric fundus
  • The end stage of type I and type II hernias occurs when the whole stomach migrates up into the chest by rotating 180deg around its longitudinal axis, with the cardia and pylorus as fixed points (intrathoracic stomach).

Type IV: When an additional organ, usually the colon, herniates as well.

Types II-IV are also known as paraesophageal hernias, as a portion of the stomach is situated adjacent to the esophagus, above the gastroesophageal junction.

How well did you know this?
1
Not at all
2
3
4
5
Perfectly
89
Q

What is the cause of dyspnea in the presence of a giant PEH?

A

Traditionally, it is believed that dysphagia and postprandial fullness are caused by compression of the adjacent esophagus by a distended cardia, or twisting of the GEJ by the torsion of the stomach that occurs as it becomes progressively displaced in the chest. Respiratory complications ensure from aspiration.

New research demonstrates that the cause of dyspnea in the presence of a giant PEH is more likely to be left atrial compression, decreasing cardiac output, than a restrictive pulmonary effect.

How well did you know this?
1
Not at all
2
3
4
5
Perfectly
90
Q

How would a patient with dysphagia secondary to a sliding hiatal hernia manifest manometrically?

A

This is reflected by a double-humped high-pressure zone at the GEJ.

The first pressure rise is due to diaphragmatic impingement on the herniated stomach.

The second is due to the true distal esophageal sphincter.

How well did you know this?
1
Not at all
2
3
4
5
Perfectly
91
Q

How to diagnose a hiatal hernia?

A

1) Upright CXR
- Air fluid level behind the cardiac shadow
- Caused by a PEH or intrathoracic stomach

2) Upper GI barium study
- Greater accuracy for a paraesophageal hernia (sliding hernias can spontaneously reduce)

3) Fiber-optic esophagoscopy
- Sliding: Gastric pouch lined with rugal folds extending above the impression caused by the crura of the diaphragm (or measuring at least 2cm between the crura), identified by having the patient sniff, and the squamocolumnar junction on withdrawal of the scope.
- PEH: On retroversion of the scope, there is a separate orifice adjacent to the GEJ into which gastric rugal folds ascend.
- Sliding-Rolling/Mixed: Gastric pouch lined with rugal folds above the diaphgram, with the GEJ entering about midway up the side of the pouch.

How well did you know this?
1
Not at all
2
3
4
5
Perfectly
92
Q

Pathophysiology of hiatal hernias?

A

24h esophageal pH monitoring shows that
- PEH: 60% with increased esophageal exposure to gastric acid
- Sliding: 71% with increased esophageal exposure to gastric acid
Hence it is now recognized that paraesophageal hiatal hernia can be associated with pathologic gastroesophageal reflux.

The competency of the cardia depends on an interrelationship between:

1) distal esophageal sphincter pressure
2) length of sphincter exposed to positive-pressure environment of abdomen
3) overall sphincter length

A deficiency in any one of these is associated with incompetency of the cardia regardless of whether a hernia is present.

How well did you know this?
1
Not at all
2
3
4
5
Perfectly
93
Q

Indications for surgical repair for hiatal hernias?

A

Patients with PEH are generally counseled to have elective repair, especially if symptomatic.

How well did you know this?
1
Not at all
2
3
4
5
Perfectly
94
Q

Why do most surgeons advocate routine addition of an antireflux procedure following repair of the hernia defect?

A

1) Physiologic testing with 24h esophageal pH monitoring has shown increased esophageal exposure to acid gastric juice in 60-70% of patients with a PEH, and 71% in those with a sliding hernia.
2) There is no relation between symptoms experienced by the patient with a PEH and the competency of the cardia.
3) A dissection of the gastroesophageal esophagus may lead to postoperative reflux despite a negative preoperative pH score.

How well did you know this?
1
Not at all
2
3
4
5
Perfectly
95
Q

What is a short esophagus?

A

The failure to achieve 2.5cm of intraabdominal esophagus with standard mediastinal dissection techniques.

How well did you know this?
1
Not at all
2
3
4
5
Perfectly
96
Q

What is Collis gastroplasty?

A

A procedure to achieve esophageal lengthening by the creation of a neoesophagus using the gastric cardia.

How well did you know this?
1
Not at all
2
3
4
5
Perfectly
97
Q

What is the most common cause of anatomic failure following laparoscopic Nissen fundoplication done for GERD?

A

Recurrent hiatal hernia (5-10%)

How well did you know this?
1
Not at all
2
3
4
5
Perfectly
98
Q

What is the management for asymptomatic recurrent hernias?

A

Asymptomatic recurrent hernias, like primary PEH, do not need to be repaired. The risk of incarceration, strangulation, or obstruction is minimal.

How well did you know this?
1
Not at all
2
3
4
5
Perfectly
99
Q

What is Schatzki’s ring?

A

Schatzki’s ring is a thin submucosal circumferential ring in the lower esophagus at the squamocolumnar junction, often associated with a hiatal hernia.

Stiennon believed the ring to be a pleat of mucosa formed by infolding of redundant esophageal mucosa due to shortening of the esophagus. Others believe it to be congenital, and still others suggest it is an early stricture resulting from inflammation of the esophageal mucosa caused by chronic reflux.

Symptoms include brief episodes of dysphagia during hurried ingestion of solid foods.

Majority of Schatzki’s ring patients without proven reflux have a history of ingestion of drugs known to be damaging to the esophageal mucosa.

How well did you know this?
1
Not at all
2
3
4
5
Perfectly
100
Q

Treatment of Schatzki’s ring?

A

The best form of treatment of a symptomatic Schatzki’s ring in patients who do not have reflux consists of esophageal dilation for relief of the obstructive symptoms.

In patients with a ring who have proven reflux and a mechanically defective sphincter, an antireflux procedure is necessary to obtain relief and avoid repeated dilation.

How well did you know this?
1
Not at all
2
3
4
5
Perfectly
101
Q

What is scleroderma

A

Scleroderma is a systemic and primarily neurogenic disorder, involving small vessel inflammation and subsequent perivascular deposition of normal collagen, leading to vascular compromise. It is accompanied by esophageal abnormalities in 80% of patients.

In the GI tract, the predominant feature is smooth muscle atrophy. In early scleroderma, an adrenergic overactivity may be present that causes parasympathetic inhibition, supporting a neurogenic mechanism for the disease. Clinically, this manifests in the patient as having a poor esophageal pump and a poor valve.

How well did you know this?
1
Not at all
2
3
4
5
Perfectly
102
Q

How is scleroderma diagnosed?

A

The diagnosis of scleroderma can be made manometrically by the observation of normal peristalsis in the proximal striated esophagus, with absent peristalsis in the distal smooth muscle portion.

LES pressure is progressively weakened as the disease advances.

Gastroesophageal reflux occurs in patients with scleroderma because they have both hypotensive sphincters and poor esophageal clearance. This combined defect can lead to severe esophagitis and stricture formation.

The typical barium swallow shows a dilated, barium-filled esophagus, stomach, and duodenum, or a hiatal hernia with distal esophageal stricture and proximal dilatation..

How well did you know this?
1
Not at all
2
3
4
5
Perfectly
103
Q

How is scleroderma treated?

A

Traditionally, esophageal symptoms have been treated with PPIs, antacids, elevation of the head of bed, and multiple dilations for strictures, with unsatisfactory results. Scleroderma patients have frequently had numerous dilations before they are referred to the surgeon.

The surgical management is controversial, but majority recommend a partial fundoplication (anterior or posterior) performed laparoscopically. Esophageal shortening may require a Collis gastroplasty in combination with a partial fundoplication.

If esophagitis is severe, or there has been a previous failed antireflux procedure and the disease is associated with delayed gastric emptying, a gastric resection with Roux-en-Y gastrojejunostomy has proved the best option.

How well did you know this?
1
Not at all
2
3
4
5
Perfectly
104
Q

What is eosinophilic esophagitis?

A

A condition with immunologic similarity to asthma, a form of allergic esophagitis.

Normally presents with chest pain (often postprandial) and dysphagia, most commonly to solids.

Unlike GERD, it does not respond to proton pump inhibitors.

A barium swallow should be the first test obtained in the patient with dysphagia. EE has a characteristic finding called the “ringed esophagus” or the “feline esophagus” as the esophageal rings are felt to look like the stripes of a housecat.

Histologically, the pathologist should see a minimum of 15 eosinophils per hpf, usually at the base of the epithelium.

How well did you know this?
1
Not at all
2
3
4
5
Perfectly
105
Q

How is Eosinophilic Esophagitis treated?

A

Treatment is symptomatic, and includes testing for food allergies and elimination of identified items from the diet.

Second-line therapy includes inhaled or ingested corticosteroids as would be used to treat asthma. If dysphagia is not relieved by steroids, it may be necessary to dilate the esophagus.

How well did you know this?
1
Not at all
2
3
4
5
Perfectly
106
Q

What are the causes of motility disorders of the pharynx and upper esophagus?

A

Termed transit dysphagia, this is usually congenital or results from acquired disease involving the central and peripheral nervous system (CVA, brainstem tumors, poliomyelitis, multiple sclerosis, Parkinson’s disease, pseudobulbar palsy, peripheral neuropathy, and operative damage to cranial nerves involved in swallowing).

They can be categorized into one or a combination of the ff abnormalities:

1) inadequate oropharyngeal bolus transport
2) inability to pressurize the pharynx
3) inability to elevate the larynx
4) discoordination of pharyngeal contraction and cricopharyngeal relaxation
5) decreased compliance of the pharyngoesophageal segment secondary to neuromuscular disease

How well did you know this?
1
Not at all
2
3
4
5
Perfectly
107
Q

What are the requirements for a successful pharyngoesophageal myotomy?

A

1) Adequate oropharyngeal bolus transport
2) Intact swallowing reflex
3) Reasonable coordination of pharyngeal pressurization with cricopharyngeal relaxation
4) Cricopharyngeal bar, Zenker’s diverticulum, or a narrowed pharyngoesophageal segment on videoesophagogram and/or presence of excessive pharyngoesophageal shoulder pressure on motility study.

How well did you know this?
1
Not at all
2
3
4
5
Perfectly
108
Q

What is Zenker’s diverticulum?

A

Zenker’s diverticulum signifies cricopharyngeal dysfunction, enlarging progressively due to the decreasing compliance of the skeletal portion of the cervical esophagus that occurs with aging.

Symptoms include dysphagia associated with spontaneous regurgitation of undigested, bland material, often interrupting eating or drinking. Occasionally this can be severe enough to cause debilitation and significant weight loss. Chronic aspiration and repetitive respiratory infection are common.

Once suspected, the diagnosis is established by a barium swallow.

How well did you know this?
1
Not at all
2
3
4
5
Perfectly
109
Q

Treatment of Zenker’s diverticulum?

A

The low morbidity and mortality associated with cricopharyngeal and upper esophageal myotomy have encouraged its use for almost any problem in the oropharyngeal phase of swallowing.

If a diverticulum is large enough to persist after a myotomy, it may be sutured in the inverted position to the prevertebral fascia using a permanent suture (ie, diverticulopexy).

If the diverticulum is excessively large, or if its walls are thickened, a diverticulectomy should be performed.

Endoscopic stapled cricopharyngotomy and diverticulotomy are also effective for larger diverticular (>2cm).

How well did you know this?
1
Not at all
2
3
4
5
Perfectly
110
Q

Prognosis of Zenker’s diverticulum?

A

Recurrence of a Zenker’s diverticulum may occur with long follow-up and is more common after diverticulectomy without myotomy, presumably due to persistence of the underlying loss of compliance of the cervical esophagus when a myotomy is not performed.

How well did you know this?
1
Not at all
2
3
4
5
Perfectly
111
Q

What are esophageal motility disorders?

A

Esophageal motility disorders should be looked at as a spectrum of abnormalities that reflect various stages of destruction of esophageal motor function.

These result from abnormalities in the propulsive pump action of the esophageal body or relaxation of the LES. They result from either primary esophageal abnormalities, or from generalized neural, muscular, or collagen vascular disease.

PRIMARY ESOPHAGEAL MOTILITY DISORDERS
Achalasia, "vigorous" achalasia
Diffuse and segmental esophageal spasm
Nutcracker esophagus
Hypertensive lower esophageal sphincter
Nonspecific esophageal motility disorders

SECONDARY ESOPHAGEAL MOTILITY DISORDERS
Collagen vascular diseases: progressive systemic sclerosis, polymyositis and dermatomyositis, mixed connective tissue disease, SLE
Chronic idiopathic intestinal pseudoobstruction
Neuromuscular diseases
Endocrine and metastatic disorders

How well did you know this?
1
Not at all
2
3
4
5
Perfectly
112
Q

Incidence of achalasia?

A

6 in 100,000 population per year

How well did you know this?
1
Not at all
2
3
4
5
Perfectly
113
Q

Pathophysiology of achalasia?

A

Although complete absence of peristalsis in the esophageal body has been proposed as the major abnormality, present evidence indicates achalasia is a primary disorder of the LES.

The pathogenesis of achalasia is presumed to be a neurogenic degeneration, which is either idiopathic or due to infection.

This degeneration leads to hypertension of the LES, a failure of the sphincter to relax on swallowing, elevation of intraluminal esophageal pressure, esophageal dilatation, and a subsequent loss of progressive peristalsis in the body of the esophagus.

The esophageal dilatation results from the combination of a nonrelaxing sphincter, which causes a functional retention of ingested material in the esophagus, and elevation of intraluminal pressure from repetitive pharyngeal air swallowing.

With time, the functional disorder results in anatomic alterations seen on radiographic studies, such as a dilated esophagus with a tapering “bird’s beak”-like narrowing of the distal end.

There is usually an air-fluid level in the esophagus from the retained food and saliva, the height of which reflects the degree of resistance imposed by the nonrelaxing sphincter. As the disease progresses, the esophagus becomes massively dilated and tortuous.

How well did you know this?
1
Not at all
2
3
4
5
Perfectly
114
Q

What is vigorous achalasia?

A

A subgroup of patients with otherwise typical features of classic achalasia has simultaneous contractions of their esophageal body that can be of high amplitude.

This manometric pattern has been termed vigorous achalasia, and chest pain episodes are a common finding in these patients. Since the development of high resolution esophageal manometry technology, the term vigorous achalasia has been replaced with Chicago type 3 achalasia.

Differentiation of type 3 achalasia from DES can be difficult. In both diseases, videographic examination may show a corkscrew deformity of the esophagus and diverticulum formation.

How well did you know this?
1
Not at all
2
3
4
5
Perfectly
115
Q

What is DES?

A

Diffuse Esophageal Spasm (DES) is characterized by substernal chest pain and/or dysphagia. It differs from classic achalasia in that it is primarily a disease of the esophageal body, produces a lesser degree of dysphagia, causes more chest pain, and has less effect on the patient’s general condition.

Nonetheless, it is impossible to differentiate achalasia from DES on the basis of symptoms alone. Esophagogram and esophageal manometry are required to distinguish these two entities. True symptomatic DES is a rare condition, occurring about 5x less frequently than achalasia.

How well did you know this?
1
Not at all
2
3
4
5
Perfectly
116
Q

What is the pathophysiology of DES?

A

The cause and neuromuscular pathophysiology of DES are unclear. The basic motor abnormality is rapid wave progression down the esophagus secondary to an abnormality in the latency gradient.

Hypertrophy of the muscular layer of the esophageal wall and degeneration of the esophageal branches of the vagus nerve have been observed in this disease, although these are not constant findings.

Manometric abnormalities in DES may be present over the total length of the esophageal body, but are usually confined to the distal 2/3.

In segmental esophageal spasm, the manometric abnormalities are confined to a short segment of the esophagus.

How well did you know this?
1
Not at all
2
3
4
5
Perfectly
117
Q

Manometric characteristics of Achalasia?

A

ACHALASIA

1) Incomplete LES relaxation (<75% relaxation)
2) Aperistalsis in the esophageal body
3) Elevated LES pressure ≤26mmHg
4) Increased intraesophageal baseline pressures relative to gastric baseline

How well did you know this?
1
Not at all
2
3
4
5
Perfectly
118
Q

Manometric characteristics of Diffuse Esophageal Spasm (DES)?

A

DES

1) Simultaneous nonperistaltic contractions >20% of wet swallows
2) Repetitive and multipeaked contractions
3) Spontaneous contractions
4) Intermittent normal peristalsis
5) Contractions may be of increased amplitude and duration

Key to the diagnosis of DES is that there remain some peristaltic waveforms in excess of those seen in achalasia.

The LES shows a normal resting pressure and relaxation on swallowing.

In patients with advanced disease, the radiographic appearance of tertiary contractions appears helical and has been termed “corkscrew esophagus” or “pseudodiverticulosis” (patients with segmental or diffuse esophageal spasm can compartmentalize the esophagus and develop an epiphrenic or midesophageal diverticulum between two areas of high pressure occuring simultaneously).

How well did you know this?
1
Not at all
2
3
4
5
Perfectly
119
Q

Manometric characteristics of Nutcracker Esophagus?

A

NUTCRACKER ESOPHAGUS

1) Mean peristaltic amplitude (10 wet swallows) in distal esophagus ≥180mmHg
2) Increased mean duration of contractions (>7.0s)
3) Normal peristaltic sequence

How well did you know this?
1
Not at all
2
3
4
5
Perfectly
120
Q

Manometric characteristics of Hypertensive Lower Esophageal Sphincter?

A

HYPERTENSIVE LES

1) Elevated LES pressure ≥26mmHg
2) Normal LES relaxation
3) Normal peristalsis in the esophageal body

How well did you know this?
1
Not at all
2
3
4
5
Perfectly
121
Q

Manometric characteristics of nonspecific/ineffective esophageal motility disorders?

A

INEFFECTIVE/NONSPECIFIC ESOPHAGEAL MOTILITY DISORDERS

1) Decreased or absent amplitude of esophageal peristalsis (<30mmHg)
2) Increased number of nontransmitted contractions
3) Increased number of multipeaked/repetitive contractions
4) Contractions of prolonged duration
5) Contractions of low amplitude

The finding of a nonspecific esophageal motility disorder may represent only a manometric marker of an intermittent, more severe esophageal motor abnormality.

High amplitude peristalsis (nutcracker esophagus) and low-amplitude peristalsis (ineffective esophageal motility) are frequently associated with GERD.

Surgery plays no role in these disorders, unless there is an associated diverticulum.

How well did you know this?
1
Not at all
2
3
4
5
Perfectly
122
Q

What is the most common of the primary esophageal motility disorders?

A

Nutcracker Esophagus

How well did you know this?
1
Not at all
2
3
4
5
Perfectly
123
Q

What is a Nutcracker Esophagus?

A

Nutcracker or supersqueezer esophagus is a manometric abnormality in patients who are characterized by peristaltic esophageal contractions with peak amplitudes greater than two SDs above the normal values. Contraction amplitudes can easily be above 400mmHg.

Treatment should be aimed at the treatment of GERD. Chest pain symptoms (at the lower end of peak pressure) may be related to GERD rather than intraluminal hypertension.

At the high end (peak pressures >300mmHg), chest pain may be the result of the nutcracker physiology, as treatment directed at reducing intraluminal pressure is more effective than when used for those with lower peak pressures.

How well did you know this?
1
Not at all
2
3
4
5
Perfectly
124
Q

What is Hypertensive Lower Esophageal Sphincter?

A

Hypertensive LES is characterized by an elevated basal pressure of the LES with normal relaxation and normal propulsion in the esophageal body.

About one-half of these patients, however, have associated motility disorders of the esophageal body, particularly hypertensive peristalsis and simultaneous waveforms. In the remainder, the disorder exists as an isolated abnormality.

Dysphagia in these patients may be cause by a lack of compliance of the sphincter, even in its relaxed state.

Myotomy of the LES may be indicated in patients unresponsive to medical therapy of dilation.

When the symptom contribution of the hypertensive sphincter is in doubt, it is possible to inject the LES with botulinum toxin, endoscopically. If symptoms are relieved (temporarily) with this technique, then it is likely that myotomy will provide more permanent benefit.

How well did you know this?
1
Not at all
2
3
4
5
Perfectly
125
Q

What are the causes behind secondary esophageal motility disorders?

A

Connective tissue disease (scleroderma, CREST syndrome) and infants treated for Esophageal atresia often develop motility disorders that manifest later in life.

Symptoms include heartburn and dysphagia. The latter may be a result of a peptic stricture rather than the esophageal dysmotility.

An esophageal motility study will usually show severely reduced or absent LES pressure.

The role of antireflux surgery under these conditions is controversial but if performed, should be limited to partial fundoplication, as full Nissen fundoplication may result in severe dysphagia.

How well did you know this?
1
Not at all
2
3
4
5
Perfectly
126
Q

What are pulsion diverticula?

A

Pulsion diverticula are those associated with motor disorders. They occur most commonly with nonspecific motility disorders, but can occur also with primary motility disorders.

In primary motility disorders, the motility disorder is usually diagnosed before the development of the diverticulum. When associated with achalasia, the development of a diverticulum may temporarily alleviate the symptom of dysphagia by becoming a receptacle for ingested food and substitute the symptom of dysphagia for postprandial pain and regurgitation of undigested food.

If a motility abnormality of the esophageal body or LES cannot be identified, a traction or congenital cause for the diverticulum should be considered.

How well did you know this?
1
Not at all
2
3
4
5
Perfectly
127
Q

What are epiphrenic diverticula?

A

Traditionally, diverticula were considered a primary abnormality. Hence they were categorized based on location.

Epiphrenic diverticula arise from the terminal third of the thoracic esophagus and are usually found adjacent to the diaphragm. They have been associated with distal esophageal muscular hypertrophy, esophageal motility abnormalities, and increased luminal pressure. They are pulsion diverticula and are associated with diffuse spasm, achalasia, or nonspecific motor abnormalities in the esophageal body.

How well did you know this?
1
Not at all
2
3
4
5
Perfectly
128
Q

What is the treatment for pulsion diverticula?

A

When diverticula are associated with esophageal motility disorders, esophageal myotomy from the proximal extent of the diverticulum to the stomach should be combined with diverticulectomy. (If diverticulectomy alone is performed, one can expect a high incidence of suture line rupture due to the same intraluminal pressure that initially gave rise to the diverticulum.)

If the diverticulum is suspended to the prevertebral fascia of the thoracic vertebra, a myotomy is begun at the neck of the diverticulum and extended across the LES. If the diverticulum is excised by dividing the neck, the muscle is closed over the excision site, and a myotomy is performed on the opposite esophageal wall, starting just above the level of the diverticulum or at the proximal extent of the spastic segment of the esophagus if high resolution motility is used. If complete, the myotomy will cross the LES, reducing distal esophageal peak pressure, and it will increase the likelihood that dysphagia will be replaced with GERD symptoms.

Increasingly, partial fundoplication is performed after LES myotomy to decrease the frequency of disabling GERD developing after myotomy and diverticulectomy.

When a large diverticulum is associated with a hiatal hernia, then hiatal hernia repair is added.

How well did you know this?
1
Not at all
2
3
4
5
Perfectly
129
Q

What are traction diverticula?

A

Midesophageal or traction diverticula were theorized to come from adhesions forming between the inflamed mediastinal nodes and esophagus of TB patients. By contraction, the adhesions exerted traction on the esophageal wall and led to a localized diverticulum.

Most of these are asymptomatic and incidentally discovered during investigation of nonesophageal complaints. In such patients, the radiologic abnormality may be ignored. Those with dysphagia, regurgitation, chest pain, or aspiration should be thoroughly investigated for an esophageal motor abnormality.

Occasionally, a patient will present with a bronchoesophageal fistula manifested by a chronic cough on ingestion of meals. The diverticulum in such patients is most likely to have an inflammatory etiology.

How well did you know this?
1
Not at all
2
3
4
5
Perfectly
130
Q

What is the treatment for traction diverticula?

A

The indication for surgery is dictated by the degree of symptomatic disability.

Usually, midesophageal diverticula can be suspended due to their proximity to the spine.

If a motor abnormality is documented, a myotomy should be performed as described for an epiphrenic diverticulum.

How well did you know this?
1
Not at all
2
3
4
5
Perfectly
131
Q

Indications for a long esophageal myotomy?

A

Used for motor disorders of the esophageal body. Indicated for dysphagia caused by any motor disorder characterized by segmental or generalized simultaneous waveforms in a patient whose symptoms are not relieved by medical therapy.

Diffuse and segmental esophageal spasm, vigorous or type 3 achalasia, and nonspecific motility disorders associated with a mid- or epiphrenic esophageal diverticulum.

How well did you know this?
1
Not at all
2
3
4
5
Perfectly
132
Q

What is POEM?

A

Peroral Endoscopic Myotomy (POEM) allows a long myotomy to be performed entirely from the lumen of the esophagus with an endoscope. It is minimally invasive and can be done outpatient.

This procedure may be used for those with type 3 achalasia (vigorous achalasia), where it is necessary to divide the esophagogastric circular muscle on both sides of the diaphragm to the extent that might not be possible with laparoscopy or thoracoscopy alone.

The major downside of POEM is that an effective antireflux valve cannot be created, exposing the patient to a 40-50% risk of GERD post procedure.

How well did you know this?
1
Not at all
2
3
4
5
Perfectly
133
Q

What is Heller Myotomy?

A

Heller Myotomy or Myotomy of the Lower Esophageal Sphincter (LES) aims to relieve the functional outflow obstruction secondary to loss of relaxation and compliance of the LES in achalasia (ie, reducing sphincter pressure to <10mmHg).

Other options for addressing LES include hydrostatic balloon dilation (similar outcome with surgical myotomy), botulinum toxin injection.

How well did you know this?
1
Not at all
2
3
4
5
Perfectly
134
Q

What is the most common functional disorder second to GERD?

A

Achalasia

How well did you know this?
1
Not at all
2
3
4
5
Perfectly
135
Q

Treatment options for Achalasia?

A

MEDICAL OR SURGICAL?
For newly diagnosed patients, pneumatic dilation achieves adequate relief of dysphagia and pharyngeal regurgitation in 50-60% of patients. If dilation fails, myotomy is indicated.

For those with a dilated and tortuous esophagus or an associated hiatal hernia, balloon dilation is dangerous and surgery (surgical myotomy) is the better option.

The best treatment for achalasia is a laparoscopic Heller myotomy with partial fundoplication.

APPROACH
In the absence of a previous upper abdominal surgery, most surgeons prefer the abdominal approach (less pain, shorter length of stay). It is also a bit easier to ensure a long gastric myotomy when the approach is transabdominal.

ARS
Excellent results have been reported following meticulously performed myotomy without an antireflux component. If an antireflux procedure is used as an adjunct to esophageal myotomy, a complete 360-deg fundoplication should be avoided. Rather, a 270-deg Belsey fundoplication, a Toupet posterior 180-deg fundoplication, or a Dor anterior 180-deg fundoplication should be used to avoid the long term esophageal dysfunction secondary to outflow obstruction afforded by the fundoplication itself.

PROGNOSIS
The earlier an effective reduction in outflow resistance can be accomplished, the better the outcome will be, and the more likely some esophageal body function can be restored.

How well did you know this?
1
Not at all
2
3
4
5
Perfectly
136
Q

Principles in performing a surgical myotomy of the LES?

A

1) Complete division of all circular and collar-sling muscle fibers
2) Adequate distal myotomy to reduce outflow resistance
3) Undermining of the muscularis to allow wide separation of the esophageal muscle
4) Prevention of postoperative reflux

How well did you know this?
1
Not at all
2
3
4
5
Perfectly
137
Q

What antireflux procedures are used to follow completion of a laparoscopic Heller myotomy?

A

Either an anterior hemifundoplication augmenting the angle of His (Dor) or posterior partial fundoplication (Toupet) can be performed.

The Dor type is slightly easier to perform, and does not require disruption of the normal posterior gastroesophageal attachments (theoretical advantage in preventing postoperative reflux).

In most RCTs, laparoscopic Heller myotomy and partial fundoplication was superior to the alternative treatments.

How well did you know this?
1
Not at all
2
3
4
5
Perfectly
138
Q

What is the most common esophageal carcinoma worldwide?

A

Squamous carcinoma of the esophagus

How well did you know this?
1
Not at all
2
3
4
5
Perfectly
139
Q

Risk factors for SCC of the esophagus?

A

Additives (nitrous compounds in pickled vegetables and smoked meat)
Mineral deficiencies (zinc and molybdenum)
Smoking
Alcohol
Long-standing achalasia
Lye strictures
Tylosis (autosomal dominant disorder with keratosis of palms and soles)
HPV

How well did you know this?
1
Not at all
2
3
4
5
Perfectly
140
Q

Most important etiologic factor in the development of adenocarcinoma of the esophagus?

A

Metaplastic, columnar-lined or Barrett’s esophagus

occurs in 10-15% of patients with GERD

How well did you know this?
1
Not at all
2
3
4
5
Perfectly
141
Q

Why does dysphagia present late in esophageal cancer?

A

Dysphagia usually presents late in the natural history of the disease because the lack of a serosal layer on the esophagus allows the smooth muscle to dilate with ease. As a result, the dysphagia becomes severe enough for the patient to seek medical advice only when more than 60% of the esophageal circumference is infiltrated with cancer.

How well did you know this?
1
Not at all
2
3
4
5
Perfectly
142
Q

General approach to Esophageal Cancer?

A

Esophageal cancer is diagnosed with endoscopic biopsy and is staged with CT of the chest and abdomen, EUS, and PET scan for all patients with CT or EUS evidence of advanced disease (T2 or greater, N1-2 or NX).

T in situ, T1a, intramucosal cancer
- Endoscopic treatment (EMR)

T1-T2, N0: Confined to esophagus (submucosal invasion)
- Esophagectomy with LN dissection

T1-T3, N1: Locally advanced
- Multimodality approach
(neoadjuvant chemo + surgery)
(chemoRT + surgery)

Any T, Any N, M1: Disseminated
- Palliation (RT, endoscopic placement of expandable esophageal stent)

How well did you know this?
1
Not at all
2
3
4
5
Perfectly
143
Q

Most common sites of esophageal CA mets?

A
Lung
Liver
Peritoneal surfaces
Omentum
Small bowel mesentery
How well did you know this?
1
Not at all
2
3
4
5
Perfectly
144
Q

Indications for curative resection based on endoscopic ultrasound of the esophagus?

A

EUS provides the most reliable method of determining cancer depth of invasion. A curative resection should be encouraged if EUS indicates that the tumor has not invaded adjacent organs (T4b), and/or fewer than 6 enlarged LNs are imaged.

How well did you know this?
1
Not at all
2
3
4
5
Perfectly
145
Q

AJCC Staging for Esophageal Cancer?

A

TX: Primary tumor cannot be assessed.
T0: No evidence of primary tumor.
Tis: High-grade dysplasia

T1: Tumor invades lamina propria, muscularis mucosae, or submucosa.

  • T1a: Tumor invades lamina propria or muscularis mucosae.
  • T1b: Tumor invades submucosa

T2: Tumor invades muscularis propria.
T3: Tumor invades adventitia.

T4: Tumor invades adjacent structures.

  • T4a: Resectable tumor invading pleura, pericardium, or diaphragm.
  • T4b: Unresectable tumor invading other adjacent structures, such as aorta, vertebral body, trachea, etc.
NX: Regional LN cannot be assessed.
N0: No regional LN mets.
N1: Mets in 1-2 regional LNs.
N2: Mets in 3-6 regional LNs.
N3: Mets in ≥7 regional LNs.

M0: No distant mets.
M1: Distant mets.

How well did you know this?
1
Not at all
2
3
4
5
Perfectly
146
Q

Clinical Staging for Squamous Cell CA of the Esophagus?

A

Esophageal SCC
Stage
0: Tis N0 M0

I: T1 N0-1 M0

II: T2 N0-1 M0
T3 N0 M0

III: T3 N1 M0
T1-3 N2 M0

IVA: T4 N0-2 M0
Any T N3 M0

IVB: Any T Any N M1

How well did you know this?
1
Not at all
2
3
4
5
Perfectly
147
Q

Clinical Staging for Adenocarcinoma of the Esophagus?

A

Esophageal AdenoCA
Stage
0: Tis N0 M0

I: T1 N0 M0

IIA: T1 N1 M0

IIB: T2 N0 M0

III: T2 N1 M0
T3 N0-1 M0
T4a N0-1 M0

IVA: T1-4a N2 M0
T4b N0-2 M0
Any T N3 M0

IVB: Any T Any N M1

How well did you know this?
1
Not at all
2
3
4
5
Perfectly
148
Q

Approach to cervical esophageal cancer?

A

8% of primary malignant tumors of the esophagus occur in the cervical portion, and are almost always SCC. They are more common in females.

The efferent lymphatics from the cervical esophagus drain completely differently from those of the thoracic esophagus. The latter drain directly into the paratracheal and deep cervical or internal jugular LNs with minimal flow in a longitudinal direction.

Cervical esophageal CA is frequently unresectable because of early invasion of the larynx, great vessels, or trachea. Hence stereotactic radiation with concomittant chemotherapy is the most desirable treatment.

How well did you know this?
1
Not at all
2
3
4
5
Perfectly
149
Q

Approach to esophageal CA in the middle third (midthoracic)?

A

These are mostly SCC, and are frequently associated with LN mets.

Generally, T1 and T2 cancers without LN mets are treated with resection only, but there is more and more data to suggest LN involvement or transmural cancer (T3) warrants treatment with neoadjuvant chemoradiation therapy followed by resection.

Although some surgeons prefer a transhiatal esophagectomy, most surgeons believe that resection of midesophageal cancer should be performed under direct vision with either thoracoscopy (VATS) or thoracotomy.

How well did you know this?
1
Not at all
2
3
4
5
Perfectly
150
Q

Approach to esophageal CA in the lower esophagus and cardia?

A

These are usually adenocarcinomas. Unless staging demonstrates an incurable lesion, resection in continuity with a LN dissection should be performed.

Because of the propensity of GI tumors to spread for long distances submucosally, long lengths of grossly normal GI tract should be resected.

The longitudinal lymph flow in the esophagus can result in skip areas, with small foci of tumor above the primary lesion, which underscores the importance of a wide resection of esophageal tumors. (*Wong has shown that local recurrence at the anastomosis can be prevented by obtaining a 10cm margin of normal esophagus above the tumor. Also, there is no submucosal lymphatic barrier between the esophagus, stomach and cardia, hence 50% of local recurrences post resection occur in the intrathoracic stomach along the line of gastric resection.)

How well did you know this?
1
Not at all
2
3
4
5
Perfectly
151
Q

How long is the esophagus?

A

17-25cm

How well did you know this?
1
Not at all
2
3
4
5
Perfectly
152
Q

What does a curative resection for distal esophageal lesions entail?

A

A curative resection requires a cervical division of the esophagus and a >50% proximal gastrectomy in most patients with carcinoma of the distal esophagus or cardia.

How well did you know this?
1
Not at all
2
3
4
5
Perfectly
153
Q

How does age factor in when approaching esophageal CA?

A

Resection for cure of esophageal CA in a patient older than 80 years old is rarely indicated because of the additional operative risk and the shorter life expectancy. However octogenarians with a high-performance status and excellent cardiopulmonary reserve may be considered candidates for esophagectomy.

How well did you know this?
1
Not at all
2
3
4
5
Perfectly
154
Q

Incidence of esophageal carcinoma by location?

A
Cervical 8%
Upper thoracic 3%
Middle thoracic 32%
Lower thoracic 25%
Cardia 32%
How well did you know this?
1
Not at all
2
3
4
5
Perfectly
155
Q

Functional grades of dysphagia?

A

I: Eating normally (11%)
II: Requires liquids with meals (21%)
III: Able to take semisolids but unable to take solid food (30%)
IV: Able to take liquids only (40%)
V: Unable to take liquids, but able to swallow saliva (7%)
VI: Unable to swallow saliva (12%)

Grades I-III can generally be palliated with RT, in combination with chemotherapy (definitive chemoradiation therapy, meaning surgery is not anticipated in the future). Radiation is increased from 45 Gy to 60 Gy, administered over 8 weeks, rather than 4 weeks for induction therapy.

Grade IV and higher necessitates additional treatment, the mainstay of which is indwelling esophageal stents. However, if cancers involve the GEJ, a stent may cause disabling reflux and heartburn; hence RT may be preferable for cancers at this level. If feeding access is desirable, laparoscopic jejunostomy is the procedure of choice.

How well did you know this?
1
Not at all
2
3
4
5
Perfectly
156
Q

How does cardiopulmonary reserve factor in when approaching esophageal CA?

A

Cardiopulmonary reserve is assessed with FEV in 1 second (≥ 2L).

A patient with an FEV in 1 second of <1.25L is a poor candidate for esophageal resection.

Poor pulmonary reserve: TRANSHIATAL Esophagectomy (less pulmonary morbidity)

Most individuals who can climb 3 flights of stairs without stopping will do well with two-field open esophagectomy especially with an epidural catheter for postoperative pain relief.

How well did you know this?
1
Not at all
2
3
4
5
Perfectly
157
Q

What is the factor most predictive of postoperative complications in esophageal resection?

A

Nutritional status. Profound weight loss (>20lb) with hypoalbuminemia (albumin <3.5g/dL) is associated with a much higher rate of complications and mortality.

Because most malnourished patients have locally advanced esophageal cancer, placement of a feeding tube prior to induction chemoRT should be considered.

How well did you know this?
1
Not at all
2
3
4
5
Perfectly
158
Q

Clinical indications of advanced esophageal carcinoma (poor candidates for surgery)?

A
Recurrence nerve paralysis
Horner's syndrome
Persistent spinal pain
Paralysis of the diaphragm
Fistula formation
Malignant effusion
Tumor >8cm in length
Abnormal axis of the esophagus on barium
>4 enlarged LNs in CT
Weight loss >20%
Loss of appetite
How well did you know this?
1
Not at all
2
3
4
5
Perfectly
159
Q

Quickest surgical procedure to perform for esophageal cancer?

A

Open transhiatal esophagectomy

How well did you know this?
1
Not at all
2
3
4
5
Perfectly
160
Q

Most radical resection?

A

Ivor-Lewis (En Bloc) Esophagectomy

The operation is started in the abdomen with an upper midline laparotomy and extensive LN dissection. Following closure of the abdominal incision, the patient is placed in the left lateral decubitus position, and an anterolateral thoracotomy is performed through the 6th ICS.

Because this is the most radical of dissections, complications are most common, including pneumonia, respiratory failure, AF, chylothorax, anastomotic leak, conduit necrosis, fistula, recurrent laryngeal nerve injury. However this also has a long-term outcome with the greatest survival.

How well did you know this?
1
Not at all
2
3
4
5
Perfectly
161
Q

What is salvage esophagectomy?

A

Salvage esophagectomy is the term used for esophagectomy performed after failure of definitive RT and chemotherapy. This is often done when distant disease renders the patient nonoperable at initial presentation. Then, systemic chemo destroys all foci of mets as shown by CT and PET, but the primary remains symptomatic and present.

1 in 4 patients undergoing this operation will be disease-free 5 years later.

Because of dense scarring by radiation treatment, this is the most challenging technically among all esophagectomy techniques.

How well did you know this?
1
Not at all
2
3
4
5
Perfectly
162
Q

What type of tumor constitutes more than 50% of benign esophageal tumors?

A

Leiomyomas constitute more than 50% of benign esophageal tumors. The average age at presentation is 38, and these are twice as common in males.

These are usually solitary and usually intramural, with 90% located in the lower 2/3. The overlying mucosa is freely movable and normal in appearance, with dysphagia and pain as the most common complaints.

A barium swallow is the most useful method to demonstrate a leiomyoma of the esophagus (smooth, semilunar, or crescent-shaped filling defect that moves with swallowing, sharply demarcated, covered and surrounded by normal mucosa).

Esophagoscopy should also be performed to exclude the reported observation of a coexistence with carcinoma.

Biopsy should NOT be done (increased chance of mucosal perforation). Endoscopic ultrasound is also a useful adjunct in the workup of leiomyomas.

Leiomyomas should be removed by simple enucleation.

How well did you know this?
1
Not at all
2
3
4
5
Perfectly
163
Q

What is the surgical approach for leiomyomas of the esophagus?

A

Lesions of the proximal and middle esophagus require a right thoracotomy, whereas distal esophageal lesions require a left thoracotomy.

Large lesions or those involving the GEJ may require esophageal resection.

How well did you know this?
1
Not at all
2
3
4
5
Perfectly
164
Q

Discuss esophageal cysts.

A

Congenital cysts are lined wholly or partly by columnar ciliated respiratory-type epithelium, glandular gastric-type epithelium, squamous epithelium, or transitional epithelium.

Acquired retention cysts also occur probably secondary to obstruction of the excretory ducts of the esophageal glands.

Enteric and bronchogenic cysts are the most common secondary to developmental abnormalities during the formation and differentiation of the lower respiratory tract, esophagus and stomach from the foregut.

Cysts are usually located intramurally in the middle to lower third of the esophagus, and present clinically similar to leiomyomas.

Surgical excision by enucleation is the preferred treatment. During removal, a fistulous tract connecting the cysts to the airways should be sought, especially in patients with repeated bronchopulmonary infections.

How well did you know this?
1
Not at all
2
3
4
5
Perfectly
165
Q

How do you diagnose esophageal perforation?

A

Perforation of the esophagus constitutes a true emergency. Spontaneous perforation, known as Boerhaave’s syndrome, comprises only 15% of cases of esophageal perforation (14% FB, 10% trauma).

Pain is a striking and consistent symptom and strongly suggests rupture (cervical area following instrumentation of the esophagus, or substernally in a patient with a history of resisting vomiting). If subcutaneous emphysema is present, the diagnosis is almost certain.

CXR may show air or an effusion in the pleural space (abnormalities may differ based on time interval, site of perforation, and integrity of mediastinal pleura):

TIME
Mediastinal emphysema, strong indicator of perforation, takes at least 1 hour to be demonstrated and is present in only 40% of patients. Mediastinal widening secondary to edema may not occur for several hours. Air in the deep muscles (erector spinae) of the neck is often the earliest sign of perforation and can be present without evidence of air in the mediastinum.

SITE
Cervical perforation: Cervical emphysema
Thoracic perforation: Mediastinal emphysema

INTEGRITY OF MEDIASTINAL PLEURA
Pleural rupture results in a PNEUMOTHORAX (77%)
2/3: Left-sided perforation
1/5: Right-sided perforation
1/10: Bilateral

Intact pleura results in MEDIASTINAL EMPHYSEMA
A pleural effusion secondary to inflammation of the mediastinum occurs late.
In 9% of patients, CXR is normal.

Diagnosis is confirmed with a CONTRAST ESOPHAGOGRAM (Extravasation in 90% of patients). The use of a water-soluble medium such as Gastrografin is preferred (ideally in right lateral decubitus position– if upright, there is a 10% false negative rate as the passage of contrast can be too rapid).

Elevation of pleural amylase may occur from extrusion of saliva through the perforation.

How well did you know this?
1
Not at all
2
3
4
5
Perfectly
166
Q

Common locations for esophageal perforation?

A

Into the left pleural cavity or just above the GEJ. The most common location is the left lateral wall of the esophagus, just above the GEJ.

50% of patients have concomitant GERD, suggesting that minimal resistance to the transmission of abdominal pressure into the thoracic esophagus is a factor in the pathophysiology of the lesion.

How well did you know this?
1
Not at all
2
3
4
5
Perfectly
167
Q

When does a Mallory-Weiss mucosal tear happen?

A

When a hiatal hernia is present and the sphincter remains exposed to abdominal pressure, the lesion produced is usually a Mallory-Weiss mucosal tear, and bleeding rather than perforation is the problem.

This is due to the stretching of the supradiaphragmatic portion of the gastric wall. In this situation, the hernia sac represents an extension of the abdominal cavity, and the GEJ remains exposed to abdominal pressure.

168
Q

Management of esophageal perforation?

A

Early diagnosis is key. The most favorable outcome is obtained following primary closure of the perforation with in 24hours, resulting in 80-90% survival.

For adequate exposure, a dissection similar to a myotomy is performed. A flap of stomach is pulled up and the soiled fat pad at the GEJ is removed. The edges of the injury are trimmed and closed primarily. Closure is reinforced with a pleural patch or construction of a Nissen fundoplication.

If the time delay before closing a perforation approached 24h and the tissues are inflamed, division of the cardia and resection of the diseased portion of the esophagus are recommended. The remainder of the esophagus is mobilized, and as much normal esophagus as possible is saved and brought out as an end cervical esophagostomy.

If retained esophagus is so long that it loops down into the chest, the contaminated mediastinum is drained and a feeding jejunostomy tube is inserted.

Nonoperative management has been advocated in select situations. The choice of conservative therapy requires skillful judgment and necessitates careful radiographic examination of the esophagus. This follows an injury occurring during dilation of esophageal strictures or pneumatic dilations of achalasia.

Conservative management should NOT be used on patients who have free perforations into the pleural space.

169
Q

Prognosis of esophageal perforation?

A

Mortality associated with immediate closure varies between 8 and 20%. After 24h, survival decreases to <50%, and is not influenced by the type of operative therapy.

The recovery from sepsis is often immediate, dramatic, and reflected by a marked improvement in the patient’s condition over a 24-hour period.

On recovery from sepsis, the patient is discharged and returns on a subsequent date for reconstruction with a substernal colon interposition. Failure to apply this aggressive therapy can result in a mortality rate in excess of 50% in those with a delayed diagnosis.

170
Q

3 criteria for nonoperative management of esophageal perforation?

A

Conservative management should NOT be used on patients who have free perforations into the pleural space.

Cameron proposed three criteria for NOM of esophageal perforation:
1) Esophagogram must show the perforation to be contained within the mediastinum and drain well back into the esophagus.

2) Symptoms should be mild, and
3) There should be minimal evidence of clinical sepsis.

If these are met, it is reasonable to treat the patient with hyperalimentation, antibiotics, and cimetidine to decrease acid secretion and diminish pepsin activity. Oral intake is resumed in 7-14 days, dependent on subsequent radiographic examinations.

171
Q

What is Mallory-Weiss syndrome?

A

Acute upper GI bleeding following vomiting, which is considered to be the cause of up to 15% of all severe upper GI bleeds. The mechanism is similar to spontaneous esophageal perforation: an acute increase in intraabdominal pressure against a closed glottis in a patient with a hiatal hernia. The tears are characterized by arterial bleeding, which may be massive.

The diagnosis needs a high index of suspicion, particularly in the patient who develops upper GI bleeding following prolonged vomiting or retching.

Upper endoscopy confirms the suspicion by identifying one or more longitudinal fissures in the mucosa of the herniated stomach as the source of bleeding.

In majority of patients, bleeding stops spontaneously with nonoperative management. In addition to blood replacement, the stomach should be decompressed and antiemetics administered, as a distended stomach and continued vomiting aggravate further bleeding.

A Sengstaken-Blakemore tube will not stop the bleeding, as the pressure is not sufficient to overcome arterial pressure. Endoscopic injection of epinephrine may be therapeutic if bleeding does not stop spontaneously.

Only occasionally will surgery be required to stop blood loss (laparotomy and high gastrostomy with oversewing of the linear tear).

Mortality is uncommon, and recurrence is rare.

172
Q

Epidemiology of caustic injury?

A

Children: Most likely accidental
Adults/teenagers: Most likely deliberate

Alkalis are more frequently swallowed accidentally than acids, because strong acids cause an immediate burning pain in the mouth.

173
Q

Pathology of caustic injury?

A

Acute injury entails care to focus on controlling immediate tissue injury and potential for perforation. The degree and lesion’s extent depends on: the nature of caustic substance, concentration, quantity swallowed, and time the substance is in contact with tissues.

Chronic injury entails a focus on treatment of strictures and disturbances in pharyngeal swallowing.

ALKALIS
Dissolve tissue and penetrate more deeply.

ACIDS
Cause a coagulative necrosis that limits their penetration.

The esophagus is more severely and preferentially affected at the level of striated-muscle-smooth-muscle interface (weakest point of contractions, slower clearance).

174
Q

Lesions caused by lye injury in caustic ingestion?

A

The lesions caused by lye injury occur in 3 phases.

1) Acute necrotic phase: 1-4 days post injury
- Coagulation of intracellular proteins results in cell necrosis, and living tissue develops an intense inflammatory reaction.

2) Ulceration and granulation phase: 3-5 day post injury
- Superficial necrotic tissue sloughs, leaving an ulcerated, acutely inflamed base, and granulation tissue fills the defect left by the sloughed mucosa.
- Lasts 10-12 days
- Esophagus is weakest at this period.

3) Cicatrization and Scarring (starts 3rd week post-injury)
- formed connective tissue begins to contract, resulting in narrowing of esophagus
- Adhesions betweeen granulating areas occur, resulting in pockets and bands.
- Efforts must be made at this time to reduce stricture formation.

175
Q

Clinical presentation of esophageal burn secondary to caustic injury?

A

Initial/acute phase

  • Pain in the mouth and substernal region
  • Hypersalivation
  • Pain on swallowing
  • Dysphagia
  • Fever (consider esophageal lesion)
  • Bleeding
  • Vomiting

Ulceration and granulation
- Complaints disappear

Cicatrization and scarring

  • Dysphagia reappears secondary to fibrosis and retraction, leading to narrowing of the esophagus
  • Of those who develop stricture formation: 60% do so in 1 month, and 80% do so in 2 months
  • If dysphagia does not develop within 8 months, it is unlikely that a stricture will occur.
  • Serious systemic reactions such as hypovolemia and acidosis resulting in renal damage can occur in cases in which the burns have been caused by strong acids.
  • Respiratory complications secondary to aspiration can happen.
176
Q

Diagnosis of esophageal injury?

A

Consider that esophageal burns can be present without apparent oral injuries. Hence early esophagoscopy is advocated to identify injuries (within 12 h).

Radiographic examination is not a reliable means to identify the presence of early esophageal injury, but it is important in later follow-up to identify strictures.

177
Q

Endoscopic grading of corrosive esophageal and gastric burns?

A

First degree: Mucosal hyperemia and edema

Second degree: Limited hemorrhage, exudate ulceration, and pseudomembrane formation

Third degree: Sloughing of mucosa, deep ulcers, massive hemorrhage, complete obstruction of lumen by edema, charring, perforation

178
Q

Acute phase treatment of caustic injury?

A

ACUTE PHASE

  • Correct hypovolemia
  • Broad spectrum antibiotics
  • Do an esophagoscopy within 12hours, and manage accordingly:

1st degree burn

  • Lye/alkali: Neutralize with 1/2 strength vinegar, lemon juice, or orange juice
  • Acid: Neutralize with milk, egg white, or antacids
  • Feeding jejunostomy tube for nutrition as necessary
  • Oral feeding can be started when the dysphagia of the initial phase has regressed.

2nd and 3rd degree burns

  • Exploratory laparotomy:
    a) Viable esophagus and stomach:
    • Intraluminal esophageal stent
    • Posterior gastric wall biopsy
    • Jejunostomy

b) Questionable esophagus and stomach
- - Second look at 36hours

c) Full thickness necrosis of esophagus and stomach
- - Esophagogastric resection
- - Cervival esophagostomy
- - Jejunostomy
- - Resection of adjacent involved organs

  • Extensive necrosis: resection
  • Extensive gastric involvement: Total gastrectomy and near-total esophagectomy
  • Air in the esophageal wall– sign of muscle necrosis and impending perforation, and is a strong indication for esophagectomy
  • Intraluminal esophageal stent: For those operated on with no evidence of extensive esophagogastric necrosis. A biopsy of the posterior gastric wall should be done to rule out occult injury, and any questionable viability by histology should prompt a second look operation within 36h. The stent should be maintained for 21 days and removed after a satisfactory barium esophagogram.

*CONTRAINDICATED:
Sodium bicarbonate is NOT used because it generates CO2, which might increase the danger of perforation.
- Emetics (vomiting renews contact of caustic substance with esophagus)

179
Q

Preventive/management of stricture prevention in caustic injury?

A

Antegrade dilation with a Hurst or Maloney bougie, and retrograde dilation with a Tucker bougie.

An adequate lumen should be re-established within 6 months to 1 year, with progressively longer intervals between dilations. Otherwise operative intervention should be considered.

180
Q

Indications for surgical intervention in stricture management secondary to caustic injuries?

A

Surgical intervention is indicated when:

1) Complete stenosis, in which all attempts from above and below have failed to establish a lumen
2) Marked irregularity and pocketing on barium swallow
3) Development of a severe periesophageal reaction, or mediastinitis with dilatation
4) Fistula
5) The inability to dilate or maintain the lumen above a 40F bougie
6) A patient who is unwilling or unable to undergo prolonged periods of dilation.

181
Q

Options for an esophageal substitute for esophageal reconstruction?

A

In order of preference:

1) Colon
2) Stomach
3) Jejunum

182
Q

Options for the site of proximal anastomosis in esophageal reconstruction?

A

When the cervical esophagus is destroyed, and a pyriform sinus remains open, the anastomosis can be made to the hypopharynx.

When the pyriform sinuses are completely stenosed, a transglottic approach is used to perform an anastomosis to the posterior oropharyngeal wall.

183
Q

Most common cause of esophageal fistulas?

A

Most acquired esophageal fistulas are to the tracheobronchial treee and secondary to either esophageal or pulmonary malignancy.

Traumatic fistulas and those associated with esophageal diverticula account for the remainder.

Fistulas associated with traction diverticula are usually due to mediastinal inflammatory disease, and traumatic fistulas usually occur secondary to penetrating wounds, lye ingestion, or iatrogenic injury.

184
Q

Clinical presentation of esophageal fistulas

A

Paroxysmal coughing following ingestion of liquids
Recurrent or chronic pulmonary infections

Onset of cough immediately after swallowing suggests aspiration, whereas brief delay suggests a fistula.

185
Q

Treatment of esophageal fistulas?

A

Surgical treatment of benign fistulas consists of division of the fistulous tract, resection of irreversibly damaged lung tissue, and closure of the esophageal defect.

To prevent recurrence, a pleural flap should be interposed.

Treatment of malignant fistulas is difficult especially post radiation. Generally, only palliative treatment is indicated, via a specially-designed esophageal endoprosthesis that bridges and occludes the fistula, allowing the patient to eat. A salivary tube is also a good option for proximal esophageal fistulas. This tube has a proximal “lip” that rests on the cricopharyngeal muscle and thereby directs the saliva into the tube and past the fistula.

Last resort (rarely) is esophageal diversion with placement of a feeding jejunostomy.

186
Q

Indications for esophageal resection and substitution?

A

Malignant and end-stage benign disease (reflux/drug-induced stricture formation that cannot be dilated without damage to the esophagus, a dilated and tortuous esophagus from severe motility disorders, lye-induced strictures, and multiple previous antireflux procedures).

187
Q

Difference between a jejunum and stomach/colon graft?

A

The jejunum is a dynamic graft and contributes to bolus transport, whereas the stomach and colon function more as a conduit.

188
Q

A patient well known to your office with a history of chronic reflux and Barrett’s esophagus presents for his scheduled surveillance EGD. Biopsies are taken during the procedure that show evidence of high grade dysplasia. What is the BEST choice in management for this patient?

a. Repeat endoscopy in 6 months
b. Add H2 blocker
c. Schedule a Nissen fundoplication
d. Esophagectomy

A

d. Esophagectomy

189
Q

An 18-year-old female is brought to the emergency department after ingestion of an acidic substance. A chest x-ray does not show evidence of free air or pneumomediastinum. An EGD is performed that shows ulcerations of the esophagus with no perforations. Which of the following is the BEST choice in management of this patient?

a. Observe clinically for 48 hours
b. Repeat EGD in 48 hours
c. Gastrografin swallow study in 48 hours
d. Schedule for an esophagectomy

A

c. Gastrografin swallow study in 48 hours

190
Q

A 58-year-old male comes to the emergency department complaining of substernal chest pain after an episode of vomiting. His blood pressure is 155/85 and his pulse is 88bpm. The patient’s abdomen is soft, but tender at the epigastrium. The patient had a normal EKG and no elevation in troponin levels. What is the BEST next test to determine the diagnosis?

A. Upper GI endoscopy

B. Gastrografin esophagogram

C. CT scan of the chest

D. Echocardiogram

E. Operative exploration

A

B. Gastrografin esophagogram

*Boerhaave’s Syndrome: Violent retching, perforation at distal left wall

191
Q

A 35-year-old female presents with complaints of dysphagia, regurgitation, and weight loss. She has had progressive dysphagia which began wiht liquids and has now progressed to solids. She needs to consume large volumes of water when she eats. She sometimes regurgitates undigested food. All of the following are consistent with her most likely disease process EXCEPT:

A. Esophagogram will show a bird’s beak appearance

B. Manometry will show a hypertensive lower esophageal sphincter (LES) which fails to relax during swallowing

C. There will be no evidence of progressive peristalsis

D. There are high amplitude pressures during manometry

E. EGD will likely show retained food particles in esophagus

A

D. There are high amplitude pressures during manometry

192
Q

A patient well-known to your office with a history of chronic reflux and Barrett’s esophagus presents for his scheduled surveillance EGD. Biopsies are taken during the procedure that show evidence of high-grade dysplasia. What is BEST choice in management for this patient?

A. Repeat endoscopy in 3 months

B. Repeat endoscopy in 6 months

C. Add H2 blocker

D. Schedule a Nissen fundoplication

E. Esophagectomy

A

E. Esophagectomy

193
Q

A 42-year-old female presents with long-standing gastroesophageal reflux disease (GERD). She has had previous endoscopies which have showed esophagitis. The most recent endoscopy showed evidence of Barrett’s esophagus without dysplasia. She has been treated with a PPI daily for 3 years. What is the MOST appropriate next step in management?

A. PPI twice daily

B. Serial endoscopies with random biopsies

C. Observation

D. Nissen fundoplication

E. Esophagectomy

A

D. Nissen fundoplication

194
Q

A 62-year-old male with known esophageal cancer presents to the emergency room complaining of retrosternal chest pain and shortness of breath that started 6 hours prior to arrival. He is febrile and tachycardic, but his blood pressure is normal. On lab workup, he has no elevation in troponins, but does have leukocytosis. A swallow study is performed which shows a free perforation at the mid-esophagus. Which of the following is the BEST choice in management for this patient?

A. Resuscitation in the ICU

B. Surgical repair via a left thoracotomy

C. Primary repair of the perforation with drain placement and a muscle flap.

D. Diverting cervical esophagostomy with chest tube placement

E. Esophagectomy with cervical esophagostomy, gastrostomy and feeding jejunostomy

A

E. Esophagectomy with cervical esophagostomy, gastrostomy and feeding jejunostomy

195
Q

A 63-year-old male presents to the hospital complaining of dysphagia and a 20-pound weight loss over the last 6 months. The patient has an EGD performed that notes a distal esophageal mass with biopsies showing evidence of adenocarcinoma and invasion into the muscularis muscle. Which of the following is the BEST treatment option?

A. Neoadjuvant chemoradiotherapy followed by esophagectomy

B. Esophagectomy alone

C. Esophagectomy with lymph node dissection

D. Esophagectomy with adjuvant chemotherapy

E. Chemoradiotherapy alone

A

B. Esophagectomy alone

196
Q

An 18-year-old female is brought to the emergency department after ingestion of an acidic substance. A chest x-ray does not show evidence of free air or pneumomediastinum. An EGD is performed that shows ulcerations of the esophagus with no perforations. Which of the following is the best choice in management of this patient?

A. Observe clinically for 48 hours

B. Repeat EGD in 48 hours

C. Gastrografin swallow study in 48 hours

D. Stent placement at time of initial EGD

E. Schedule for an esophagectomy

A

C. Gastrografin swallow study in 48 hours

197
Q

A 65-year-old male with past medical history significant for tobacco abuse present with difficulty swallowing. He states his symptoms have increased in severity and he now is having problems swallowing even soft foods. You perform an upper endoscopy and find a tumor obstructing 65% of the circumference of the middle portion of the esophagus. Biopsy reveals an esophageal squamous carcinoma. Which BEST determines his chance for potential cure?

A. Endoscopic Ultrasound (EUS)

B. Positron emission tomography (PET) scan

C. Magnetic resonance imaging (MRI) of the chest and abdomen

D. Computed tomography (CT) of the chest and abdomen

E. Barium swallow study

A

A. Endoscopic Ultrasound (EUS)

198
Q

Name the layers of the esophagus:

A

Mucosa and muscularis propria (the esophagus has no serosa)

199
Q

What is the arterial blood supply to the cervical esophagus?

A

Inferior thyroid arteries (branch of thyrocervical trunk on the left and subclavian artery on the right)

200
Q

What is the arterial supply to the thoracic esophagus?

A

Direct blood supply from 4 to 6 esophageal arteries off of the aorta; esophageal branches off of right (R) and left (L) bronchial arteries; supplemented by descending branches off the inferior thyroid arteries, intercostal arteries, and ascending branches of the paired inferior phrenic arteries

201
Q

What is the arterial blood supply to the abdominal esophagus?

A

Left gastric artery and the paired inferior phrenic arteries

202
Q

Describe the venous drainage for the cervical esophagus:

A

The submucosal venous plexus drains into the inferior thyroid veins (tributaries of L subclavian vein and R brachiocephalic vein)

203
Q

Describe the venous drainage for the thoracic esophagus:

A

The submucosal venous plexus of the thoracic esophagus joins with the more superficial esophageal venous plexus and the venae comitantes that surround the esophagus at this level
This plexus then drains into the azygos veins on the right and the hemiazygos veins on the left.

204
Q

Describe the venous drainage of the abdominal esophagus:

A

Drains into both the systemic and portal venous systems through the Land R phrenic veins and the L gastric (coronary) vein and short gastric veins

205
Q

In what direction is the lymphatic flow in the upper two-thirds of the esophagus?

A

Cephalad

206
Q

In what direction is the lymphatic flow in the distal third of the esophagus?

A

Caudad

207
Q

Describe the sympathetic innervation of the esophagus:

A

Cervical esophagus receives branches from the cervical sympathetic trunk (from superior ganglion in the neck); the thoracic esophagus receives branches from the thoracic sympathetic trunk (from the stellate ganglion). which form an esophageal plexus that envelops the thoracic esophagus anteriorly and posteriorly; the distal thoracic esophagus receives innervation from the greater and lesser splanchnic nerves

208
Q

The parasympathetic fibers to the esophagus arise from which cranial nerve?

A

Vagus nerve

209
Q

The esophagus is composed of which 2 concentric muscle bundles?

A

Inner circular and outer longitudinal

210
Q

What kind of muscle composes the upper one-third of the esophagus?

A

Striated muscle (voluntary)

211
Q

What kind of muscle composes the lower two-thirds of the esophagus?

A

Smooth muscle (involuntary)

212
Q

At what vertebral level does the esophagus enter the diaphragm through the esophageal hiatus?

A

T11

213
Q

What are the areas of anatomic narrowing of the esophagus?

A

Cricopharyngeus muscle; compression by the left mainstem bronchus and aortic arch; diaphragm

214
Q

What is the Z-line?

A

The transition of the distal 1 to 2 cm of esophageal mucosa to cardiac mucosa/junctional columnar epithelium

215
Q

Identification of gastroesophageal junction (external):

A

The collar of Helvetius (loop of Willis) and the gastroesophageal fat pad

216
Q

Identification of gastroesophageal junction (internal):

A

1) The squamocolumnar epithelial junction (Z-line), provided the patient does not have Barrett esophagus
2) The transition from the smooth esophageal lining to the rugal folds of the stomach

217
Q

What 6 events occur during the oropharyngeal phase of swallowing?

A

1) Elevation of the tongue,
2) Posterior movement of the tongue,
3) Elevation of the soft palate,
4) Elevation of the hyoid,
5) Elevation of the larynx,
6) Tilting of the epiglottis

218
Q

What are primary peristaltic contractions of the esophagus?

A

Progressive contractions (2 to 4 cm/s) that move down the esophagus and reach the LES after the initiation of swallowing (-9 seconds)

219
Q

What are secondary peristaltic contractions of the esophagus?

A

Progressive contractions generated from distention/irritation of the esophagus or from an independent local reflex to clear the esophagus of material that was left behind after a primary peristaltic wave rather than voluntary swallowing

220
Q

What are tertiary contractions of the esophagus?

A

Uncoordinated contractions of smooth muscle that are nonprogressive, nonperistaltic, monophasic, or multiphasic simultaneous waves that can occur either after voluntary swallowing or spontaneously between swallows

221
Q

Normal upper esophageal sphincter pressure at rest:

A

50 to 70mmHg

222
Q

Normal upper esophageal sphincter pressure with food bolus:

A

12 to 14 mm Hg

223
Q

Give examples of pulsion (false; mucosa and submucosa herniate through esophageal musculature) diverticula in the esophagus:

A

Zenker diverticulum and epiphrenic diverticulum

224
Q

What esophageal diverticulum is a traction (true) diverticulum that results from external inflammatory mediastinal lymph nodes adhering to the esophagus that heal and contract and pull the esophagus, resulting in the diverticulum?

A

Parabronchial (midesophageal) diverticulum.

225
Q

Best initial test for diagnosis of esophageal diverticula?

A

Barium esophagram

226
Q

On what side are midesophageal diverticula usually found?

A

Typically present on the right secondary to the overabundance of structures in the midthoracic region of the left chest

227
Q

Treatment for a midesophageal diverticula:

A

If a symptomatic patient with inflamed mediastinal lymph nodes from tuberculosis or histoplasmosis, treat medically with anti-tuberculin or antifungal agents;

if diverticulum <2 cm, observe;

if patient symptomatic or diverticulum >2 cm, perform diverticulopexy (suspend from thoracic vertebral fascia). and a long esophagomyotomy is indicated in patients with severe chest pain or dysphagia and a documented motor abnormality

228
Q

Treatment for epiphrenic diverticulum:

A

If diverticula <2 cm, perform diverticulopexy;

in patients with severe chest pain, dysphagia, or a documented motor abnormality, perform long esophagomyotomy:

if diverticulopexy, begin long esophagomyotomy at the neck of diverticulum and extend onto LES;

and if diverticulectomy, perform esophagomyotomy on opposite esophageal wall, extending from the level of the diverticulum onto the LES;

if large associated hiatal hernia, perform diverticulectomy, long esophagomyotomy, and repair of hiatal hernia

229
Q

Killian triangle (site of a Zenker diverticulum):

A

Point of potential weakness at the transition between the oblique fibers of the thyropharyngeus muscle and the horizontal fibers of the cricopharyngeus muscle

230
Q

The surgical treatment of a Zenker diverticulum:

A

If diverticulum > 3 cm, do a cricopharyngeal myotomy with diverticulectomy with TA stapler versus invert and perform diverticulopexy to precervical fascia through a left neck incision

If <2 cm - leave alone

231
Q

Most common site and cause of esophageal perforation:

A

Cricopharyngeus muscle, iatrogenic

232
Q

Management of esophageal perforation:

A

(1) Treatment of contamination with broad-spectrum antibiotics and antifungals.
(2) Wide local drainage.
(3) Source control with covered esophageal stents or VATS.
(4) Enteric feeding access with G or Jtubes.

233
Q

Best initial test for suspected esophageal perforation:

A

Gastrografin esophagram

234
Q

Recommended first test in any patient presenting with esophageal dysphagla:

A

Barium esophagram

235
Q

Indications for a barium esophagram:

A

Regurgitation. globus sensation. dysphagia. GERD, noncardiac chest pain. esophageal neoplasm

236
Q

What characteristics of the esophagus help determine if a lower esophageal sphincter is mechanically defective?

A

Pressure < 6 mm Hg
total length < 2 cm
abdominal length < 1 cm

237
Q

What is the definition of a hypertensive lower esophageal sphincter?

A

LES with a sphincter pressure above the ninety-fifth percentile of normal

238
Q

What are the manometric characteristics of achalasia?

A

Hypertensive LES resting pressure, incomplete or nonrelaxing LES, aperistalsis of the esophageal body, esophageal pressurization, and elevated lower esophageal baseline pressure

239
Q

What is the gold standard for the diagnosis of achalasia?

A

Esophageal manometry

240
Q

What is the presumed pathogenesis of achalasia?

A

Primary destruction of nerves to the LES with secondary degeneration of the neuromuscular function of the body of the esophagus from idiopathic or infectious neurogenic degeneration

241
Q

What is the classic triad of presenting symptoms for achalasia?

A

Dysphagia, regurgitation, weight loss

242
Q

What is the most common esophageal carcinoma identified with achalasia?

A

Squamous cell carcinoma

243
Q

Standard surgical treatment for achalasia:

A

Heller myotomy

244
Q

What are the classic manometry findings with diffuse esophageal spaam?

A

Simultaneous, multipeaked contractions of high amplitude(> 120 mm Hg) or long duration (>2.5 seconds)

245
Q

What is the mainstay of treatment for diffuse esophageal spasm?

A

Medical treatment or endoscopic intervention (nonsurgical)

246
Q

Indications for surgery in a patient with diffuse esophageal spasm:

A

Incapacitating chest pain or dysphagia after failure of medical and endoscopic therapy or presence of a pulsion diverticulum of the thoracic esophagus

247
Q

If indicated, what surgical procedure should be performed in patients with diffuse esophageal spasm?

A

Long esophagomyotomy with the proximal extent high enough to include the entire length of the abnormal motility as determined by manometric measurements and the distal extent of the myotomy down onto the LES, with or without extension onto the stomach through a left tboracotomy or a left video-assisted technique;

Dor fundoplication can be performed to provide protection from reflux and prevent healing of the myotomy site.

248
Q

What is the gold standard for the diagnosis of nutcracker esophagus?

A

Manometry demonstrating high-amplitude peristaltic contractions with normal relaxation of LES

Peristaltic esophageal contractions 2 standard deviations above the normal values on manometric tracings (amplitudes >400 mm Hg)

249
Q

Treatment of nutcracker esophagus:

A

Medical (calcium channel blockers, nitrates, and antispasmodics) for temporary relief during acute spasms; avoidance of caffeine, cold, and hot foods

250
Q

Manometric findings with hypertensive LES:

A

Elevated LES pressure (> 26 mm Hg) and normal relaxation of the LES

251
Q

Treatment of hypertensive LES:

A

Initially endoscopically with Botox injections and hydrostatic balloon dilation; surgery indicated in symptomatic patients who fail interventional treatments

252
Q

If indicated, what is the operation of choice for hypertensive LES?

A

Laparoscopic modified Heller esophagomyotomy with a partial antireflux procedure (Dor, Toupet) in patients with normal esophageal motility

253
Q

What is the manometric definition of ineffective esophageal motility?

A

The sum total of the number of low-amplitude contractions ( <30 mm Hg) and nontransmitted contractions exceeds 30% of wet swallows

254
Q

Best treatment for ineffective esophageal motility:

A

Prevention with effective treatment of GERD; altered motility is irreversible

255
Q

Treatment for a patient with Barrett esophagus with no dysplasia?

A

Acid suppression medication and EGD every 3 to 5 years

256
Q

Treatment for a patient with Barrett esophagus with low-grade dysplasia:

A

Acid suppression medication and surveillance endoscopy performed at 6 to 12 months

257
Q

Recommended treatment for a patient with Barrett esophagus with high-grade dysplasia:

A

Confirmation of pathology by two experienced pathologists.

( 1) Esophagectomy or

(2) endoscopic mucosal resection with radiofrequency ablation or
(3) endoscopic surveillance every 3 months.

Choice of treatment will depend on if patient is a surgical candidate.

258
Q

What is the most common aortic arch anomaly that creates an incomplete vascular ring around the esophagus?

A

Right subclavian artery arising from the descending aorta and traveling behind the esophagus in its course to the right upper extremity

259
Q

What is a pulmonary artery sling?

A

An anomaly of the pulmonary arterial trunk where the left pulmonary artery arises from the right pulmonary artery (instead of the main pulmonary artery trunk) and courses between the trachea and the esophagus with resultant significant anterior compression of the esophagus

260
Q

What can develop with a long-standing, untreated pulmonary artery sling?

A

Tracheal stenosis and left pulmonary artery narrowing

261
Q

Treatment of a pulmonary artery sling:

A

Open sternotomy with cardiopulmonary bypass and anatomic repositioning of the great vessels

262
Q

What are the theories regarding the etiology of a Schatzki ring?

A

A result of reflux esophagitis versus overcontractility of the circular esophageal musculature of the inferior esophageal sphincter combined with the sliding gastric mucosa of a hiatal hernia results in persistent apposition of the 2 mucosal layers and fibrosis of the submucosal layer below

263
Q

How is the diagnosis of a Schatzki ring made?

A

Barium esophagram

264
Q

Treatment for an asymptomatic patient incidentally found to have a Schatzki ring:

A

No treatment

265
Q

Treatment for a patient with a Schatzki ring who presents with acute obstruction:

A

Administration of oral papain (2.5% solution) in 5-mL aliquots every 30 minutes for a total of 4 doses for proteolytic digestion of impacted protein food; intravenous (IV) meperidine (25-50 mg) to encourage spontaneous dislodgment of the impacted food bolus; esophagoscopy with the use of an overtube (rigid or flexible) for extraction

266
Q

Treatment for a patient with a Schatzki ring who presents with dysphagia:

A

Disruption of the ring by oral dilation (SO-French tapered Maloney bougie) with sequential bougienage dilation as symptoms recur

267
Q

Indications for surgery in a patient with a Schatzki ring:

A

Patients who fail bougienage or have intractable reflux, intraoperative bougienage followed by a Nissen fundoplication is recommended, but excision of the ring is not indicated

268
Q

If indicated, what surgical procedure is performed in a patient with a Schatzki ring?

A

Intraoperative bougienage followed by Nissen fundoplication without excision of the ring

269
Q

How is an esophageal web distinguished from a Schatzki ring based on epithelium?

A

Esophageal web has squamous cell epithelium above and below the web.

Schatzki ring is composed of esophageal epithelium above and gastric epithelium below the ring.

270
Q

What is the treatment for a thin esophageal web?

A

Membranous disruption through an endoscope or bougie, versus piecemeal excision with biopsy forceps, versus laser lysis, versus balloon dilation

271
Q

What is the treatment for a thick esophageal web refractory to bougienage?

A

Surgical mucosal resection via a transcervical or transthoracic approach, longitudinal myotomy created and circumferential excision of the web performed, mucosa is circumferentially reapproximated with interrupted absorbable sutures, and muscle is closed longitudinally

272
Q

What is the characteristic appearance of a leiomyoma on barium esophagram?

A

A smooth, well-defined, noncircumferential mass with distinct borders

273
Q

What is the treatment for a leiomyoma?

A

Surgical enucleation; observation acceptable in patients with significant comorbidities or with small ( <2 cm) asymptomatic tumors

274
Q

What is a type I hiatal hernia?

A

Sliding hiatal hernia; simple herniation of GE junction into chest

275
Q

What is a type II hiatal hernia?

A

GE junction remains at esophageal hiatus and gastric fundus herniates alongside the esophagus into the chest

276
Q

What is a type III hiatal hernia?

A

Combination of type I and type II hiatal hernia; GE junction and gastric fundus/body in chest

277
Q

What is a type IV hiatal hernia?

A

Advanced stage of hiatal hernia with entire stomach and other intraabdominal content herniated into the chest

278
Q

What kind of necrosis occurs with a caustic esophageal injury from alkali?

A

Liquefaction necrosis

279
Q

What kind of necrosis occurs with a caustic esophageal injury from acid?

A

Coagulation necrosis

280
Q

What are the 3 phases of tissue injury from alkali ingestion?

A

1) Acute necrosis, ulceration and granulation
2) Cicatrization
3) Scarring

281
Q

What is a grade I caustic injury to the esophagus and the associated endoscopic findings?

A

Superficial mucosal burn, mucosal edema, and hyperemia

282
Q

What is a grade IIA caustic injury to the esophagus, and what are the associated endoscopic findings?

A

Transmucosal injury, patchy ulcerations, exudates, sloughing mucosa

283
Q

What is a grade IIB caustic injury to the esophagus, and what are the associated endoscopic findings?

A

Transmucosal injury, circumferential injury

284
Q

What is a grade III caustic injury to the esophagus, and what are the associated endoscopic findings?

A

Transmural injury with periesophageal/perigastric extension; deep ulcerations; black/gray discoloration; full-thickness necrosis

285
Q

Which grade or grades of caustic injury will progress to stricture?

A

Grades IIB and III

286
Q

Treatment for a patient who presents within the first hour following alkali ingestion:

A

Neutralization with half-strength vinegar or citrus juice

287
Q

Treatment for a patient who presents within the first hour following acid ingestion:

A

Neutralization with milk, egg whites, or antacids

288
Q

Management for a patient with no evidence of burn on endoscopy or physical exam following corrosive ingestion in the acute phase:

A

Observation and oral nutrition when the patient can painlessly swallow saliva

289
Q

Management for a patient with an endoscopically identified first-degree burn following corrosive ingestion in the acute phase:

A

48 hours of observation and oral nutrition when the patient can painlessly swallow saliva; repeat endoscopy and barium esophagram are performed in follow-up at intervals of 1, 2, and 8 months

290
Q

Management for a patient with an endoscopically identified second- or third-degree burn following corrosive ingestion in the acute phase:

A

Monitor in the ICU and keep NPO with IV fluids; start IV antibiotics and a proton pump inhibitor; if evidence of acute airway involvement, airway obstruction can be relieved with aerosolized steroids with the possible need for fiber-optic intubation

291
Q

How can the diagnosis of corrosive injury to the esophagus/stomach be made if not originally secured with endoscopy?

A

Exploratory laparoscopy in stable patients or laparotomy in unstable patients

292
Q

After performing an exploratory laparotomy/ laparoscopy after corrosive injury, a viable stomach and esophagus are encountered; what should be performed?

A

The viable stomach and esophagus are left in situ, and a feeding jejunostomy tube is placed with endoscopic placement of an esophageal stent

293
Q

After performing an exploratory laparotomy/laparoscopy after corrosive injury, a questionable esophagus and stomach are encountered; what should be performed?

A

The questionable esophagus and stomach are left in situ with a second-look operation performed in 36 hours, with further management based on the findings

294
Q

After performing an exploratory laparotomy/laparoscopy after corrosive injury, full-thickness necrosis, or perforation of the esophagus/stomach is found, what should be performed?

A

Resection of the esophagus, stomach, and all affected surrounding organs and tissues; creation of an end-cervical esophagostomy; and placement of a feeding jejunostomy

295
Q

How is a patient who develops an esophageal stricture secondary to corrosive injury managed?

A

After re-epithelialization, aggressive treatment with bougie dilation, regardless of symptoms; dilations performed daily for 2 to 3 weeks, then every other day for 2 to 3 weeks, then weekly for months and lengthening the interval as time passes

296
Q

Name some types of conduits that can be used for esophageal reconstruction:

A

Gastric pull-up, jejunal interposition, colon interposition

297
Q

How much does the presence of Barrett esophagus increase the risk of developing adenocarcinoma?

A

50x

298
Q

What is the most common esophageal cancer?

A

Adenocarcinoma

299
Q

Where does adenocarcinoma of the esophagus usually occur?

A

Lower one-third of the esophagus

300
Q

Where does squamous cell carcinoma of the esophagus usually occur?

A

Upper two-thirds of the esophagus

301
Q

Most common presentation of esophageal cancer:

A

Dysphagia

302
Q

Complications of esophagectomy:

A

Bleeding, airway injury, insufficient conduit length, thoracic duct injury, respiratory (pneumonia), esophageal conduit necrosis, anastomotic leak, anastomotic stricture, hoarseness, chylothorax, transhiatal herniation of abdominal contents

303
Q

What are the characteristic findings of a chylous effusion?

A
Turbid, milky white fluid; 
specific gravity 1.020 to 1.030; 
protein content 3 to 4 g/100 mL; 
fat content 1 to 4 g/ 100 mL; 
triglyceride content > 110 mg/dL
304
Q

What is the reported incidence of chylothorax as a complication of esophageal surgery?

A

-4%

305
Q

Management for a chylous effusion with <500 cc/24 h chest tube output:

A

Nonoperative management (should stop spontaneously)

306
Q

Management for a chylous effusion with >500 cc but < 1000cc/24 h chest tube output:

A

Initially, nonoperative management; if not resolved or significantly improved in 5 to 7 days, perform operative duct ligation

307
Q

Management for a chylous effusion with > 1000 cc/24 h chest tube output:

A

Early duct ligation

308
Q

The most common cause of chylothorax is:

A. Trauma to the spine and chest wall

B. Lymphatic obstruction from mediastinal lymphoma

C. Lymphatic leakage after mediastinal lymph node dissection

D. Thoracic duct injury during esophagectomy

A

Answer: D.

The thoracic duct ascends in the mediastinum parallel to the esophagus and can easily be injured during mobilization for esophagectomy. Lymphatic leaks from lymph node dissections are often trivial and will usually seal on their own with no intervention.

309
Q

A 34-year-old male patient has been experiencing gastric reflux symptoms and recently underwent a workup, which included an upper endoscopy. The upper endoscopy revealed Barrett esophagus. The most appropriate therapy for this patient is:

A. Esophagectomy

B. Repeat endoscopy in 6 months

C. Medical therapy with acid suppression

D. Medical therapy with acid suppression and referral for fundoplication

A

Answer: D

In a patient with reflux symptoms and Barrett epithelium, this is an indication for antireflux surgery, that is, Nissen fundoplication.

310
Q

What is the histologic subtype of Barrett tissue?

A. Junctional epithelium

B. Specialized columnar epithelium

C. Gastric fundus epithelium

D. Specialized squamous epithelium

A

Answer: B.

Barrett esophagus is an acquired condition thought to be caused by chronic reflux. It is associated with ulceration and stricture and has a higher rate of malignant degeneration than normal esophageal endothelium.

311
Q

After a 3-hole esophagectomy with a cervical anastomosis, the patient begins to develop leukocytosis, fever, and erythema of the wound. A barium swallow is performed and an anastomotic leak is noted. What is the most appropriate management of this patient?

A. Open the wound and redo the anastomosis

B. Open the wound and widely drain the anastomosis

C. Endoluminal stent

D. IV antibiotics and NPO

A

Answer: B.

Patients with anastomotic leaks at the cervical area need to be made NPO; given antibiotics; and have the neck wound explored, debrided, and drained.

Revising the anastomosis is unlikely to be successful due to inadequate length and friability of the tissues.

In cases of catastrophic conduit necrosis, the esophagus can be diverted proximally and reconstruction performed in a delayed fashion with alternative conduits (jejunum, colon)

312
Q

Boerhaave esophageal rupture normally occurs at:

A. Cervical esophagus

B. Proximal thoracic esophagus

C. Middle thoracic esophagus

D. Distal thoracic esophagus

A

Answer: D.

Boerhaave ruptures normally occur in this area after periods of coughing or retching. The patient will normally present with severe chest pain, fever, and subcutaneous emphysema.

313
Q

An elderly man was experiencing progressive dysphagia with regurgitation of undigested food. A thorough workup revealed he had a cervical Zenker diverticulum. What is the treatment of choice for this condition?

A. Observation

B. Ligation

C. Diverticulectomy

D. Diverticulectomy with cricopharyngeal myotomy

E. Esophagectomy

A

Answer: D.

Symptoms may vary, but this is largely a progressive disease.

Smaller diverticuli may be left in situ; however, the key aspect to the procedure is to release the cricopharyngeal muscle by performing an adequate myotomy.

The patient is at a higher risk for recurrence if an appropriate myotomy is not made.

314
Q

A 32-year-old man is brought to the emergency room after ingesting some drain cleaner in a suicide attempt. He is hemodynamically stable, and a referral is placed for surgical evaluation. A CT scan has already been performed, and there is no evidence of perforation. What is the next step in management?

A. Observation

B. EGD

C. Barium upper GI swallow

D. Immediate laparotomy for exploration

E. Immediate thoracotomy for exploration

A

Answer: B.

Caustic injuries can cause catastrophic damage to the esophagus; aggressive management is necessary. An EGD should be the first invasive test to assess the degree of damage to the esophageal mucosa and needs to be serially repeated throughout the healing process. There is no role for exploratory surgery in the absence of perforation or sepsis.

315
Q

A 48-year-old woman who is morbidly obese is referred to your office with an incidental CT finding of an aberrant right subclavian artery running in a retroesophageal path. What is the management strategy of choice?

A. Reimplantation or bypass of the subclavian artery

B. Upper GI swallow study

C. EGD

D. Observation

E. Gastric bypass

A

Answer: D.

An aberrant right subclavian artery is a common anomaly. Normally, it doesn’t cause any symptoms and the patient can be reassured.

In a patient with no symptoms of dysphagia and concomitant morbid obesity, there is little doubt that food intake is a problem and no specific treatment is necessary.

In patients who present with dysphagia and weight loss, this can be termed “dysphagia lusoria” and consideration is made for reimplantation or bypass of the subclavian artery to provide relief of the esophageal obstruction.

316
Q

A 50-year-old man comes in complaining of progressive dysphagia. CT scan shows no evidence of tumor. An upper GI study shows no tumor or functional obstruction. An EGD is performed, and a small tear is iatrogenically created with the EGD scope. The patient has some mild chest pain and is started on antibiotics. A stat CT scan shows a small, contained perforation just above the GE junction. What is the treatment of choice?

A. Immediate laparotomy and primary repair

B. NPO, antibiotics, and observation

C. Esophageal stenting

D. Endoluminal repair with clips

A

Answer: B.

A patient with a contained esophageal perforation may be treated conservatively, provided there are no systemic signs of sepsis or peritonitis. These patients often heal within 2 weeks.

Surgery should be reserved for patients who develop peritoneal signs, a leukocytosis, or an uncontained perforation.

Esophageal stenting is a novel approach, but it would be difficult to place at the GE junction to create a good seal without creating significant reflux.

The role of endoluminal repair is evolving and is not currently a standard approach.

317
Q

A patient recently underwent an Ivor-Lewis esophagectomy for cancer. The initial operation was straightforward and took 4 hours. On postoperative day 7, the patient began to develop fever, tachycardia, and shortness of breath. He developed hypotension and was refractory to IV fluid boluses and is now on vasopressors to maintain an adequate perfusion pressure. A CT demonstrated a large right-sided pleural effusion and extravasation of the oral contrast material. The patient was brought back for re-exploration, and the anastomosis was disrupted. In addition, the gastric conduit was dusky and there was concern about its viability. What is the best treatment option at this point?

A. Staple off the necrotic stomach; perform a diverting esophagostomy

B. Resect the necrotic stomach and redo the anastomosis

C. Place an endoluminal stent across the anastomosis and allow it to granulate

D. Resect the gastric conduit and perform jejunal interposition bypass

E. Wide drainage with chest tubes and long-term antibiotics with total parenteral nutrition support

A

Answer: A.

In a septic patient with conduit necrosis, the best choice is a damage control-type procedure: resect the obviously necrotic gastric conduit and perform an esophageal diversion (Spit fistula).

Presumably, the patient already has a Jtube for enteral feeds from the first operation. This will allow for resuscitation and stabilization, and delayed reconstruction can be considered with alternative conduits.

There is no role for a complex operation in a septic patient with a failed bypass.

318
Q

A 60-year-old patient presents with dysphagia and a 20-year history of GERD.

On endoscopy you find a concentric ring of tissue at the GE junction. Proximal and distal biopsies reveal stratified squamous and simple columnar epithelium, respectively. What is the treatment of choice?

A. Botox injection

B. Esophagectomy

C. Dilation

D. Medical management

E. Endoscopic mucosal resection

A

Answer: C.

Schatzki ring is a fibrous thickening in the esophagus that causes narrowing. It is nonmalignant in nature. Biopsies reveal stratified squamous cells above the ring and columnar cells distally. The treatment of choice is esophageal dilation.

319
Q

A 70-year-old female presents with complaints of difficulty swallowing, sensation of food stuck in her throat, halitosis, and multiple bouts of pneumonia over the past year. What is the best initial test in diagnosis?

A. CT of the neck and chest

B. EGD

C. Esophagram

D. pH monitoring

E. Manometry

A

Answer: C.

The symptoms he presents with are consistent with a Zenker diverticulum, especially given his history of recurrent pneumonia. The best initial test for diagnosis of a suspected Zenker diverticulum is a barium esophagogram

320
Q

Eight months after undergoing an Ivor-Lewis esophagectomy, a 54-year-old male presents complaining of dysphagia to solids after he swallows. He is still able to tolerate liquids; however, he is having trouble maintaining his weight. EGD does not show any signs of mallgnancy. What is the initial treatment of choice?

A. Radiation therapy

B. Revision of the anastomosis with right VATS

C. Revision of the anastomosis with right posterolateral thoracotomy

D. Encourage patient to maintain a clear liquid diet

E. Esophageal dilation

A

Answer: C.?

Stricture is a known late complication of esophagectomy. If patients are symptomatic with dysphagia to solids and having issues maintaining weight, the treatment of choice is esophageal dilation.

321
Q

A 45-year-old female presents with medical history significant for smoking and GERD that has been managed with medical therapy. She undergoes an EGD, which demonstrates Barrett esophagus with low-grade dysplasia. What is the next best step?

A. EGD in 6 to 12 months

B. EGD in 3 to 5 years

C. Endoscopic mucosa! resection

D. Ivor-Lewis esophagectomy

A

Answer: A.

Patients found to have Barrett esophagus with low-grade dysplasia should undergo screening EGD in 6 to 12 months with 4-quadrant biopsy every 1 cm in concerning areas.

B is appropriate for Barrett esophagus without dysplasia.

C is appropriate for Barrett esophagus with high-grade dysplasia.

D is appropriate for esophageal carcinoma.

322
Q

A 37-year-old female presents with a 1-year history of progressive dysphagia. She was diagnosed with GERD 8 years ago and takes omeprazole daily, which does not relieve her chest pain. Barium esophagram reveals a dilated esophagus that tapers down to a narrow segment just proximal to the stomach. What results would you expect to find on esophageal manometry?

A. Hypertensive LES resting pressure, aperistalsis of the esophageal body

B. Hypotensive LES resting pressure, aperistalsis of the distal esophagus

C. High-amplitude peristaltic contractions with normal LES relaxation

D. Simultaneous, multipeaked contractions of high amplitude or long duration

A

Answer: A.

Based on her esophagram findings, she has achalasia. Achalasia is associated with a hypertensive LES with a diffuse dilation of the intrathoracic esophagus that is aperistaltic.

B is a manometry finding associated with scleroderma.

C is associated with nutcracker esophagus.

D is associated with diffuse esophageal spasm.

323
Q

Which of the following is true regarding the anatomy of the esophagus?

A. The narrowest point of the esophagus is at the level of the bronchoaortic constriction.

B. The Meissner’s plexus is located in the submucosa.

C. The Auerbach’s plexus is located between the longitudinal muscle and the adventitia.

D. The serosa is the strongest layer of the esophagus.

E. The outer longitudinal layer is an extension of the cricopharyngeus muscle.

A

ANSWER: B

COMMENTS: The esophagus is a two-layered muscular tube, and it is approximately 25 to 30 cm in length.

The esophagus is unique from the other parts of the alimentary tract in its lack of a serosal layer.

The inner circular muscle is an extension of the cricopharyngeus muscle.

Two nerve plexuses, the Meissner’s and Auerbach’s plexuses, are found in the submucosa and between the muscle layers of the esophagus, respectively.

They comprise the intrinsic autonomic nerve system of the esophagus and are responsible for peristalsis.

Three distinct anatomic constrictions of the esophagus occur at the level of the cricopharyngeus muscle (approximately 14 mm), left mainstem bronchus (15 to 17 mm), and the diaphragmatic hiatus (16 to 19 mm), in order of increasing diameter.

324
Q

Which of the following is true of the esophageal sphincters?

A. The upper esophageal sphincter (UES) is mainly composed of the inferior constrictor muscle.

B. The mean resting pressure of the UES is approximately 20 to 30 mmHg.

C. The lower esophageal sphincter (LES) is approximately 2 to 5 cm in length.

D. The LES can be identified by an area of hypertrophic muscle.

E. The LES resting pressure is between 6 and 26 mmHg and can be overcome by normal peristalsis.

A

ANSWER: C

COMMENTS: The upper and LESs are high pressure zones rather than actual anatomic landmarks.

The cricopharyngeus muscle is thought to be the main contributor to the upper high-pressure zone.

When swallowing, UES pressure can reach 90mmHg and return to an average resting pressure of 60 mmHg.

The LES is characterized by a resting pressure zone of approximately 6 to 26 mmHg and measures 2 to 5 cm in length.

Vagus-mediated relaxation of the LES occurs during normal food transit.

Gastrin and motilin increase LES pressure, whereas cholecystokinin and secretin decrease LES pressure.

325
Q

The blood supply to the cervical esophagus arises from the:

A. Bronchial arteries

B. Direct branches from the aorta

C. Inferior thyroid arteries

D. Left gastric artery

E. Short gastric arteries

A

ANSWER: C

COMMENTS: The blood supply to the cervical esophagus arises from the inferior thyroid arteries, a branch from the thyrocervical trunk of the right subclavian artery.

The proximal thoracic esophagus receives its blood supply from the bronchial arteries.

The distal thoracic esophagus receives its blood supply from direct branches arising from the aorta.

The intraabdominal esophagus receives its blood supply from the inferior phrenic arteries.

The short gastric arteries cross the gastrosplenic ligament arising from the end of the splenic artery and anastomosing with the branches of the left gastric and left gastroepiploic arteries.

326
Q

Esophageal contractions are coordinated in the:

A. Midbrain

B. Pons

C. Hypothalamus

D. Medulla

E. Cerebellum

A

ANSWER: D

COMMENTS: Esophageal contractions are controlled by the swallowing center located in the medulla.

Swallowing is a complex series of coordinated events, which can be broken down into the oral, pharyngeal, and esophageal phases.

The oral phase is voluntary and controlled by the medial temporal lobes and limbic system of the cerebral cortex.

When a food bolus is passed to the pharyngeal phase, it becomes involuntary. This phase involves the closure of the larynx, protecting the airway, via elevation of the hyoid among other coordinated events.

The food bolus eventually passes through the UES to begin the esophageal phase.

Esophageal peristalsis is separated into primary, secondary, and tertiary peristalsis as delineated on manometry.

Primary peristalsis is triggered voluntarily after swallowing but becomes involuntary after initiation. Waves travel the entire length of the esophagus and work to propel the food bolus toward the stomach.

Secondary peristalsis is involuntary and triggered by esophageal distension or irritation. They are thought to have defensive purposes working to propel reflux contents back toward the stomach.

Tertiary peristalsis involves uncoordinated waves that are nonprogressive and nonperistaltic, representing smooth muscle contractions and related to motility disorders and spasm.

327
Q

Lifestyle modifications are thought to help with mild symptoms of gastroesophageal reflux disease (GERD). Cigarette smoking is thought to contribute to GERD by:

A. Decreasing LES pressure and impairing contractility

B. Increasing acid production

C. Decreasing the esophageal clearance of acid

D. Increasing secondary peristalsis

E. Increasing saliva production

A

ANSWER: A

COMMENTS: A number of lifestyle factors have been found to be associated with GERD, including acidic liquids and foods, smoking, drinking, obesity, supine sleeping, and certain medications.

Although modification of these associated risk factors is plausible and has proven benefits in symptoms of GERD patients, they are limited and often provide inconsistent relief.

Smoking has been proven to cause decreased pressure of the LES on high resolution manometry immediately after smoking; the pressure returns to baseline several minutes after completion.

Cigarettes also lead to decreased saliva production and decreased contractility, leading to prolonged acid secretion, and impair the pharyngeal/UES reflex during deglutition.

328
Q

Which of the following is the least important when performing a Nissen fundoplication for reflux disease?

A. Use of pledgets to prevent suture tears

B. Lengthening the intraabdominal esophagus

C. Division of the short gastric vessels

D. Hiatal dissection and closure

E. Short and floppy fundoplication around the esophagus with a bougie

A

ANSWER: A

COMMENTS: The principles of antireflux surgery (ARS) that have been studied and accepted are hiatal dissection and closure, lengthening of the intraabdominal esophagus, division of the short gastric vessels, creation of a short (2 cm) and floppy fundoplication, and the use of a bougie.

Common techniques often used by many surgeons but not well established are fixation of wrap, use of pledgets, bougie size, and number of sutures used.

329
Q

Which of the following findings is a contraindication to ARS?

A. Presence of severe esophagitis on endoscopy

B. A DeMeester score of 55

C. Type III hiatal hernia seen on an esophagogram

D. Barrett’s esophagus with high-grade dysplasia

E. A shortened esophagus

A

ANSWER: D

COMMENTS: The indications for ARS are severe esophageal injury, incomplete resolution of symptoms with medical therapy, patient preference against long-term pharmacologic therapy, or complications from a hiatal hernia.

The success of ARS depends on the accuracy of diagnosing GERD, which can be enhanced by monitoring pH.

The DeMeester score is used to assess the degree of abnormality (>14) of the pH study.

The presence of a short esophagus requires a lengthening procedure such as a Collis gastroplasty in addition to fundoplication, but it is not a contraindication to ARS.

The presence of high-grade dysplasia within a Barrett’s esophagus requires resection and is a contraindication to ARS.

330
Q

Seven years after her initial ARS, a patient undergoes a reoperation for recurrence of symptoms. During the reoperation, what is the most likely finding?

A. Disrupted wrap

B. Loose wrap

C. Herniated wrap

D. Slipped wrap

E. Stricture

A

ANSWER: C

COMMENTS: The long-term success rate of ARS approaches 90% with approximately a 1%-per-year failure rate.

The most common operative finding on repeated fundoplication is a herniated fundoplication (33%) above the diaphragm, followed by a disrupted wrap (18%), a tight wrap (13%), and a slipped wrap (10%) onto the body of the stomach.

331
Q

Several endoscopic options exist as alternatives to surgical ARS. Transoral incisionless fundoplication (TIF) is an endoscopic wrap. Which of the following is true regarding TIF?

A. Approved for use with hiatal hernias larger than 2 cm

B. Performs a complete 360-degree wrap

C. Provides good symptom relief in the majority of patients for up to 1 year

D. Requires radiofrequency ablation to bulk LES with fibrosis

E. Can perform a cruroplasty at the same time

A

ANSWER: C

COMMENTS: TIF is a totally endoscopic procedure for performing a partial fundoplication using T-bar fasteners.

This wrap is usually performed as a 270-degree to 290-degree wrap. It is not approved as a modality to treat hiatal hernias >2 cm but is approved to reduce hernias <2 cm.

This is performed using a suction traction device to pull the stomach downward.

Several studies with 12-month follow-up data have reported more than 90% of patients remaining off proton pump inhibitors (PPIs). No trial has compared TIF to surgical ARS, and longer-term follow-up studies are required.

ARS remains an option after TIF if the wrap has failed.

Radiofrequency ablation is a therapeutic modality utilized in the treatment of Barrett’s esophagus with high-grade dysplasia and is an endoscopic treatment option (Stretta).

This technique is thought to bulk the LES with fibrotic reaction changes thus decreasing reflux.

TIF does not provide a cruroplasty as part of the repair of small hiatal hernias.

332
Q

Which of the following is true regarding hiatal hernia repair?

A. A 5- to 6-cm wrap is recommended.

B. Mobilization of intrathoracic esophagus is required to the level of the aortic arch.

C. Permanent synthetic mesh is preferred for large crural defects.

D. Iatrogenic pneumothorax during laparoscopic repair often requires tube thoracostomy.

E. Shortened esophagus can be lengthened with a Collis gastroplasty.

A

ANSWER: E

COMMENTS: A 2-cm floppy wrap is recommended, providing adequate control of reflux.

Longer wraps may control reflux better but have been associated with increased symptoms of dysphagia.

The intrathoracic esophagus is mobilized to lengthen the intraabdominal portion in an attempt to relocate the LES at its physiologic level.

This typically requires at least 2 to 3 cm of mobilization.

Pneumoperitoneum during laparoscopic dissection can artificially make this length appear longer intraoperatively.

Use of mesh to buttress hiatal hernia repairs is typically used in large defects, defined as >5 cm, or when crural dissection has left much of the peritoneum stripped.

There is no consensus on the use of mesh; however, the use of permanent mesh has been largely abandoned due to problems with erosion.

Often, dissection of large hernias results in the entering of one or both of the pleural spaces.

Carbon dioxide is rapidly resorbed and does not require repair or tube thoracostomy. Shortened esophagus is relatively uncommon in smaller hiatal hernias with a higher incidence in large hiatal hernias, peptic strictures, and reoperative ARS.

Insufficient mobilization of intraabdominal esophagus can result in high rates of recurrence, up to 40% in large and giant hernias.

Creation of a tension-free wrap then requires lengthening of the esophagus via Collis gastroplasty.

This involves excising a portion of the cardia, creating a “neo-esophagus,” and thus moving the gastroesophageal junction (GEJ) and angle of His into the abdominal cavity.

333
Q

A 53-year-old male has typical symptoms of GERD refractory to medical management and is referred to your clinic to discuss surgical options. He has previously had a distal pancreatectomy/ splenectomy from a gunshot wound and three subsequent laparotomies for adhesive bowel obstruction. What surgical option allows for satisfactory control of reflux while avoiding the abdomen?

A. Toupet 
B. Hill
C. Dor
D. Belsey Mark-IV
E. Gastropexy
A

ANSWER: D

COMMENTS: The Belsey Mark-IV operation recreates the LES while reducing an intrathoracic stomach and closing a hiatal defect.

This operation is approached via a left thoracotomy, allowing the surgeon to avoid a hostile abdomen and is typically reserved for redo situations in the current era of laparoscopic Nissen fundoplication.

This approach can also be useful in the dissection of large and giant chronic paraesophageal hernias.

The Belsey Mark-IV provides a 240-degree partial wrap, imbricating cardia against the distal esophagus. Each of the other operations requires an abdominal approach.

The Toupet is a 270-degree, posterior partial wrap, while the Dor is a 180-degree anterior partial wrap.

The Hill operation focuses on the creation of a gastroesophageal flap valve in an attempt to recreate normal anatomy.

Gastropexy is reserved for the highest-risk patients with hernias at risk for volvulus and can include a percutaneous endoscopic gastrostomy tube.

This, however, should not be considered in patients with significant abdominal surgical histories.

334
Q

A 55-year-old man is evaluated for dysphagia and chest pain. A barium esophagogram shows a 3-cm smooth filling defect in the distal end of the esophagus. Which of the following is true of his condition?

A. Cystic transformation or central necrosis is often associated with these lesions.

B. Patients often have hematemesis or chronic anemia because of ulceration.

C. Endoscopic ultrasound (EUS) will show a hypoechoic mass in the submucosa.

D. Endoscopic biopsy should be performed to rule out malignancy.

E. Esophagectomy is recommended for lesions larger than 2 cm.

A

ANSWER: C

COMMENTS: Benign tumors of the esophagus are rare and represent less than 1% of esophageal neoplasms.

Leiomyomas account for 60% of these lesions and are often found in the distal two-thirds of the esophagus.

Most of these tumors are asymptomatic. Pain and dysphagia are the most common complaints.

They have a characteristic smooth filling defect on contrast-enhanced study and are described as a hypoechoic mass within the submucosa or muscularis propria on EUS.

Recently, they have been classified as a gastrointestinal stromal tumor.

Most of these tumors occur from mutations of the c-KIT oncogene.

Leiomyomas are removed by enucleation, and biopsy should be avoided because of the increased risk for perforation.

335
Q

Which of the following most likely contributes to GERD?

A. Intraabdominal LES length of 3 cm

B. LES resting pressure of 12 mmHg

C. Thirty percent tertiary waveforms

D. Total LES length of 5 cm

E. Attachment of the phrenoesophageal ligament 4 cm above the GEJ

A

ANSWER: C

COMMENTS: Factors that contribute to the failure of the intrinsic antireflux mechanism are intraabdominal LES length less than 1 cm, LES resting pressure less than 6 mmHg, the presence of esophageal dysmotility, LES total length less than 2 cm, and a low attachment of the phrenoesophageal ligament

336
Q

A 28-year-old male presents with symptoms of dysphagia to solids and liquids with regurgitation. Esophogography shows distal narrowing to a point. What is the most likely finding on high-resolution manometry?

A. Aperistalsis without relaxation of the LES with deglutition

B. Uncoordinated peristalsis

C. Esophageal pressure of 250 mmHg

D. Contraction wave of 9 s

E. LES pressure 20 mmHg

A

ANSWER: A

COMMENTS: This patient has achalasia, an esophageal motility disorder affecting 1 in 10,000. Its exact etiology is unknown, and it results from an uncontrolled balance of excitatory and inhibitory signals.

Symptoms may be vague but typically involve dysphagia, regurgitation, and weight loss.

Its diagnosis can be made by a characteristic distal narrowing on esophagogram, creating the classic “bird’s beak” appearance.

Manometry is required to confirm the diagnosis and typically displays aperistalsis with a hypertensive resting LES that fails to relax with deglutition (Table 20.1 and Fig. 20.1).

High-amplitude waves, pressure, and prolonged contraction waves are characteristic of Nutcracker esophagus.

These waves result in typically complete propagation and in LES relaxation. Uncoordinated peristalsis is present in diffuse esophageal spasm (DES).

The normal resting pressure of the LES is between 15 and 25 mmHg.

337
Q

The above patient decides to undergo surgical therapy for his diagnosis. Which of the following is true regarding the surgical management of achalasia?

A. Nonsurgical therapy results in durable relief of dysphagia.

B. Myotomy should extend at least 5 cm cephalad and 2 cm caudad onto the stomach.

C. Addition of antireflux component to operation is not required.

D. Accidental esophagostomy can be repaired primarily with continued use of the same myotomy site.

E. Only the longitudinal muscle fibers are incised.

A

ANSWER: B

COMMENTS: Heller myotomy remains the most common surgical treatment for achalasia.

This can be performed open or laparoscopically and begins with hilar dissection and mobilization of intrathoracic esophagus for visualization.

A myotomy is performed to release the LES to provide symptomatic relief and typically begins just proximal to the GEJ anteriorly and extends a minimum of 5 cm cephalad and 2 to 3 cm caudad.

It is important that both longitudinal (outer) and circular (inner) muscle layers be released.

An antireflux component is routinely added as postoperative reflux can be disabling after myotomy.

Typically, a partial wrap, either the Dor or Toupet, is used because of fear of recurrent dysphagia with complete Nissen wrap, although this has been successfully utilized.

No wrap has been proven to be superior.

Entrance into the esophageal lumen requires repair with the creation of a new myotomy site.

Nonsurgical options include Botox injections and pneumatic dilation.

Botox injections provide good relief but lack durability with recurrent symptoms typically occurring 1 to 4 months after injection. Additionally, Botox can significantly increase the difficulty of surgical myotomy and should only be reserved for nonsurgical candidates.

Pneumatic dilation provides good relief of dysphagia but lacks durability, often requiring repeat dilations with up to half of the patients at 5 years having recurrent symptoms. The most devastating complication of dilation is esophageal perforation, and it occurs in up to 4% of patients.

Peroral endoscopic myotomy, POEM, is an interesting endoscopic approach to performing a surgical myotomy and has been shown to have good results.

338
Q

A patient arrives at the emergency department 8h after balloon dilation of her esophagus with complaints of dysphagia and chest pain. She was found to be febrile, tachycardic, and normotensive. Esophagography showed “bird’s beak” narrowing and a leak at the distal end of the esophagus with contrast material in the left side of the chest. After fluid resuscitation and antibiotics, which of the following is the most appropriate management?

A. Nasogastric tube decompression and observation

B. Endoscopic evaluation of the injury and stenting

C. Left thoracotomy, primary repair, myotomy, and drain placement

D. Laparotomy, primary repair, and gastrostomy tube placement

E. Laparotomy, esophagectomy, and cervical esophagogastrostomy

A

ANSWER: C

COMMENTS: The rate of esophageal perforation after endoscopic pneumatic dilation is low (4%).

Early diagnosis plus treatment of esophageal perforation is associated with improved survival.

In stable patients with a contained perforation, there is a role for nonoperable management consisting of nothing by mouth and intravenous antibiotics.

If the perforation is because of an underlying pathology that causes distal obstruction (e.g., achalasia, esophageal cancer, or stricture), the operative treatment must address the underlying disease.

A myotomy should be performed in patients with achalasia and esophagectomy considered only in those with a sigmoid esophagus or megaesophagus.

339
Q

A 42-year-old female presents with symptoms of dysphagia. Thus far, a workup has included a normal esophagogastroduodenoscopy (EGD), esophagogram, and manometry. She is frustrated with the lack of answers and is referred to your clinic by her gastroenterologist. You order a computed tomography (CT) of chest, which helps diagnose her with dysphagia lusoria. What findings would be present on the CT scan?

A. Left pulmonary artery arising from the right pulmonary artery

B. Right subclavian artery arising from the descending thoracic aorta

C. Double aortic arch

D. Left common carotid arising from the brachiocephalic artery

E. Late, posterior takeoff of the brachiocephalic artery

A

ANSWER: B

COMMENTS: Vascular rings are a known cause of tracheoesophageal compressive symptoms.

They typically present the children as diet is transitioned from milk to solids but can vary widely in presentation.

A left pulmonary artery arising from the right pulmonary artery and a double aortic arch may form a sling around the esophagus and cause symptoms of dysphagia but typically present at a younger age.

The left common carotid arising from the brachiocephalic artery is a normal variant of arch anatomy; it is asymptomatic and referred to as a bovine arch.

An aberrant right subclavian artery that arises from the descending aorta and travels posterior to the esophagus can present as dysphagia in the adult.

Surgical correction is required in the symptomatic patient and involves ligation at its origin and reimplantation on the right common carotid artery.

340
Q

A 40-year-old woman complains of chest pain and dysphagia. Manometric studies show simultaneous multipeaked contractions of 140 mmHg, lasting 4 to 5 s, and normal LES relaxation. Which of the following is true of her disease?

A. Esophagography will show a “corkscrew esophagus.”

B. It can be caused by infection with Trypanosoma cruzi.

C. It is the result of fibrous replacement of the esophageal smooth muscle.

D. It is also known as “vigorous” achalasia.

E. Bougie dilation is the first-line treatment.

A

ANSWER: A

COMMENTS: DES is a poorly understood motility disorder of the esophagus.

Chest pain and dysphagia are often present.

The diagnosis is made by esophagography demonstrating a classic picture of a corkscrew esophagus (Fig. 20.2).

Manometry will show simultaneous multipeaked contractions similar to those seen in achalasia; however, the LES will have normal receptive relaxation.

A variant of achalasia in which amplitude pressure is normal or elevated is also known as vigorous achalasia.

Pharmacotherapy (nitrates, calcium channel blockers, and phosphodiesterase inhibitors) aimed at smooth muscle relaxation is the first-line treatment of DES.

Bougie or pneumatic dilations are used with variable results for severe dysphagia with documented LES hypertension.

Surgery, which involves a long esophagomyotomy from the level of the aortic arch to the LES, is reserved for patients who fail pharmacologic and endoscopic therapies.

341
Q

A 68-year-old male presents with symptoms of dysphagia and halitosis. An esophagogram displays outpouching at the cricothyroid on lateral view. Which of the following is true regarding his condition?

A. It is a true diverticulum.

B. Endoscopic esophagodiverticulostomy is best for small, <2 cm, diverticula.

C. Diverticulectomy alone is sufficient.

D. It is caused by a traction mechanism.

E. Diverticulopexy and myotomy are preferred approaches.

A

ANSWER: E

COMMENTS: Esophageal diverticula may be classified by type (true or false), location, and pathophysiology (traction or pulsion).

Parabronchial diverticula are the only true diverticula and result from traction associated with inflamed mediastinal lymph nodes.

Epiphrenic diverticula are false resulting from pulsion and occur in the distal esophagus.

Zenkers, or pharyngoesophageal, diverticula occur in Killian’s triangle at the level of the cricopharyngeus and are thought to be the result of increased pressure and pulsion during swallowing.

Symptoms involve dysphagia, halitosis, and regurgitation. Symptomatic Zenkers requires surgical therapy.

Diverticulopexy and myotomy remain the preferred approaches, minimizing potential salivary fistulas that diverticulectomy predisposes.

For small diverticula, myotomy alone has proven sufficient.

Videoendoscopic stapling via esophagodiverticulostomy combines the myotomy and enlarges the pouch but is contraindicated in diverticula <3 cm.

This is because there is insufficient room for placement of the stapler jaws.

342
Q

A 65-year-old man with a 10-year history of heartburn undergoes an endoscopy with a distal esophageal biopsy, which shows intestinal columnar metaplasia. Which of the following is true of his condition?

A. The metaplastic cells are more prone to reflux injury than the squamous epithelium.

B. The condition is found in 50% of patients with GERD.

C. Helicobacter pylori is associated with the condition.

D. More than 70% of cases are found in men in their fifth and sixth decades.

E. The condition is associated with a fivefold increase in the risk for adenocarcinoma.

A

ANSWER: D

COMMENTS: Esophageal mucosal injuries result from reflux of gastric juice that may contain bile salts from the duodenum.

Within a pH range of 2 to 6.5, bile salts are soluble and nonionized; they are therefore better absorbed by esophageal mucosa cells and cause the greatest cell damage.

Barrett’s esophagus is a condition in which intestinal columnar epithelium replaces the esophageal squamous epithelium as a result of inflammation secondary to chronic reflux.

The metaplastic cells are more resistant to injury from reflux but are more prone to malignant transformation.

Barrett’s esophagus is found in 10% of patients with GERD, and more than 70% of cases are found in men aged 55 to 63 years.

Patients with Barrett’s esophagus have a 40-fold increased risk for esophageal carcinoma.

343
Q

The patient in the above question is found to have high-grade dysplasia on follow-up surveillance. Which of the following is true regarding Barrett’s esophagus with high-grade dysplasia?

A. Surveillance endoscopy every 6 months is acceptable.

B. Photodynamic therapy (PDT) is superior to radiofrequency ablation.

C. Surveillance protocol dictates four-quadrant biopsies at the z-line.

D. ARS is indicated.

E. His risk of progression to adenocarcinoma is 6% per year.

A

ANSWER: E

COMMENTS: The treatment of Barrett’s esophagus varies depending on the presence or absence of dysplasia and whether it is low or high grade.

The differentiation between low-grade dysplasia (LGD) and high-grade dysplasia (HGD) is purely histologic and subjective with reported interobserver variability (LGD < 50% agreement vs. HGD 85% agreement).

Barrett’s without dysplasia may be observed every 3 to 5 years via endoscopy with biopsies.

Treatment consists of managing the acidic reflux and includes medical therapy with PPI and ARS.

LGD requires initial 6-month surveillance endoscopies with biopsies performed in four quadrants every 1 cm with subsequent endoscopies yearly.

HGD should be treated; if it is not treated, it should be surveilled every 3 months.

Endoscopic therapies for dysplasia include radiofrequency ablation and PDT; however, no head-to-head trial has been performed to show superiority.

Endoscopic mucosal resection, EMR, is a less invasive option for evaluation of the depth of lesions as well as resection of HGD and early adenocarcinoma, T1a lesions <2 cm.

The risk of progression of HGD to adenocarcinoma is 6% per year; for nondysplastic Barrett’s esophagus, it is approximately 0.5% per year, which is similar to colon polyps.

344
Q

An otherwise healthy 40-year-old man seeks treatment in the emergency department because of hematemesis after a night of binge drinking and retching. Which of the following is true of his condition?

A. It is caused by a pulsion diverticulum.

B. Endoscopy should not be performed because of the increased risk for perforation.

C. The bleeding is from an arterial source.

D. Surgical resection is often required.

E. H. pylori infection is a known risk factor.

A

ANSWER: C

COMMENTS: Mallory-Weiss tears are linear tears in the esophagogastric mucosa that cause bleeding in patients with repeated emesis.

The diagnosis is made by endoscopy, and most bleeding stops spontaneously.

Because the source of the bleeding is arterial, pressure tamponade is not helpful and may lead to perforation of the esophagus.

For refractory bleeding, endoscopic injection or cautery can be used, but definitive treatment requires a gastrotomy and suture ligation.

345
Q

A 60-year-old man has GERD and episodic dysphagia. An upper gastrointestinal contrast-enhanced study shows a type I hiatal hernia and thin band-like narrowing of the distal end of the esophagus. Which of the following is true of his condition?

A. Oral dilation is the treatment of choice.

B. It is the result of hypertrophy of the circular muscle layer.

C. Endoscopic mucosal resection is recommended.

D. There is squamous mucosa above and below the narrowing.

E. Surgical resection is indicated.

A

ANSWER: A

COMMENTS: Schatzki rings are concentric constrictions of the distal end of the esophagus (Fig. 20.3) occurring at the squamocolumnar junction; as a result, there is esophageal mucosa above and gastric mucosa below.

The rings consist of muscularis mucosa, connective tissue, and submucosal fibrosis.

Treatment involves oral dilation, which can provide relief for up to 18 months.

Excision of the rings should be avoided because the esophageal strictures that result from resection are much more difficult to manage.

346
Q

With regard to squamous cell carcinoma of the esophagus, which of the following is true?

A. Worldwide, it is the most common type of esophageal cancer.

B. It affects mainly Caucasian males.

C. It has a strong association with columnar epithelial metaplasia.

D. There is no proven association with alcohol or tobacco.

E. The male-to-female ratio is 15:1.

A

ANSWER: A*

COMMENTS: Esophageal cancer is the sixth most common malignancy and has an incidence of 20 per 100,000 in the United States.

Worldwide, squamous cell carcinoma is the most common type; however, in the United States, adenocarcinoma accounts for up to 70% of patients with esophageal cancer.

The male-to-female ratio is 3:1 for squamous cell carcinoma and 15:1 for adenocarcinoma. In addition, squamous cell carcinoma mainly affects African-American men, whereas adenocarcinoma mainly affects white men.

Alcohol and tobacco smoking increase the risk for esophageal cancer fivefold each and 25-fold to 100-fold in combination.

Additives such as nitrosamines in pickled and smoked foods have been implicated in the risk for cancer.

347
Q

A 75-year-old white man with a history of alcohol abuse, 40-pack-per year tobacco use, and long-standing GERD controlled by antacids is evaluated for dysphagia and weight loss. Esophagography shows an apple core lesion at the distal end of the esophagus. Which of the following is true regarding further workup?

A. Endoscopic biopsy should be avoided because of the risk for perforation.

B. CT is excellent for tumor staging.

C. Positron emission tomography (PET) is an excellent tool for staging and can be used as a single diagnostic modality.

D. Magnetic resonance imaging (MRI) is a poor imaging modality for liver metastasis.

E. EUS is more sensitive than CT for evaluating the celiac lymph nodes.

A

ANSWER: E

COMMENTS: Many imaging modalities are available for the characterization of esophageal cancers.

Barium esophagography is a good first test for patients with dysphagia and a history suspicious for cancer.

Although CT is accurate for M staging, it is only 57% accurate for T staging. PET is an excellent tool that can be used to evaluate N and M staging but should not be used as a single diagnostic modality.

MRI is excellent for detecting metastatic and T4 lesions. EUS is the most important diagnostic tool in esophageal cancer staging.

Tissue samples can be obtained from lymph nodes, as well as from the primary lesion.

EUS is more sensitive and specific than CT in evaluating the celiac lymph nodes.

348
Q

The patient in Question 25 underwent EUS that showed a T2 lesion. The biopsy specimen is positive for adenocarcinoma of the esophagus. His chance of having a positive lymph node is:

A. 20% 
B. 40% 
C. 60%
D. 80% 
E. 100%
A

ANSWER: C

COMMENTS: The risk of lymph node involvement is directly proportional to tumor depth or T stage.

The incidence of positive lymph nodes is:
18% for T1a intramucosal,
55% for T1b submucosal,
60% for T2 not beyond the muscularis propria,
80% for T3 with the involvement of paraesophageal tissue but not adjacent structures, and
100% for T4 with the involvement of adjacent structures.

This is related to the anatomy of lymphatic drainage in the esophagus.

The lymph channels travel within the submucosa in an axial fashion, allowing cancer to travel significant distances in cranial and caudal directions before causing an obstruction or draining beyond the muscularis.

349
Q

The patient in Question 25 undergoes neoadjuvant chemo-radiation therapy. Which of the following is true regarding multimodality therapy?

A. A complete histologic response occurs in approximately 25% of patients.

B. Squamous cell carcinoma and adenocarcinoma cell types have similar response rates to radiation therapy.

C. Survival beyond 5 years has not been reported in patients with stage IV disease.

D. Cisplatin-based combination therapy is no longer used because of the high rate of neuropathy.

E. Radiation therapy alone is an option for stage I disease.

A

ANSWER: A

COMMENTS: Treatment of esophageal cancer is complex, and multiple modalities are often necessary.

The decision regarding treatment options depends on whether the intent is curative or palliative.

Squamous cell carcinoma is much more radio responsive than adenocarcinoma, although with the latter, a complete histologic response is seen in approximately 25% of patients undergoing neoadjuvant chemoradiation therapy.

Neoadjuvant radiation therapy is limited to 4500 cGy to avoid the surgical morbidity associated with high-dose radiation.

Cisplatin, in combination with 5-fluorouracil and epirubicin, is an established chemotherapy regimen, but the use of mitomycin C, etoposide, and paclitaxel as third agents is gaining favor.

Stage I disease is best treated with curative surgical resection.

350
Q

The patient in Question 25 undergoes transhiatal esophagectomy. Which of the following is true of the procedure?

A. Three incisions are required: cervical, thoracic, and abdominal.

B. A gastric conduit is preferred, and the blood supply is based on the right gastroepiploic artery.

C. More lymph nodes can be harvested than with en- bloc esophagectomy.

D. A substernal route of the replacement conduit is preferred because of the shorter route and improved function.

E. Cervical anastomotic leak rates are lower than thoracic leak rates but carry the same morbidity.

A

ANSWER: B

COMMENTS: Transhiatal esophagectomy was first described by Wolfgang Denk in 1913 and popularized by Orringer in the 1980s.

Incisions on the left side of the neck and abdomen are used, and a thoracotomy is avoided.

The esophagus is bluntly dissected, and the tabularized stomach is pulled through the posterior mediastinum to create a cervical esophagogastric anastomosis.

The blood supply to the gastric conduit is based on the right gastroepiploic artery.

A gastric pull-up procedure, based on the right gastroepiploic artery, in the posterior mediastinal position has the best functional result.

Alternative routes (subcutaneous, substernal, or right pleural space) or conduits (colon or jejunum) can be used but result in an inferior function.

Thoracic anastomosis, as the result of an Ivor-Lewis esophagectomy, results in a lower leak rate but carries a much higher morbidity and mortality as a result of associated mediastinitis.

Thoracic esophagectomy is also associated with a greater lymphadenectomy due to direct access of the mediastinum for dissection.

This has not been equivocally proven to yield improved disease-free or overall survival.

When all three incisions are combined (cervical, thoracotomy, and laparotomy), this is referred to as a three hole, or a McKeown.

351
Q

An 88-year-old female with known advanced adenocarcinoma of the midesophagus presents with progressive dysphagia and is found on esophagogram to have a severe narrowing of her distal esophageal lumen. Which of the following is true regarding palliation of dysphagia in esophageal carcinoma?

A. Covered stents have less migration than uncovered stents.

B. PDT provides good relief in bulky, deeply penetrating tumors.

C. Neodymium:yttrium-aluminum-garnet (Nd:YAG) laser ablation is effective in proximal lesions.

D. Chemoradiation provides immediate relief.

E. Cervical esophagostomy (spit fistula) is a first-line
therapy.

A

ANSWER: C

COMMENTS: The goal of palliative care is to relieve suffering and improve the overall quality of life by reducing tumor burden and restoring nutritional access.

Chemotherapy, radiation therapy, PDT, laser treatment, stenting, and feeding tubes are options for palliation.

Dysphagia is the most common initial symptom of patients with esophageal cancer, especially in those with advanced disease.

Endoscopic laser fulguration with the Nd:YAG laser is 75%–80% successful in treating dysphagia, but multiple treatments may be required, and the risk of perforation is highest at 10%.

PDT, an alternative form of laser therapy, is also an excellent tool for palliation in patients suffering from dysphagia.

A photosensitizing drug [porfimer (Photofrin)] is injected intravenously before treatment and is selectively taken up by neoplastic cells.

Activation of Photofrin with a red light at 630 nm releases singlet oxygen, which kills the host cell.

The depth of penetration is limited to 5 mm and thus not great for deep lesions or extrinsic compression. Self- expanding metal stents (SEMS) provide good immediate relief from dysphagia.

Covered stents prevent ingrowth of cancer into the stent, which can lead to subsequent recurrent stenosis.

Covered stents also have a high incidence of migration compared with uncovered stents.

In general, stents cannot be used for cervical lesions due to difficulty in swallowing after placement and for GEJ lesions due to severe reflux after placement.

PDT may work best for GEJ lesions, and Nd:YAG ablation works well in cervical lesions that cannot be stented.

Chemotherapy is used to treat systemic disease and reduce overall tumor burden but needs to be administered with radiation therapy to control local disease and can take weeks to be effective.

As such it should be reserved for patients with a longer life expectancy.

352
Q

The patient in the above question undergoes serial dilation and Nd:YAG ablation. She is observed overnight for symptoms. The next morning, she complains of chest pain, is febrile to 102.4°F, and has a heart rate of 128 beats/min. Esophagogram confirms extravasation of contrast. What is the most common cause of esophageal perforation?

A. Iatrogenic

B. Trauma

C. Cancer

D. Boerhaave’s syndrome

E. Caustic injury

A

ANSWER: A

COMMENTS: Iatrogenic causes account for the largest cause of esophageal perforation, typically as the result of instrumentation including esophagoscopy, transesophageal echocardiography, and dilation (pneumatic and bougie).

Boerhaave’s syndrome is the most common cause of spontaneous esophageal perforation, accounting for 15% of cases.

Cancer only accounts for 1% of esophageal perforations.

353
Q

Which of the following is true regarding the management of
esophageal perforation?

A. The distal esophagus is best approached through the right chest.

B. Cervical perforations occur most commonly at the thoracic inlet.

C. Primary repair alone is sufficient in early perforation.

D. Diversion and wide drainage are best in high-risk patients.

E. Resection is required in perforation with achalasia.

A

ANSWER: D

COMMENTS: Esophageal perforation carries a high morbidity and between a 10% and 40% mortality.

Traditional teaching is that repair of esophageal perforation should only occur if identified early; however, modern studies have reported great success with repair of delayed injuries as well.

Esophageal repair should involve a two-layer repair with an onlay buttress, typically of pericardium, pleural fat, or a pedicle intercostal muscle flap.

The cervical esophagus is best approached via a left neck incision anterior to the sternocleidomastoid.

Perforations here are most commonly found at the cricopharyngeus/UES as this is the highest pressure zone.

The upper two-thirds of the thoracic esophagus is best approached via a right thoracotomy and the distal one-third via a left thoracotomy.

The intraabdominal esophagus is best approached via an upper midline laparotomy.

In achalasia, most perforations are the result of pneumatic dilation and can be managed with repair and myotomy at a different location; however, resection is not required.

For patients with perforation associated with a malignancy, resection should be considered if feasible.

Recently covered stent placement with wide mediastinal drainage allows temporization and an oncologic operation to occur under more elective circumstances.

Cervical esophagostomy as a diversion with wide mediastinal drainage and feeding tube placement should be reserved for those with prohibitive risk to resection, short life expectancy, or significant gross contamina- tion and sepsis.

354
Q

A 45-year-old man arrives at the emergency department after ingesting lye in a suicide attempt. Which of the following is true?

A. Injury to the esophagus is the result of coagulative necrosis.

B. Endoscopy should not be performed within the first 72 h because of the risk for perforation.

C. The lye should be neutralized with milk or egg whites if the patient is seen within the first hour of ingestion.

D. Before reepithelialization, dilations should be performed to decrease the long-term stricture rate.

E. For a long-segment interposition graft, the colon is the preferred conduit.

A

ANSWER: E

COMMENTS: Caustic injury to the esophagus can be attributed to the ingestion of acidic or alkaline liquids.

Alkaline substances (lye) produce liquefactive necrosis of tissue and can cause deep tissue penetration. If diagnosed within the first hour of ingestion, half-strength vinegar or citrus juice can be used to neutralize the ingested alkali.

After careful examination of the oropharynx, airway, chest, and abdomen, endoscopy should be performed to grade the burn.

Serial esophagograms should be performed to evaluate for stricture formation rather than waiting for symptoms of obstruction to develop.

Early stent placement or bougie dilation is effective in preventing long-term strictures; however, dilation should be performed only after reepithelialization has been confirmed with endoscopy.

For long-segment strictures requiring resection, colonic interposition is the preferred graft.

355
Q

A 65-year-old patient had progressive dysphagia, weight loss and melena, the most important initial diagnostic test for this patient is:

A. Endorectal ultrasound

B. PET scan

C. Manometry

D. 24h pH monitoring

E. Endoscopy

A

E. Endoscopy

356
Q

A 29-year-old patient was suspected to have a lower third esophageal carcinoma. The least important work-up is:

A. PET CT scan

B. CT scan of the liver

C. Peritoneal lavage with fluid cytology

D. Barium swallow

E. Endoscopic ultrasound

A

C. Peritoneal lavage with fluid cytology

357
Q

Odynophagia:

A. Abnormal bleeding in the upper esophagus

B. Abdominal pain on swallowing

C. Continuous bleeding in the upper esophagus

D. Pain on swallowing

E. Upper airway obstruction

A

D. Pain on swallowing

358
Q

A 22-year-old female ingested a 100mL of a strong alkali solution. Treatment strategy include:

A. Induce emesis

B. Emergency gastrostomy tube insertion for feeding

C. Give fresh frozen plasma

D. Intravenous fluid resuscitation

E. Give acid orally to neutralize solution

A

D. Intravenous fluid resuscitation

359
Q

Corkscrew esophagus

A. Nutcracker esophagus

B. Zenker’s diverticulum

C. Epiphrenic diverticulum

D. Achalasia

E. Diffuse esophageal spasm

A

E. Diffuse esophageal spasm

360
Q

Not true regarding Barrett’s esophagus

A. Ulceration in the columnar lined segment

B. Stricture formation in the lower esophagus

C. Most common malignant transformation is adenocarcinoma

D. Severe stage of GERD

E. Presence of squamous mucosa extending at least 3cm into the esophagus

A

E. Presence of squamous mucosa extending at least 3cm into the esophagus

361
Q

The best approach to a patient with distal third stage II esophageal cancer is

A. Chemotherapy

B. Rubber band ligation to promote necrosis of tumor

C. Insertion of a gastrostomy tube

D. Left thoracotomy with resection of distal esophagus, gastric pull-up with abdominal anastomosis and mediastinal and upper abdomen lymph node dissection

E. Transhiatal esophagectomy with pyloromyotomy, gastric pull-up, and anastomosis to the cervical esophagus

A

E. Transhiatal esophagectomy with pyloromyotomy, gastric pull-up, and anastomosis to the cervical esophagus

362
Q

Mallory-Weiss tear:

A. Typical longitudinal mucosal tear with overlying fibrinous exudate extending from the proximal esophagus to the gastric cardia

B. Associated with severe smoking

C. Very much associated with the development of malignancy

D. Sucralfate is contraindicated

E. Can be treated with endoscopic band ligation

A

A. Typical longitudinal mucosal tear with overlying fibrinous exudate extending from the proximal esophagus to the gastric cardia

363
Q

To readily assess the presence of lymph nodes adjacent to the esophageal cancer, you can request for:

A. Ultrasound of the chest

B. Mediastinoscopy

C. Manometry

D. Bronchoscopy

E. Endoscopic ultrasound

A

E. Endoscopic ultrasound

364
Q

A 60-year-old male, heavy smoker and heavy alcohol drinker who works as a painter presented with progressive dysphagia for 2 years. Endoscopy revealed a 1.2cm mass in the gastroesophageal junction. The mass was ulcerated, with bleeding, and the mucosa is rugged over the mass. It is most likely:

A. Adenocarcinoma

B. Squamous cell carcinoma

C. Leiomyoma

D. Epiphrenic diverticulum

E. GERD

A

A. Adenocarcinoma

365
Q

Obviously not true of esophageal cancer:

A. Anemia is a common problem

B. PET/CT scan may show tumors not seen in CT or MRI

C. Laparoscopy is used in checking for invasiveness in surrounding tissues in lower third esophageal tumors

D. Esophageal cancer is associated with parasitic infections

E. May metastasize to the lungs

A

D. Esophageal cancer is associated with parasitic infections

366
Q

Indications for laparotomy in a patient who ingested 200mL of a strong alkali solution are the following except:

A. Persistent retrosternal pain

B. Abdominal pain and tenderness

C. Continuous bleeding

D. Melena

E. Mediastinitis

A

D. Melena

367
Q

A 22-year-old female ingested a 100mL of a strong alkali solution. Treatment strategy includes

A. Induce emesis

B. Gastrostomy tube insertion within 2 days for feeding purposes

C. Immediate diagnostic laparoscopy to assess if there is perforated bowel

D. Immediate oral feeding to minimize malnutrition

E. Colon interposition if esophagus will be removed.

A

E. Colon interposition if esophagus will be removed.

368
Q

Obviously not true of esophageal cancer:

A. Adenocarcinoma is most common in the middle third

B. PET scan may show tumors not seen in CT or MRI

C. Upper GI endoscopy is helpful

D. Esophageal cancer may cause vomiting

E. Anastomosis after transmittal esophagectomy is usually done in the neck.

A

A. Adenocarcinoma is most common in the middle third

369
Q

Which if the following statements about the surgical treatment of esophageal carcinoma is/are correct?

A. The finding of severe dysplasia in association with Barrett’s mucosa is an indication for an anti reflux operation to prevent subsequent development of carcinoma.

B. Long-term survival is improved by radical en bloc resection of the esophagus with its contained tumor, adjacent mediastinal tissues, and liver metastasis.

C. The morbidity and mortality rates for cervical esophagogastric anastomotic leak are substantially less than those associated with intrathoracic esophagogastric anastomotic leak.

D. The leading complications of transthoracic esophagectomy and intrathoracic esophagogastric anastomosis are bleeding and wound infection.

E. Transhiatal esophagectomy without thoracotomy achieves better long-term survival than transthoracic esophagectomy.

A

C. The morbidity and mortality rates for cervical esophagogastric anastomotic leak are substantially less than those associated with intrathoracic esophagogastric anastomotic leak.

370
Q

Which of the following is most reliable for confirming the occurrence of a significant esophageal caustic injury?

A. History of the event.

B. Physical examination of the patient

C. Barium swallow

D. Endoscopy

E. Chest x-ray

A

D. Endoscopy

371
Q

A 58-year-old male comes to the emergency room complaining of substernal chest pain after an episode of vomiting. His BP is 155/85 and his pulse is 88bpm. The patient’s abdomen is soft, but tender at the epigastrium. The patient had a normal EKG and no elevation in troponin levels.

What is the next best test to determine the diagnosis?

A. Upper GI endoscopy

B. Gastrograffin esophagogram

C. CT scan of the chest

D. Echocardiogram

E. Operative exploration

A

B. Gastrograffin esophagogram

Boerhaave’s Syndrome

372
Q

A 35-year-old female presents with complaints of dysphagia, regurgitation, and weight loss. She has had progressive dysphagia which began with liquids and has now progressed to solids. She needs to consume large volumes of water when she eats. She sometimes regurgitates undigested food.

All of the following are consistent with her most likely disease process except:

A. Esophagogram will show a bird’s beak appearance

B. Manometry will show a hypertensive lower esophageal sphincter (LES) which fails to relax during swallowing

C. There will be no evidence of progressive peristalsis

D. There are high amplitude pressures during manometry

E. EGD will likely show retained food particles in esophagus

A

D. There are high amplitude pressures during manometry

373
Q

A patient well known to your office with a history of chronic reflux and Barrett’s esophagus presents for his scheduled surveillance EGD. Biopsies are taken during the procedure that show evidence of high grade dysplasia. What is the BEST choice in management of this patient?

A. Repeat endoscopy in 3 months

B. Repeat endoscopy in 6 months

C. Add H2 blocker

D. Schedule a Nissen fundoplication

E. Esophagectomy

A

E. Esophagectomy

374
Q

A 42-year-old female presents with long-standing gastroesophageal reflux disease (GERD). She has had previous endoscopies which have showed esophagitis. The most recent endoscopy showed evidence of Barrett’s esophagus without dysplasia. She has been treated with a proton pump inhibitor (PPI) daily for 3 years. What is the MOST appropriate next step in management?

A. Proton pump inhibitor (PPI) twice daily

B. Serial endoscopies with random biopsies

C. Observation

D. Nissen fundoplication

E. Esophagectomy

A

D. Nissen fundoplication

375
Q

A 62-year-old male with known esophageal cancer presents to the emergency room complaining of retrosternal chest pain and shortness of breath that started 6 hours prior to arrival. He is febrile and tachycardic, but his blood pressure is normal. On laboratory work-up, he has no elevation in troponins, but does have leukocystosis. A swallow study is performed which shows a free perforation at the mid-esophagus. Which of the following is the BEST choice in management for this patient?

A. Resuscitation in the ICU

B. Surgical repair via a left thoracotomy

C. Primary repair of the perforation with drain placement and a muscle flap

D. Diverting cervical esophagostomy with chest tube placement

E. Esophagectomy with cervical esophagostomy, gastrostomy, and feeding jejunostomy

A

E. Esophagectomy with cervical esophagostomy, gastrostomy, and feeding jejunostomy

376
Q

A 63-year-old male presents to the hospital complaining of dysphagia and a 20-pound weight loss over the last 6 months. The patient has an EGD performed that notes a distal esophageal mass with biopsies showing evidence of adenocarcinoma and invasion into the muscularis mucosa. Which of the following is the BEST treatment option?

A. Neoadjuvant chemoradiotherapy followed by esophagectomy

B. Esophagectomy alone

C. Esophagectomy with lymph node dissection

D. Esophagectomy with adjuvant chemotherapy

E. Chemoradiotherapy alone

A

B. Esophagectomy alone

377
Q

An 18-year-old female is brought to the emergency department after ingestion of an acidic substance. A chest x-ray does not show evidence of free air or pneumomediastinum. an EGD is performed that shows ulcerations of the esophagus with no perforations. Which of the following is the BEST choice in management of this patient?

A. Observe clinically for 48hours

B. Repeat EGD in 48h

C. Gastrograffin swallow study in 48hours (esophagogram)

D. Stent placement at time of initial EGD

E. Schedule an esophagectomy

A

C. Gastrograffin swallow study in 48hours (esophagogram)

378
Q

A 65-year-old male with past medical history significant for tobacco abuse presents with difficulty swallowing. He states his symptoms have increased in severity and he now is having problems swallowing even soft foods. You perform an upper endoscopy and find a tumor obstructing 65% of the circumference of the middle portion of the esophagus. Biopsy reveals an esophageal squamous carcinoma.

Which BEST determines his chance for potential cure?

A. Endoscopic ultrasound (EUS)

B. Positron emission tomography (PET) scan

C. Magnetic resonance imaging (MRI) of the chest and abdomen

D. Computed tomography of the chest and abdomen

E. Barium swallow study

A

A. Endoscopic ultrasound (EUS)

379
Q

Functional grades of dysphagia?

A

I: Eating normally

II: Requires liquids with meals

III: Able to take semisolids but unable to take solids

IV: Liquids only

V: Able to swallow saliva

VI: Unable to swallow saliva

380
Q

Management of Esophageal CA?

A

Locoregional disease
Stage I-III
- Esophagectomy.
- Others: Endoscopic therapy (eg mucosal resection), Preoperative chemoradiation

Metastatic disease
Stage IV
- Unresectable
- ECOG less than or equal to 2: Chemo+palliation
- ECOG >2: Palliation
381
Q

Contraindications for curative surgery in esophageal CA?

A
Age >75yo
FEV1 <1.25 and EF <40%
>20% weight loss
Locally advanced tumor
— Horner syndrome
— RLN paralysis
— Paralysis of diaphragm
— Malignant pleural effusion
— Length >9cm
— >4 LN involved

Distant Mets

382
Q

Role of chemo radiation in esophageal CA?

A

Current standard for non-surgical management of locally advanced disease.

CIS+ 5FU

For advanced resectable disease: neoadjuvant chemo radiation is the current approach

383
Q

Surgical approaches to esophageal CA?

A

1) Vagal-sparing esophagectomy
- reserved for patients with esophageal cancer limited to the intramucosal layer without the need for lymphadenectomy

2) Transhiatal (Orringer and Sloan)
- Performed via upper midline laparotomy and left cervical incision
- Gastric conduit based on the right gastroepiploic artery is used to establish GI continuity (if stomach is unusable, a colonic interposition can be performed)

3) Transthoracic (Ivor-Lewis)
- Upper midline abdominal incision and a right thoracotomy is done
- Esophagus is dissected out of its bed, stomach is pulled up into the thorax through esophageal hiatus and divided at its cardiac portion to allow creation of an anastomosis between the distal end of the esophagus and the fundus of the stomach.

4) Three-field (McKeown)
- Carried out through separate laparotomy, right thoracotomy, and cervical incisions

5) Left thoracoabdominal (Akiyama)
- Performed by making an oblique incision from the midpoint between the xiphoid and umbilicus across the costal arch to the tip of the scapula.
- Abdomen is opened, the costal arch is divided, and the chest is entered through the 7th ICS
- Diaphragm is opened in a circumferential manner along the chest wall

6) Minimally Invasive
- Include laparoscopic, hand-assisted, thoracoscopic and robot-assisted esophagectomy

384
Q

Siewert and Stein Classification of Esophagogastric Junction Carcinoma?

A

Siewert I: Esophageal
TTE + two-field LAD

Siewert II: Cardia
TTE or
THE (R0)

Siewert III: Subcardiac
Total Gastrectomy + D2 LAD

385
Q

Staging of esophageal and esophagogastric cancer?

A

T1 Invaded lamina propria, muscularis mucosae, or submucosa
T1a Lamina propia/ muscularis mucosae
T1b Submucosa

T2 Invades muscularis propria

T3 Invades adventitia

T4 Invades adjacent structures
T4a Resectable tumor invading pleura, pericardium/diaphragm
T4b Unresectable tumor invading other adjacent structures, such as aorta, vertebral body, trachea, etc.

N1 1-2 LN
N2 3-6 LN
N3 >7 regional LN

386
Q

Two-field lymphadenectomy involves?

A

1) Paraesophageal, peritumoral, subcarinal LN

2) Aortopulmonary window along mainstream bronchi and along L recurrent nerve.

387
Q
  1. A 79-year-old retired opera singer presents with dysphagia, which has become progressively worse during the last 5 years. He states that he is sometimes aware of a lump on the left side of his neck and that he hears gurgling sounds during swallowing. He sometimes regurgitates food during eating. What is the likely diagnosis?

(A) Carcinoma of the esophagus

(B) Foreign body in the esophagus

(C) Plummer-Vinson (Kelly-Patteson)
syndrome

(D) Zenker’s (pharyngoesophageal)
diverticulum

(E) Scleroderma

A
  1. (D) A Zenker’s (pharyngoesophageal) diverticulum is a mucosal outpouching through the
    triangular bare area between the cricopharyngeus muscle and the inferior constrictor muscle
    of the pharynx (Killian’s triangle).

Most present on the left side of the neck.

388
Q
  1. A symptomatic patient has a barium swallow that reveals a 3-cm Zenker’s diverticulum. The next step in management is?

(A) H2 blockers

(B) Anticholinergic drugs

(C) Elemental diet

(D) Bougienage

(E) Surgery (cricopharyngeal myotomy and diverticulectomy)

A
  1. (E) The current surgical treatment for a symptomatic pharyngoesophageal diverticulum is
    myotomy.

If the diverticulum is >2 cm, it should
be resected.

Small asymptomatic diverticula require no treatment.

Failure of relaxation of the cricopharyngeus muscle is thought to result in the development of the diverticulum.

389
Q
  1. A 30-year-old psychiatric patient has a barium
    swallow after removal of a foreign body to rule out a small perforation of the esophagus. No perforation is seen, but an epiphrenic diverticulum is visualized. An epiphrenic diverticulum may be associated with which of the following?

(A) Duodenal ulcer

(B) Gastric ulcer

(C) Cancer of the tongue

(D) Cancer of the lung

(E) Hiatal hernia

A
  1. (E) An epiphrenic (supradiaphragmatic) diverticulum is a pulsion diverticulum and is associated without any obvious lesions (35%) or
    with hiatal hernia (30%), diffuse esophageal
    spasm (DES) (20%), achalasia (10%), and miscellaneous causes (5%).

It is located with 10 cm of the cardia.

An epiphrenic diverticulum is commonly asymptomatic and should not be treated surgically unless symptoms are clearly related to it.

Parabronchial lymphadenopathy can cause traction diverticulae (which are located at a higher level).

390
Q
  1. A 64-year-old man develops increasing dysphagia over many months. A barium swallow
    is performed. What is the most likely cause of
    his clinical presentation?

(A) Carcinoma of the esophagus

(B) Achalasia

(C) Sliding hiatal hernia

(D) Paraesophageal hernia

(E) Esophageal diverticulum

A
  1. (A) The appearance of unexplained dysphagia in adults requires urgent evaluation.

Esophageal carcinoma is particularly prevalent in certain parts of Africa and Asia, but the incidence is
increasing in Western countries.

In achalasia there is initially a greater tolerance for solids over liquids.

In carcinoma, dysphagia for solids is noted initially, and later there is difficulty in swallowing liquids as well.

Esophagoscopy is required in the workup of dysphagia. It is imperative to rule out an underlying carcinoma.

391
Q
  1. A 63-year-old woman from Norway is visiting the United States. She presents with dysphagia. On endoscopy, an esophageal web is identified and the diagnosis of Plummer-Vinson syndrome is established. What would be the next step in management?

(A) Esophagostomy

(B) Dilatation of the web and iron therapy

(C) Esophagectomy

(D) Gastric bypass of the esophagus

(E) Cortisone

A
  1. (B) In addition to the presence of an upper
    esophageal web leading to dysphagia, the
    Plummer-Vinson syndrome is characterized by
    atrophic oral mucosa, spoon-shaped brittle nails
    (koilonychia), and iron deficiency anemia.

Endoscopy reveals a fibrous web just below the
cricopharyngeus muscle.

There is an increased risk of developing cancer of the esophagus.

392
Q
  1. A 53-year-old moderately obese woman presents with heartburn aggravated mainly by eating and lying down in the horizontal position. Her symptoms are suggestive of gastroesophangeal reflux disease (GERD). Which of
    the following statements is TRUE?

(A) It is best diagnosed by an anteroposterior (AP) and lateral film of the chest.

(B) It may be alleviated by certain drugs, especially theophylline, diazepam, and calcium channel blockers.

(C) It is not relieved by cessation of smoking.

(D) If it is associated with dysphagia, it suggests a stricture or motility disorder.

(E) It should be immediately treated with
surgery.

A
  1. (D) Nonoperative therapy is the initial treatment of GERD.

The treatment is weight loss, avoidance of fatty meals, smoking cessation, abstinence from alcohol, positional awareness, avoidance of lying supine, and avoidance of certain foods (e.g., chocolate) and drugs (e.g., theophylline, anticholinergic agents, a-adrenergic antagonists).

Dysphagia requires special attention to rule out a stricture, cancer, or a motility disorder. Poor results are more likely when previous surgery has failed and in patients with scleroderma.

393
Q
  1. A 64-year-old man has symptoms of reflux
    esophagitis for 20 years. The barium study shown (Fig. 5–2) demonstrates a sliding hiatal hernia. Whis is TRUE in sliding hiatal hernia?

(A) A hernia sac is absent.

(B) The cardia is displaced into the posterior mediastinum.

(C) Reflux esophagitis always occur.

(D) A stricture does not develop.

(E) Surgery should always be avoided.

A
  1. (B) The cardia is displaced into the posterior
    mediastinum. The term sliding hernia (Fig. 5–8)
    indicates that a part of the peritoneum slips or
    slides with the hernia into the posterior mediastinum.

The wall of the sac is formed medially by the stomach and laterally by the peritoneum.

Reflux esophagitis is more likely to occur with
this type of hernia.

The sliding hiatal hernia may be entirely symptomatic or lead to reflux esophagitis and possibly esophageal stricture.

394
Q
  1. A 45-year-old man presents with a long history of heartburn, especially at night. He uses three pillows to sleep and has medicated himself with a variety of antacids over the past 15 years. Recently he has been complaining of dysphagia that he localized to the precordial area. Which is the most likely diagnosis?

(A) Adenocarcinoma of the esophagus

(B) Angina pectoris

(C) Benign peptic stricture of the esophagus

(D) Achalasia of the esophagus

(E) Lower esophageal ring (Schatzki’s ring)

A
  1. (C) Benign peptic strictures of the esophagus
    are submucosal fibrotic rings that narrow the lumen and obstruct the passage of food.

They present with dysphagia.

They tend to be between 1 and 4 cm in length.

GERD is the most common cause.

Other associated motility disorders often occur.

Heartburn may improve because of the obstruction to refluxed bile.

395
Q
  1. A 54-year-old man presents with dysphagia,
    heartburn, belching, and epigastric pain. A barium swallow shows a sliding hiatal hernia and a stricture situated higher than usual in the mid-esophagus. Endoscopic findings suggest Barrett’s esophagus (ectopic gastric epithelium lining the esophagus). Marked esophagitis with linear ulcerations are seen during endoscopy. A biopsy shows columnar epithelium at the affected area and normal squamous epithelium above, confirming the diagnosis. What statement is TRUE regarding this condition?

(A) Adenocarcinoma is less common in Barrett’s esophagus

(B) Most patients do not have associated gastroesophageal reflux

(C) The presence of ectopic gastric lining protects against aspiration during sleep and prevents recurrent pneumonitis.

(D) The present treatment is aimed at preventing esophagitis.

(E) When strictures form, they are always malignant.

A
  1. (D) The present treatment is aimed at preventing esophagitis.

Barrett’s esophagus is regarded as a premalignant condition and is characterized by columnar metaplastic of the normal squamous epithelial lining of the esophagus.

The cancer risk is increased 20–50-fold.

About one-third of patients present with malignancy, and many cases of adenocarcinoma of the esophagus arise from Barrett’s mucosa.

There is an increased risk for the development of squamous carcinoma. It is found in 8–10% of patients with long standing reflux.

396
Q
  1. A 75-year-old woman presents with a paraesophageal hiatal “rolling” hernia. Diagnosis is
    made by radiologic studies (Fig. 5–3). What
    can this patient be told about paraesophageal
    hernias?

(A) They constitute about 50% of all esophageal hiatal hernias and are more common in women over the age of 60.

(B) They cause the gastroesophageal (GE) junction to become displaced from its normal position below the diaphragm to above the diaphragm.

(C) They prevent herniation of the stomach and intestine above the diaphragm.

(D) They may result in volvulus and strangulation of the stomach or bleeding.

(E) They are treated medically with attention to diet, position during sleep, antacids, and omeprazole [H+/K+ adenosine triphosphate (ATP-ase) pump
inhibitors]

A
  1. (D) This is a type 4 hiatal hernia (Fig. 5–9).

In the classic case of a paraesophageal “rolling” hernia, the GE junction remains below the hiatus, allowing the stomach and sometimes other viscera to migrate upward into the chest alongside the esophagus.

Paraesophageal hernias are prone to obstruction, bleeding, and volvulus (either mesoaxial or organoxial rotation).

Chronic symptoms include pain and postprandial fullness, with heartburn in 90% of cases.

Gastric ulcers develop in as many as 30% of cases and they may cause acute or chronic (MISSING BOTTOM LINE) indicated and effective to relieve symptoms and to prevent complications, which may be catastrophic.

397
Q
  1. A 52-year-old gastroenterologist suffers from intermittent dysphagia attributed to the presence of a lower esophageal stricture. The doctor’s condition is characterized by which of the following?

(A) A full thickness scar in the upper esophagus

(B) Symptoms of mild-to-moderate dysphagia

(C) A low incidence in men

(D) The absence of a sliding hiatal hernia in
most cases

(E) The need for antireflux surgery at an early stage

A
  1. (B) Schatzki’s ring is a thin, circumferential
    scar in the lower esophagus, more common in
    men (65) or greater.

It is acquired and probably results from repeated trauma to the mucosa with chronic inflammation and fibrosis.

Endoscopic dilation is the usual treatment. It is usually successful but antireflux surgery is occasionally necessary for severe GERD, especially if it is worsened by dilatation.

Associated hiatal hernia is very common.

398
Q
  1. A 54-year-old clerk complains of having had dysphagia for 15 years. The clinical diagnosis of
    achalasia is confirmed by a barium study. What
    is TRUE in this condition?

(A) The most common symptom is dysphagia.

(B) In the early stages, dysphagia is more
pronounced for solids than liquids.

(C) The incidence of sarcoma is increased.

(D) Recurrent pulmonary infections are rare.

(E) Endoscopic dilatation should be avoided.

A
  1. (A) Dysphagia in esophageal achalasia is
    described as paradoxical in that it is more pronounced for liquids than solids.

There are numerous reports of an increased incidence of carcinoma in achalasia, ranging from 3% to 10%. In 1975, Belsey reported a 10% incidence in 81 patients in whom symptoms tended to occur at a younger age.

Recurrent lung infections from aspiration of esophageal contents are a troublesome complication.

The treatment is surgical myotomy or endoscopic dilatation.

399
Q
  1. A 69-year-old man is admitted to the emergency department with an acute UGI hemorrhage following a bout of repeated vomiting. Fiberoptic gastroscopy reveals three linear mucosal tears at the GE junction. What is the diagnosis?

(A) Reflux esophagitis with ulceration

(B) Barrett’s esophagus

(C) Carcinoma of the esophagus

(D) Mallory-Weiss tear

(E) Scleroderma

A
  1. (D) A Mallory-Weiss tear is characterized by
    acute and sometimes massive UGI hemorrhage.

It accounts for up to 10% of UGI bleeds.

It is due to arterial bleeding following repeated vomiting (which causes mucosal tears at the GE junction).

The cause is the same as that for spontaneous
rupture of the esophagus (i.e., an increase in
intra-abdominal pressure against a closed glottis).

Causes other than vomiting such as paroxysmal coughing or retching, may sometimes lead to this condition.

Upper endoscopy confirms the diagnosis. Surgery may occasionally be necessary to stop the bleeding.

400
Q
  1. A 60-year-old man presents with excruciating chest pain. The pain follows an episode of violent vomiting that occurred after a heavy meal. Subcutaneous emphysema was noted in the
    neck. X-rays shows air in the mediastinum and
    neck, and a fluid level in the left pleural cavity.
    What is the most likely diagnosis?

(A) Perforated duodenal ulcer

(B) Spontaneous rupture of the esophagus

(C) Spontaneous pneumothorax

(D) Inferior wall myocardial infarction

(E) Dissecting aortic aneurysm

A
  1. (B) Spontaneous rupture of the esophagus, or
    Boerhaave’s syndrome, is most common in men
    between 35 and 55 years of age.

The usual presentation is severe pain in the precordium, lower thorax, or epigastrium. Classically it follows an episode of violent vomiting.

A chest film shows hydropneumothorax usually on the left side, but it may be on the right side or bilateral.

Free air below the diaphragm is not a usual finding.
The tear is usually located above the diaphragm
and is longitudinal on the left posterolateral wall. Air passes around the mediastinum, which results in subcutaneous emphysema.

401
Q
  1. A patient is diagnosed with Boerhaave’s syndrome. Management involves which of the
    following?

(A) Administration of intravenous antibiotics and TPN

(B) Administration of intravenous antibiotics and TPN, and insertion of a chest tube and a nasogastric tube

(C) Administration of intravenous antibiotics and TPN, and insertion of a nasogastric tube

(D) Resuscitation and emergency surgery
either by laparotomy or thoracotomy

(E) Resuscitation, administration of intravenous antibiotics, replacement of fluids and electrolytes; elective surgical intervention when the general status of the patient improves

A
  1. (D) Spontaneous rupture of the esophagus is an
    acute emergency.

It requires efforts to establish a rapid diagnosis followed by an emergency operation.

Rapid resuscitation and antibiotics should be instituted prior to surgery.

Shock is not a contraindication to surgery because it is unlikely that the patient’s condition will improve until surgery has been performed.

The surgical approach is usually thoracic, but the abdominal approach may also be used.

402
Q
  1. A chest CAT scan is done to further delineate
    an abnormality seen on a chest x-ray. The superior mediastinum at the level of T4 is evaluated.
    Which structure is remote from the esophagus?
(A) Trachea
(B) Recurrent laryngeal nerves
(C) Aorta
(D) Azygous vein
(E) Brachiocephalic vein
A
  1. (E) The esophagus is a posterior mediastinal
    structure in much of its course.

The thymus gland is located in the anterior mediastinum.

The recurrent laryngeal nerve runs between the
trachea and the esophagus.

The aorta loops backward over the left side of the esophagus. At this level the thoracic duct is on the left side of the esophagus.

The brachiocephalic vein is the most anterior structure in the superior mediastinum.

403
Q
  1. A 69-year-old man is informed that the cause of
    his dysphagia is a benign lesion. The barium swallow is shown in (Fig. 5–4). What should he
    be told regarding benign tumors and cysts of
    the esophagus?

(A) They occur more commonly than malignant tumors.

(B) They are symptomatic at an early age.

(C) Diagnosis is best confirmed on chest x-ray.

(D) Leiomyoma is the most common benign tumor encountered in the esophagus.

(E) Malignant transformation of a benign leiomyoma into a malignant leiomyosarcoma is common.

A
  1. (D) Leiomyoma is the most common benign
    tumor encountered in the esophagus.

Malignant transformation is thought to be rare.

Less than 10% of alimentary tract leiomyomas are found in the esophagus.

They are composed of spindle cells and grow slowly and may progressively cause obstructive symptoms.

Leiomyomas are not referred to as a benign GIST.

Other benign lesions are congenital or acquired cysts, adenomatous polyps, papillomas, lipomas, neurofibromas, and hemangiomas.

404
Q
  1. A 45-year-old pilot has retrosternal burning, especially when he eats and lies down to go to
    sleep. He has self-medicated himself with over
    the counter heartburn medications. Upper
    endoscopy reveals an erythematous and
    inflamed distal esophagus. In severe reflux
    esophagitis, the resting pressure of the LES is
    decreased. This may be physiologically
    increased by which of the following?

(A) Pregnancy

(B) Glucagon

(C) Gastrin

(D) Secretin

(E) Glucagon

A
  1. (C) The gastroesophageal zone of elevated
    pressure is 3–4 cm long and has a resting pressure of 15-cm H2O.

Pregnancy, obesity, and gastric dilatation, all result in increased intra-abdominal pressure and can result in reflux.

Alkalinization of the stomach, gastrin, epinephrine, cholinergic agents (bethanecol), and a-adrenergic agents (metoclopromide) increase the resting pressure of the LES.

Anticholinergic agents (atropine), glucagon, and secretin decrease the resting pressure, and is released by the vagus nerve and it stimulates the production of acid in the stomach.

405
Q
  1. A 46-year-old man has a long history of heartburn (GERD). His barium study shows an irregular, ulcerated area in the lower third of his
    esophagus. There is marked mucosal disruption and overhanging edges. What is the most
    likely diagnosis?

(A) Sliding hiatal hernia with GERD

(B) Paraesophageal hernia

(C) Benign esophageal stricture

(D) Squamous carcinoma of the esophagus

(E) Adenocarcinoma arising in a Barrett’s
esophagus

A
  1. (E) The history of GERD coupled with these findings is highly suggestive of an adenocarcinoma
    arising in a Barrett’s esophagus.

Squamous carcinoma is more likely to occur higher up in the middle third of the esophagus.

Endoscopy and biopsy prove the diagnosis.

The patient should be treated surgically by esophagectomy if carcinoma is confirmed.

Inoperable upper esophageal squamous cell carcinomas can be treated with chemoradiation (survival outcomes are similar to surgery with less morbidity).

406
Q
  1. A 46-year-old man present with dysphagia of recent onset. His esophagogram shows a lesion in
    the lower third of his esophagus. Biopsy of the
    lesion shows adenocarcinoma. His general
    medical condition is excellent, and his metastatic workup is negative. What should his management involve?

(A) Chemotherapy

(B) Radiation therapy

(C) Insertion of a stent to improve swallowing

(D) Surgical resection of the esophagus

(E) Combination of chemotherapy and radiation therapy

A
  1. (D) Surgical resection of the esophagus remains
    the recommended treatment for patients with
    carcinoma of the lower esophagus, provided
    that there is no metastatic disease and the
    patient’s overall medical condition is compatible with a major operation.

This offers the best palliation and the only hope for cure. The 5-year survival rates vary between 15 and 25%.

Radiation and chemotherapy, in combination with surgery in selected patients, may improve these statistics.

There are four types of esophagectomy—transthoracic, en bloc, transhiatal, and video-assisted.

Regardless of whatntype of operation is performed, complete macroscopic and microscopic removal of
tumor, is the goal.

407
Q
  1. A 25-year-old man arrives in the emergency
    department in respiratory distress following a motor vehicle collision. A chest x-ray shows abdominal viscera in the left thorax. What is the most likely diagnosis?

(A) Traumatic rupture of the diaphragm

(B) Sliding esophageal hernia

(C) Short esophagus with intrathoracic stomach

(D) Rupture of the esophagus

(E) Bochdalek hernia

A
  1. (A) Blunt trauma is the most common cause of
    diaphragmatic rupture.

Associated injuries are common.

In blunt trauma, the left diaphragm is ruptured more frequently than the right. The stomach, spleen, colon, and omentum may enter the left pleural cavity.

Diaphragmatic injury without herniation of abdominal contents is difficult to diagnose.

Patients may present with symptoms many years after the initial trauma.

Early surgery is indicted.

408
Q
  1. A 32-year-old man undergoes a laparotomy for
    multiple organ injury resulting from trauma. He is discharged after 2 weeks in the hospital, only to be readmitted 3 days later because of abdominal pain and sepsis. The CAT scan shows an accumulation of fluid in the subhepatic space (Fig. 5–5). This space is likely to be directly related to an injury involving which structure?

(A) Inferior pole of the right kidney

(B) Stomach

(C) Uncinate process of the pancreas

(D) Aortic bifurcation

(E) Right psoas muscle

A
  1. (B) Subhepatic (intrahepatic) space infection
    usually occurs after surgery or peritonitis in
    the supracolic compartment.

It is an unlikely complication of biliary pancreatitis.

Infections in the subhepatic space may extend to the infracolic compartment via the paracolic gutter
(of Morrison).

In addition to the stomach, the subhepatic space may be involved with infection secondary to injury or diseases of the gallbladder, the first part of the duodenum, the anterior portions of the pancreas, or the liver.

The uncinate lobe of the pancreas is the part of
the head located posteriorly to the superior
mesentric artery vein.

409
Q
  1. A 38-year-old man attempts suicide by ingesting drain cleaner fluid. His family brings him
    to the local emergency department. Which of
    the following is TRUE?

(A) Copious neutralizing (acid) solutions
should be given.

(B) Emetics should be administered.

(C) Stricture formation is inevitable.

(D) Fluids and solid foods can usually be
started several days after the injury.

(E) Esophagoscopy should be performed to
visualize the distal extent of the injury.

A
  1. (D) Oral fluids and solid foods can usually be
    started several days after the injury.

Feeding at this stage is encouraged if the patient continues to show favorable improvement.

If the caustic injury is superficial, stricture formation is unlikely to occur.

Endoscopy to the proximal extent of the injury is recommended unless perforation is suspected.

No attempt should be made to pass the endoscope beyond the proximal portion of the inflammatory segment.

Emetics should not be administered because the esophagus will be reexposed to the agent when the patient vomits.

410
Q
  1. A patient is admitted to the hospital after ingesting lye. The following day he complains
    of chest pain. His pulse is 120 bpm. On physical examination he is found to have subcutaneous crepitus on palpation. His chest x-ray shows widening of the mediastinum and a pleural effusion. What has occurred?

(A) Aortic rupture

(B) Coagulation necrosis

(C) Esophageal perforation

(D) Oropharyngeal inflammation

(E) Spontaneous pneumothorax

A
  1. (C) Esophageal perforation has occurred.
    Caustic alkali ingestion results in liquefactive
    necrosis while acid ingestion causes caogulation necrosis.

The esophagus is more often involved than the stomach during alkaline ingestion (and conversely, the stomach is more often involved than the esophagus during acid ingestion).

Features on x-ray suggesting esophageal perforation include pneumothorax, pneumomediastinum, and pleural effusion.

Ewald tubes or nasogastric tubes should be
avoided because of the risk of perforation.

411
Q
  1. A 50-year-old gynecologist complains of dysphagia, regurgitation, and weight loss. She also
    states that she feels as if food is stuck at the level of the xiphoid. An upright chest x-ray shows a dilated esophagus with an air-fluid level. Which of the following is FALSE?

(A) A barium swallow will show a “bird’s beak” deformity

(B) Manometry will demonstrate that the LES fails to relax during swallowing.

(C) Upper endoscopy should be avoided because of the risk of complications.

(D) Medical treatment includes nitrates and calcium channel blockers.

(E) Intersphincteric injection of botulinum toxin can be therapeutic.

A
  1. (C) The symptoms and radiologic findings in this
    patient suggest achalasia.

Evaluation includes endoscopy to rule out a stricture or cancer.

Barium swallow will show a dilated esophagus, failure of the LES to relax during swallowing,
and a lack of peristalsis.

Nonoperative management is of limited usefulness but may be considered in high-risk patients who are not candidates for surgery.

412
Q
  1. A patient has been diagnosed with achalasia.
    He refused surgery initially, preferring to try nonoperative therapy. He tried lifestyle modification, calcium channel blockers, botulin
    toxin injection, and endoscopic pneumatic
    dilatation. None of the treatments alleviated
    his symptoms. What are his surgical options?

(A) Esophagectomy

(B) Surgical esophagomyotomy proximal to
the LES

(C) Modified Heller myotomy and partial
fundoplication

(D) Repeat pneumatic dilation using
pressures of loops

(E) Nissen fundoplication

A
  1. (C) A healthy patient with achalasia who has
    failed nonoperative management should be
    considered for surgical intervention.

Pneumatic dilatation is first-line therapy. It causes disruption of the muscular layers of the LES. A balloon is placed endoscopically at the level of the
LES. Fluoroscopy is used to visualize the
balloon as it is inflated to pressures no higher
than 10 psi.

If pneumatic dilatation fails, or if symptoms return after successful dilation, surgery should be considered.

The procedure may be done open or endoscopically. The operation involves a myotomy that divides the circular and longitudinal muscle fibers. It extends from the distal 6 cm of the esophagus, through the LES, and the proximal gastric cardia. A partial fundoplication is usually included to prevent gastroesophageal reflux.

413
Q
  1. A 50-year-old man presents to the emergency
    department with chest pain. The patient is evaluated for a myocardial infarction. The workup
    is negative. On further questioning, his symptoms include dysphagia (with both liquids and solids). Which of the following is NOT TRUE?

(A) A barium swallow will always show a corkscrew esophagus.

(B) Manometry shows simultaneous high-amplitude contractions.

(C) Initial evaluation should exclude coronary artery disease.

(D) A pulsion diverticulum may be present.

(E) Patients refractory to medical management may respond to long esophagomyotomy.

A
  1. (A) It is important to rule out coronary artery
    disease in patients who have DES because the
    symptoms may be similar.

Barium swallow and endoscopy are used to evaluate the esophagus.

A corkscrew esophagus is highly suggestive of
DES, however, it is not always seen.

Manometry is the diagnostic study of choice.

Medical management includes nitrates and calcium channel blockers.

414
Q
  1. A 60-year-old man with a long history of GERD
    has worsening symptoms. He has an upper endoscopy that shows esophagitis. A biopsy is taken that shows intestinal metaplasia. Which
    of the following is TRUE?

(A) Barrett’s esophagus is more common in women.

(B) 50% of patients with GERD have Barrett’s esophagus.

(C) High-grade dysplasia is an indication for prophylactic esophagectomy.

(D) Cells typically found in the esophagus are columnar develop adenocarcinoma.

(E) 100% of patients with Barrett’s esophagus develop adenocarcinoma.

A
  1. (C) Barrett’s esophagus is a metaplastic change
    found in 10–15% of GERD patients.

The normal squamous cells of the esophagus are transformed into columnar cells.

It is more commonly seen in men.

Patients with Barrett’s esophagus (without dysplasia) require lifelong surveillance.

Patients with severe dysplasia have a 40–50% chance of developing adenocarcinoma of the esophagus.

Prophylactic esophagectomy is recommended.

415
Q
  1. A known HIV positive patient complains of
    severe odynophagia. He avoids eating and drinking because of the intense pain, and he has lost a significant amount of weight. Which of the following is TRUE?

(A) Esophagectomy is the treatment of choice.

(B) Cancer is the only condition that can explain these findings.

(C) UGI series is not useful.

(D) Candida is the most common cause of infectious esophagitis.

(E) Esophageal candidiasis is almost certain if the patient has oral thrush.

A
  1. (D) Candida is the most common cause of infectious esophagitis.

Predisposing factors include malignancy, AIDS, and antibiotic use.

A double contrast esophageal swallow or esophagogastroduodenoscopy (EGD) can be used to make the diagnosis.

Not all patients with oral thrush have candida esophagitis; cytomegalovirus (CMV) esophagitis can also occur in these patients.

Other infecious causes include tuberculosis (TB) and herpes.

Antibiotics can be effective.

A superficial spreading carcinoma of the esophagus may have a similar appearance in diagnostic studies.

416
Q
  1. A 65-year-old man has a chest x-ray done for an insurance physical. A posterior mediastinal
    mass is seen. After a complete evaluation, he is
    diagnosed with an esophageal duplication cyst.
    Which of the following is TRUE regarding these congenital cysts?

(A) Communication with the true lumen is uncommon.

(B) Malignant degeneration is common.

(C) Most cysts are symptomatic.

(D) All cysts should be removed.

(E) Thoracoscopic excision is contraindicated.

A
  1. (A) Duplication cysts are congenital.

Communication with the true lumen is uncommon.

They are usually asymptomatic. Symptoms and
complications can include dysphagia, infection, perforation, and bleeding.

Malignant degeneration is rare.

Symptomatic cysts can be removed by open thoracotomy or videoassisted thoracoscopic (VATS) techniques.

417
Q
  1. A patient has compressive symptoms of the esophagus. He has a barium esophagram that shows posterior extrinsic compression of the esophagus. Which of the following is true?

(A) Vascular rings are acquired atherosclerotic lesions.

(B) Both the trachea and esophagus can be
affected by vascular rings.

(C) The two most common types of complete vascular rings are double aortic arch and left aortic arch.

(D) There is no role for Echo and Doppler.

(E) Surgery involves division of the esophagus.

A
  1. (B) Vascular rings are congenital.

They can encircle the trachea and esophagus and cause compressive symptoms.

The two most common types are double aortic arch and right aortic arch with left ligamentum arteriosum.

Diagnostic studies include chest x-ray, barium study, Echo, CAT scan, MRI, and angiogram.

Surgery involves division of the ring.